Download as pdf or txt
Download as pdf or txt
You are on page 1of 157

Chapterwise/Topicwise

Daily Practice Problems (DPP)


(CafenBiis
n
IF Main &
|L_ (L, Advanced

Practice concepts and improve your grip on problem-solving skills

Authored by
G. Tewani

; CENGAGE
Learning*
Andover • Melbourne • Mexico City • Stamford. CT • Toronto • Hong Kong • New Delhi • Seoul • Singapore • Tokyo
CENGAGE
Learning'
Chapterwise/Topicwise © 2016 Cengage Learning India Pvt. Ltd.
Daily Practice Problems (DPP):
Calculus ALL RIGHTS RESERVED. No part of this work covered by the copyright herein may
be reproduced, transmitted, stored or used in any form or by any means graphic,
G. Tewani
electronic, or mechanical, including but not limited to photocopying, recording,
scanning, digitizing, taping, Web distribution, information networks, or information
storage and retrieval systems, without the prior written permission of the publisher.

For permission to use material from this text or product, submit


all requests online at www.cengage.com/permissions

Further permissions questions can be emailed to


lndia.permission@cengage.com

ISBN-13: 978-81-315-2992-8
ISBN-10: 81-315-2992-4

Cengage Learning India Pvt. Ltd


418, F.I.E., Patparganj
Delhi 110092

Cengage Learning is a leading provider of customized learning solutions with office


locations around the globe, including Andover, Melbourne, Mexico City, Stamford
(CT), Toronto, Hong Kong, New Delhi, Seoul, Singapore, and Tokyo. Locate your local
office at: www.cengage.com/global

Cengage Learning products are represented in Canada by Nelson Education, Ltd.

For product information, visit www.cengage.co.in


I Contents
I

Chapter 1: Functions 1.1—1.8


DPP 1.1 Function: Definition, Domain and Range 1.1
DPP 1.2 Odd and Even Functions, Periodic Functions 1.3
DPP 1.3 Different Types of Mapping 1.5
DPP 1.4 Composite and Inverse Functions 1.6
DPP 1.5 Functional Relations and Graphical Transformation 1.7

Chapter 2: Limits 2.1-2.8


DPP 2.1 Limits: Definition 2.1
DPP 2.2 ■ Limits of Algebraic and Trigonometric Functions 2.3
DPP 2.3 Limits of Exponential and Logarithmic Functions, Limits of Functions^x)^^ Type 2.5
DPP 2.4 Limits Using L’Hospital’s Rule and Expansion Formula 2.7
DPP 2.5 Finding the Unknown 2.8

Chapter 3: Continuity and Differentiability 3.1-3.6


DPP 3.1 Continuity (Part I) 3.1
DPP 3.2 Continuity (Part II) 3.3
DPP 3.3 Differentiability 3.5

Chapter 4: Methods of Differentiation 4.1—4.7


DPP 4.1 Differentiation: Definition, Different Rules of Differentiation 4.1
DPP 4.2 Differentiation of Function in Parametric Form, Differentiation of
One Function w.r.t. Another Function, Differentiation Using Logarithm 4.3
DPP 4.3 Higher Order Derivatives 4.4
DPP 4.4 Differentiation Using First Definition of Derivatives and Miscellaneous Problems 4.6

Chapter 5: Applications of Derivatives 5.1-5.6


DPP 5.1 Tangent and Normal 5.1
DPP 5.2 Applications of Tangent and Normal 5.3
DPP 5.3 Measure of Rate 5.4
DPP 5.4 Mean Value Theorem 5.5

Chapter 6: Monotonocity and Maxima-Minima of Functions 6.1-6.7


DPP 6.1 Monotonocity and Its Applications (Part I) 6.1
DPP 6.2 Monotonocity and Its Applications (Part II) 6.3
Iv Contents
DPP 6.3 Extrema and Its Applications (Part I) 6.5
DPP 6.4 Extrema and Its Applications (Part II) 6.7

Chapter 7: Indefinite Integration 7.1-7.10

DPP 7.1 Integration of Elementary Functions 7.1


DPP 7.2 Integration by Elementary Substitution 7.3
DPP 7.3 Integration by Typical Substitution 7.5
DPP 7.4 Integration of Rational Functions 7.7
DPP 7.5 Integration Using by Parts 7.9

Chapter 8: Definite Integration 8.1-8.9

DPP 8.1 Definite Integration: Definition 8.1


DPP 8.2 Properties of Definite Integration, Definite Integration of Odd-Even and Periodic Functions 8.3
DPP 8.3 Definite Integration Using Substitution 8.5
DPP 8.4 Leibnitz Rule 8.6 . 1 'is?c.-.d0
DPP 8.5 Miscellaneous Exercises 8.8 bill. "it.-'I ..'iriiJf.iioC! . T >s i'.-fti;' i x.i <13 <1
■ ■■ ' < '

<!!!' iijuu I .JI .-.ri'/i-jti;;-;


Chapter 9: Area 9.1—9.4
is ;; j-:; T ifiLTjiht t
' l', 8.r '*•••

-K
■ ( 1.

X •\>.

DPP 9.1 Area Bounded by Curves (Elementary) 9.1 ?.! jfuj'! r.-\ : vJ;-<> ) -£.I
DPP 9.2 Area Bounded by Curves (Typical) 9.3. I i<' idqtnD jf.r-'-j-jnij'-i 3.1 <£-83

Chapter 10: Differential Equations 10.1-10.6

DPP 10.1 Forming Differential Equation, Differential Equations with .


Variable Separable, Homogeneous Differential Equations 10.1 .
DPP 10.2 Linear •Differential Equations 10.3‘
■ . '' j I ■ 8.S 'Tia
of Variable Separation 10.5
DPP 10.3 General Form ofVariable , " , ,
• . I '■ ' ‘ • r ; -if;;; ‘jfi. A ■■ ■ jk& qHG.
DPP 10.4 Applications of Differential Equations 10.6
g.s
Solutions S.1-S.78
v; ilidftDf 3 j sViltl i: ex
r ;•!! <» Ji Jui.'fv I i.c.
(li Jr.-ii £ £ Sf'ia
vjibHjjfij-i'Ti'G 8.8 -'7-G

\.b~ IJ> * •> ...J?'. .7■ i

I.F n« iJGi’uoi'jfiiU hryiyfiiU ,nuilinf?jQ :noiii5*sns":oTti<I !.£• gsta


ticCi.iinorjliiC! ..-moH uiti-iffiGH.'l j: fio'jjfifl ■.<) nviJfrrfit ibtib I
(fulfill •. o.l pui/J noiti?Ut JiyYliCI jtoipjituH ‘iOfitonZ J.i // no?j >rib'l ;n()
k'r -?.-/t'.c/itoCl 'j' 8.^
j er II: , be • ’to (loJinibC! > r'-l tit.i fiG

0.3-Jt .G
i i ,ti:i bi:<: jnsu.nf.T 1 zn crcFft

no/1 bnE ifi'jgttrj io enoinj'jilqqZ S.g


l-.C StE/l "to VP.P'.ul/ 8.3 CC-‘U
tn i 'jiilc/ neuM t.c r-icxa

V. 9-1.3 ’ anoilonju'S io jEjrnfniM findxfiM hire V^onoXonoM :0 leiqndD


!.-) I1 nf 'i; ;ilq<;/ i;<>e 'hog tm* noM X.G 3HG
f II »>n<• ■ job /iiooniniotX
- —- -.
ISJ & $ T X & 1 !'■ •nr ;i
Zi

?
4
} f,

,r it.)

J Functions
ir
, f
.fI
'■ HIj '

< r •ft noii-jftut • J: : *jd; / pj

i ’ 1 0 (,;)

DPP 1.1
i.i\ noijjfib’l ar! f .61 <

Function: Definition, Domain and Range n

tf"l. ■<.

) • j

Single Correct Answer Type


ZJ 2. If 0(.v) =
1 +0
and 5= 0(5) + 0(4) + 0(3) + ... + 0(-3)
n
1. /(x) = sin[x] + [sin x), 0 < x < — (where [.) represents the + 0 (-4) + 0(-5). then the value of S is,
\a) 5 ? (b) 11/2 (c) 6 ‘ (d) 13/2 '*
greatest integer function), can also be represented as
. . . .. 4 I ) 1 ■ ■' H. 3. For non-negative integers m and zz a function is defined as
0, 0<x<l
0<x<I follows:
: ■ i n.J t
(a)
1 + sin 1, l<x<- zz + 1 if m = Q
2
f(m,n) = - if zw^O, w = 0
1 f(m n- I)) if mtO. n*0
. 0<x<— .di
4
(b) Then the value of/( 1. 1) is
1 1 J3
I +- + -— + —
K ' 71 (a) 1 (b) 2 (c) 3 (d) 4
2’’ —<X<—
2 V? 2 4 2 4. If/: R —> Q (Rational numbers), g : R -» Q (Rational
. . ;t <— = h, (■!)
i • > numbers) are two continuous functions such that
fO, 0<x<1 V3/(x) + g(x) = 4, then (1 -/x))3 + (g(x) - 3)3 is equal
(C) to
sin 1, 1<X<-
2 (a) 1 (b) 2 (c) 3 '(d) 4
• b,‘ 5. If /and g are two functions defined on N. such that
o, 0<.v<- 2n - 1 if n is even
4 ; />0 = ’ and g(n) =j\n) +J{n + 1).
C.■ 24 £ 2zz + 2 if n is odd
(d) I,
Then range of g is
(a) (zzz g N:m = multiple of 4}
sin 1, l<x< —
2 (b) (set of even natural numbers)
(c) [in e N:m = 4£ + 3, k is a natural number)
(d) [me N:in = multiple of 3 or multiple of 4}
1.2 Calculus

6. The number of points on the real line where the function 13. Number of integers in domain of function
fix) = log| 2 lx - 31 is not defined is fix) = log^2] (4 - Ixl) + log2{ Vx } (where [.] is G.I.F and
(a) 4 (b) 5 (c) 6 (d) 7 {.) is fractional part function ofx) is
7. For relation 2 log y - log x — log (j’ — 1) (a) 0 (b) 1 (c) 2 (d) 3
(a) domain = [4, +°°), range = (1, +<») 14. The domain of the function fix) = loge{sgn(9 - x2)} +
(b) domain = [4, +°°), range = (2, +°°) 7[x]3-4[x] (where [.] represents the greatest integer
(c) domain = (2, +°°), range = (2, +«>)
function is
(d) none of these
(a) [-2, 1) u [2, 3) (b) l)u[2, 3)
8. The range of the function y = [x2] - [xj2, x g [0,2] (where
[.] denotes the greatest integer function), is (c) [4, l)u[2,3) " (d) [2, l)o [2, 3)
(a) {0} (b) {0,1} 15. Number of integers in the range of
(c) {1,2} (d) {0,1,2} X+1
fix) = — (sin-1 x + tan-1 x) + --------------- IS
9. The number of elements in the domain of the function it x + 2x + 5
x2-2x (a) 0 (b) 3 (c) 2 (d) 1
fix) = sin-1 + a/[a'] + [”*], (where [.] denotes
3
the greatest integer function) is equal to Multiple Correct Answers Type
(a) 6 (b) 4 (c) 3 (d) 5
'cos-,(71og4 x) - ~ ' 2[|sin x| + |cos x|]
10. The range of the function fix) = 16. The function^) = cos ~2 is defined
sin x + 2sinx +—
+ sin
1+x2
is equal to
I 4J
4x if x belongs to (where [.] represents the greatest integer
/ function)
(a) o,- +
k 2 JI] (b)
It It
—, — +
1.2 2 JI] (a)
L 6J
(b)
L0,-6J
it it |tt1
(c) (d) flU n
l6j (c) ----- -,2it (d) fr,2n]
L 6 .
" x2 + l ' 17. Which of the following functions are defined for all x?
11. The range of the function fix) = tan , x € R is
(a) sin[x] + cos [x] ([x] denotes the greatest integer <x)
n it £ 71 (b) sec-1 (1 + sin2x)
(a) (b) (c) tan (log x)
.6’2 .6’3

it it (d) - + cos x + cos 2x


(c) (d) None of these 8
.6’4
18. Which of the following functions is/are bounded?
12. The domain of the function /(x) = J10-J.x4-21x2 2x
is (a) fix) = [-2, 2]
1 + x2 ’
(a) (5,oo)

(b) [-VH.VH] (b) fix) = ,xe [0,2] - {1}


1-x
(c) [-5,-V21]u[>/21,5]u{0} x3 - 8x + 6
(c) fix) = ,[0.5]
(d) (-o,-5) 4x + l
(d) none of these

Answers Key
Single Correct Answer Type Multiple Correct Answers Type
1. (c) 2. (b) 3. (c) 4. (b) 5. (c) 16. (a. b, c) 17. (a, b, d)
6. (c) 7. (a) 8. (d) 9. (d) 10. (d) 18. (a, c)
11. (c) 12. (c) 13. (c) 14. (a) 15. (c)
Functions 1.3

DPP

Odd and Even Functions, Periodic Functions

r Single Correct Answer Type

1 1 K, X u
sm x + sin 3x
7. What is the fundamental period of f (x) =---------------- ?
cos x + cos 3x
1. Let G(x) = —— + — F(x), where a is a positive real (a) n!2 (b) n (c) 2it (d) 3/r
ax — 1 2J
8. If/: R -> R is a function satisfying the property fix + 1) +
number not equal to 1 and F(x) is an odd function. Which J(x + 3) = 2 for all x g R than/is
of the following statements is true? (a) periodic with period 3
(a) G(x) is an odd function (b) periodic with period 4
(b) G(x) is an even function
(c) non periodic
(c) G(x) is neither even function nor odd function
(d) periodic with period 5
(d) Whether G(x) is an odd function or an even function,
it depends on the value of a 9. Period of/» = sgn([x] + [-*]) is equal to (where [.] denotes
greatest integer function)
2. Let7(x) = ([a]2- 5[a] + 4^- (6{a}2- 5{a} + 1) x-
(a) 1 (b) 2
(tanx) sgn x, be an even function for all x g R, then sum of
(c) 3 (d) does not exist
all possible values of ‘a’ is (where [ ] and { } denote greatest
10. If F(x) and G(x) are even and odd extensions of the
integer function and fractional part functions respectively)
functions/(x) = xlxl + sinLd + xe\ where x g (0, 1), g(x) =
(a) _ 17 (b) —53
_ 31
(c) V
_ (d) V35
_ cosLd + x2 -x, where x g (0, I) respectively to the interval
(b) (c) (d)
(-1,0) then Fix) + G(x) in (-1,0) is
3. Let fix) = sin2x + cos4x + 2 and g (x) = cos (cos x) + (a) sin x + cos x + xe~x
cos (sin x). Also let period of/(x) and g (x) be and (b) -(sin x + cos x + xe-r)
respectively then (c) -(sin x + cos x + x + xe-r)
(a) T1=2T2 (b) 2T1 = T2 (d) -(sin x + cos x + x2 + xe~x)
(c)T1 = T2 (d) T1=47'2 11. Let P(x) = x10 + a-x8 + a3 x6 + a4X* + a^x2 be a polynomial
4. A continuous, even periodic function/with period 8 is such with real coefficients. If P( 1) = 1 and P(2) = - 5, then the
that7(0) = 0,7(1)= -2,7(2) = 1,7(3) = 2,7(4) = 3, then the minimum number of distinct real zeroes of P(x) is
value of tan-han(7(-5) +/(20) + cos^-lO)) +7(17)} is (a) 5 (b) 6 (c) 7 (d) 8
equal to
(a) 27T-3
(c) 2n + 3
(b) 3-2#
(d) 3 - n
5. If a and b are natural numbers and
i
L_
Multiple Correct Answers Type

12. The function ‘g’ defined by



gix) = sin (sin-1J{x})+ cos(sin"‘ y[{x}) - 1
fix) = sin (/a2-3)x + cos(//r + 7)x is periodic with (where {x} denotes the functional part function) is
finite fundamental period, then period of7(x) is: (a) an even function
(a) # (b) a periodic function
(b) 2# (c) an odd function
(d) neither even nor odd
(c) 27tiy]a2-3+ylb2 + l) 13. Let/be a differential function such thatTfx) =fi2-x) and
g(x) =7(1 + x) then
(d) ^(7^-3 +T^2 + 7) (a) g(x) is an odd function
6. Period affix) = sin 3x cos[3x] - cos 3x sin [3x] (where [ ] (b) gix) is an even function
denotes the greatest integer function), is (c) graph of/(.v) is symmetrical about the line x = I
(a) 1/6 (b) 2/3 (c) 5/6 (d) 1/3 <d) r(o=o
1.4 Calculus

Answers Key
Single Correct Answer Type Multiple Correct Answers Type
1. (b) 2. (d) 3. (c) 4. (d) 5. (b) 12. (a, b) 13. (b, c d)
6. (d) 7. (b) 8. (b) 9. (a) 10. (c)
11. (a)

I•
I i -< / i-j i J
7.
Hr ■ . ! ' J ij. ip'J It.-il
•: i!';!
>■ <■!!./. : ri., . i •..
•;< .ns -s hg >; k.:
•'It:'. :sf’ is?’ ,-! i ■ >

!ij: ■
h; ".j i.ciisn.ij hb(- fie zi (>.•<) tncH'jil // <b‘
1 . >;;iL7 •• I '? zi.u'• ]j

1
«; ■

L.-.h • >.1 A
• ■ ‘ ■■ •

• -fl • . s is
:;t L(!:, s'lirjfir 1 i*j; Jni
.I' ! ’
A

{'! ft ' J ! .E
m; • >‘Vj

<-* i

t; ! 'Jill ■ .'-.H' HnjhlOS A J


J --if : in'll
■ C ■
i;' ■ fi,

<:l imps
:d 1 . <r>

uf: MJ.’.fi Otf; A bnn i: ‘ I .c

;i ■■ b ■ I : JfiSfii.i.;:; J! L.iioH
»i

■k r. !' i i

i
r.

■ •' > -

■ i • is
Functions 1.5

DPP
Different Types of Mapping . S/J?. . maoGfno i

7. Let/:/?*—>{-1,0,1}definedby/(x) = sgn(x-x4+x7-x8-1)
Single Correct Answer Type .‘j
where sgn denotes signum function. Then/x) is
1. Let / : R -» [1, °°) be defined as ,/ (x) = (a) many-one and onto (b) many-one and into
(c) one-one and onto (d) one-one and into
logi0(73x2 - 4x + k + 1 +10). If/(x) is surjective, then 8. Which of the following statements are incorrect? 1 • 1
(aU=| , (b)l<| I. If f(x) and g (x) are one-one then fix) + gix) is also
•u „ one-one.
( )'l • (!■ . . \ I .. - •
II. If / (x) and g (x) are one-one then / (x) • gix) is also
<c)*>4 . , (d)*-l i one-one. .- . ■ - ■; .£
III. If/(x) is odd then it is necessarily one-one.
‘•’ 2. Let/: (-oo, 0](l, o°) be defined as f - (a) I and II only • (b) II and III only
fix) = (1 + 4-x) - i-J-x - x), then/(x) is (c) III and I only (d) I, II and III
-■ (a) injective but not surjective 9. Which of the following functions is one-one?
(b) injective as well as surjective
(c) neither injective nor surjective i: ;.. j (a) /: /? -> R defined as/(x) = e•sgnx + /
(d) surjective but not injective ,x2+ix|
(b) ../: [— 1, > (0, °°) defined by f (x) = e
3, Let A be a non-empty set of real numbers and/: A —»A
. be such that///)) =x, V x e R. Then/x) is (c) f: [3,4] —> [4, 6] defined by/(x) = lx- 11 +lx-21 +
(a) a bijection Votnjjon >\ gnix/olioi •Jib to
lx-~ 31 + lx - 41
(b) one-one but not onto (d) fx) = 7 In (cos (sin x))
(c) onto but not one-one
(d) neither one-one nor onto
n-.rb :• > .aix, •...'■j T'ldiJrj/"! 'i ■ ? < Y' • I Comprehension Type
4. f:R->R defined by/x) = — xlxl + cos X + 1 is
For Questions 10-12 - '■
(a) one-one and onto (b) one-one and into
Let fix) = x2 - 2x - 1 V x g R. Let/: (-«>, a] —» [b. «>), where a
(c) many-one and onto (d) many-one and into
is the largest real number for which/(x) is bijective.
2x + a2, x>2
10. If/: R -» R. g (x) =/(x) + 3x - 1, then the least value of
5. Let f. R ax
R, fix) - . If f(x) is onto
—+ 10, x<2 function y = #(l x I) is
. 2 .i i
'(a) -9/4 ‘ (b) -5/4 '(c) -2 (dj -1 ‘
function, then a belongs to 11. If/: [a. oo) _> [b, oo/ then/_,(x) is given by
(a) (1,4]' ; :- : - (b) (-2,3)
(c) (0,3] (d) (2,5] (a) 1 + ^+2 (b) l-V^+3
(c) 1 - y/x+2 (d) 1 + y]x + 3
6. Let0,—'. defined as/(x) = cor^x2 - 4x + a).
I 3J 12. If/: R -+ R, then range of values of k for which equation
Then the smallest integral value of a such that fix) is into •fr./(Ixl) = k has 4 distinct real roots is . > .■
function is (a) (-2,-1) (b) (-2.0)
(a) 2 (b) 4 (c) 6 (d) 8 (c) (-1.0) (d) (0. 1)
73/? —----- “* ----- ------------------- ------ ————

Answers Key ----------


Single Correct Answer Type Comprehension Type
1. (a) 2.2. (b)
(b) 3. (a) 4. (a) 5. (c) 10. (c) 11. (a) 12. (a)
6. (b) 7. (b)
7. (b) 8. (d) 9. (c)
1.6 Calculus

DPP
Composite and Inverse Functions

8. Let a > 1 be a real number and/(x) = log^x2 for x > 0. If


Single Correct Answer Type j /■' is the inverse function off and b and c are real numbers
i then f~\b + c) is equal to
14 1 7 |7
1. Ifg(x)=l4cos x-2cos2x ——cos4x-x , then the (a) /'((>)-PM (b) f~\b)+f-\c)
value of g(g(100)) is equal to 1 1
(a) -1 (b) 0 (c) 1 (d) I00 (c) (d)
f(b + c) f-\b) + f~\c)
3
2. If fix) = x2 + x + — and g(x) = x2 + ax + 1 be two real
4 x+1 if x<l
9. If the function /(*) = and
functions, then the range of a for which g(/(x)) = 0 has no 2x + l if l<x<2
real solution is
x2 -l<x<2
(a) (—,-2) (b) (-2,2) g(x) then the number of roots of
(c) (-2,oo) (d) (2,-) x+2 2<x<3
3. If domain of /(x) is [1, 3], then the domain of the equation/g(x)) = 2 is
/log/x2 + 3x - 2)) is (a) 4 (b) 3 (c) 2 (d) 1
(a) [-5,-4] o [1, 2] (b) [-13,-2] o —,5
,J 10. Suppose/(x) = ax + b and g (x) = bx + a, where a and b
L5 J are positive integers. If/g(20)) -g(/(20)) = 28, then which
(c) [4, l]u[2,7] (d) [-3,2] of the following is not true?
X FX (a) a= 15 (b) a = 6
andlet
4. Let/(x) = ----- and g(x)==------- . Then the number of
letg(x)
1 + x----------------- 1-x (c) 6=14 (d) 6 = 3
values of r such that/(g(x)) = g (f(x)) for infinitely many 11. If/x) is an invertible function, and g(x) = 2/x) + 5, then
real number x is the value of g-1(x), is
(a) 1 (b) 2 (0 3 (d) 5
1
ax + b (a) 2/-1(x)-5 (b)
5. Let/x) = then f [/(x)] = x provided that 2/-'(x) + 5
cx + d
(a) <7 = - a (b) d = a (c) |/-1(x) + 5 x- 5
(c) a = 6 = 1 (d) a = b = c =■ d = \ (d) ~2~
6. If/: R R,fix) = x3+ 3, and g : R -> R, g(x) = 2x + 1,
then/"’og-1 (23) equals -x + 1, x<0
(a) 2 (b) 3 (c) (14)l/3 (d) (15),/3 12. If /(x) = - then the number of solutions
l-(x-l)2, x>l ’
I 1 1
7. Ifflx) = x(x- 1) is a function from — —\ 00 to — 00 of/(x)-/'l(x) = 0is
then {x e 7? :/-,(x) =/x)) is .2
*•- 4
(a) 0 (b) 2 (c) 3 (d) 4
(a) null set (b) {0,2}
(c) {2} (d) a set containing 3 elements

Answers Key
Single Correct Answer Type 6. (a) 7. (c) 8. (a) 9. (c) 10. (d)
1. (d) 2. (c) 3. (a) 4. (b) 5. (a) 11. (d) 12. (d)
Functions 1.7

DPP 1.5
Functional Relations and Graphical Transformation

Single Correct Answer Type L Multiple Correct Answers Type


J
1. Let f\ x + —
n
x — = 2/(x)/| - |Vx, y € R,
io. If a function satisfies (x - y)/(x + y) - (x + y)/x - y) =
2(x2y - y3) Vx, y g R and AI) = 2, then
I y. y) \y)
y*0 and A0) = 0 then the value ofAD +A2) = (a) /x) must be polynomial function
(a) -1 (b) 0 (c) 1 (d) none of these (b) A3)=12
2. If Ax2 - 6x + 6) +Ax2 - 4x + 4) = 2x Vx g R then A~3) +
(c) A0) = 0
(d) Ax) may not be differentiable
A9)-5AD =
(a) 7 (b) 8 (c) 9 (d) 10 11. A function /, defined for all x, y g R is such that AD = 2;
3. If a function f:R-*R be such that Ax - A?)) =/Ax)) + A2) = 8 and/(x + y) - kxy =f{x) + 2y2, where k is some
x/j) +Ax) - I, V x, y e R, then A2) = constant, then
(a) 1 (b) 3 (c) -1 (d) none of these (a) A0) = 0 (b) AO) cannot be determined
4. If/: R R is a function satisfying Ax + y) = Axy) for all (c) k = 2 (d) k cannot be determined
12. Suppose that/:/?-»/? is continuous and satisfying the
MA 33 f 99 A
x, y e R and / — = —, then / — = equation Ax) /(/(x)) = 1, for all real x. Let/(1000) = 999,
<47 4 1<16
14 ' then which of the following is/are true?

5.
(a) I (b) ’
16
(0 f (d) 0
A function/: R —> R satisfies the equation Ax)/y) -fixy)
(a)
(a) /(500) =^
/(500) = —

/(199) =
(b) /(199)
(b) =—^
= x + y Vx, y g R and AD > 0, then
(a) Ax)T1(x) = x2-4 (b) Axir1w=.r2-6 ' 1 „ „ _
(c) AD/“‘(x)=x2- 1 (d) none of these (c) /(x) = -Vxg/?-{0}
x
6. Let / be a function defined from R*. If (/(xy))2 = 1
x(AD)2 for all positive numbers x and y. Iffi2) = 6, find (d) /(1999) =
1999
A50) = ? 13. The graph of a function/(x) which is defined in [-1,
(a) 20 (b) 30 (c) 5 (d) 40 4] is shown in the adjacent figure. Identify the correct
7. Suppose/is a real valued function satisfying Ax +A-D) - statement(s).
4 Ax) and A D = 4. The value ofA21) is ■Ax)
(a) 16 (b) 64 (c) 4 (d) 44 2-
8. The graph of a function y = g(x) is shown in the following 1--
figure. If/(x) = -3X2 - kx - 12, k g R and Ag(x)) > 0 V 4. O’ X
x g R then least integral value of k is equal to 0 1 2 4
g(x)-‘
0 ■x
(a) domain of/(Ixl - 1) is [-5, 5]
_______ Li______ (b) range of/(Ixl 4- 1) is [0,2]
(c) range of/(-1x1) is [-1,0]
-2
(d) domain ofAM) is [-3,3]
(a) 13 (b) 14 (c) 15 (d) 16 14. The graph of y = l/(x)l is as shown in the following figure
9. Let/: I ->I be a function (/ is set of integers) such that
A0) = L/A0) =Afl» + 2) + 2) = n. Then
(a)A3) = 0 (b)A2) = 0
(c) A3) = -2 (d) /is many- one function -2 O 1
1.8 Calculus

Graph of y =A') may be Comprehension Type


L. - -
For Questions 15 and 16--------------------------- — -----
Consider a differentiable function f : R -» R for which
i (a) /(1) = 2 and/(x + y) = 2'Av) + 4> AO V x, y g R.
-2 o 1 :■ i <j »}| MS? (I f. j . 15. The value of/(4) is ‘ k S r >' I- Jsi- :
(a) 160 (b) 240
(c) 200 (d) none of these
16. The minimum value ofAO is

(b) 1
(a) 1
j ? ;•>
.<bH ■
■ .V.;. \C
1
(0 -- ; (d) none of these
\ ■■ • ‘
•A . -■ ' rfX ’ ■ : -

', ‘..i • i ■ •' For Questions 17 and 18


f: . ’• i'
Let Av) be a real valued and differentiable function on R such
(c) o 1
■ : J ; \ I-/«•/(;<) "
17. A» equals . 'f'
• si > dj
(a) 1 (b) 0 . (c) -1 ” (d) norieofthese
< ii : ,
; .:’' J r’ ’: .y t -is-Aois ■ ■ f *A■ > •< . < V . i
A"
- (d) (a) odd function■ ,
o ;i: 1-^
: -j ■ r (r". fib'
(b) even function ...„
(c) odd and even function simultaneously
■ . • * •
(d) neither even nor odd , <

0 ■'
Answers Key
-- A : ; r.-.'Z'fiL. A .c
Single Correct Answer Type Multiple Correct Answers Type f ./7 ! -I

1. (b) 2. (c) 3. (c) 4. (a) 5. (c) 10. (a. b, c) :. , , 11. (a, c) ,12. (a, b)
6. (b) 7. (b) 8. (c) 9. (c) 13. (a. b. c) . 14. (a, b, c, d) )' ',( ■ !\ f l!

Comprehension Type ’ ’1 1 ‘,n \ r>. 1 .«)


• L-
’■ • ;
15. (b) ~ 16. (c) 17. (b) 'is:
Uf fii A'- h;I
i> ■; >; .<':■/<' A

>, ■<- li!1e - Ji/’.'- (1 A or c . />f


• Avi U) fl) <>! (C>
,.‘;r! : f| /.* •• - ' ij i. ic ,k;,;; > •’

•• c o' .) • 'a .2! - ■ ; - L* 'jlja!'


oi !a i > ’ Lf jmi nsrii ‘A 1 r
A; V;
0
I
: f ’ <> : < ' O’.'."I 'Hi --------- ..__ji- ________ __ _____
;! i ! ’ ■
1 :>)
fl •.!/•

A*

r
C I® A Ip ipII1
1
f I J!

Limits I

■ 4 1

- V. ■ 7(1 • I ■H
■fi

’’ - A
l - 1 >I !!’!jjj> <■ [ (Li

;. > u GJ
n*>ri > ;I
iO. ) 1- 1 '
DPP
" >/
Limits: Definition
Tv-h. PT..'
'< I

.'Cfi fliii'.il-:')

■ r/1 ■ i.

r “-"4
Single Correct Answer Type -J ,
3.
.. [x]2+15(4+56 _
nm------------------------ (where [.] denotes the greatest
lim
. x-»-7sin(x + 7)sin(x + 8)
1. If/: /?->« is defined by fix) = (x-3] + lx - 41 for a e R, integer function)
then lim f(x) is equal to (where [.] represents the greatest (a) is 0 (b) is I
(c) is -1 (d) does not exist
integer function) 6. Which of the following limit exists finitely? - -
(a) -2 (b) -1 (c) 0 (d) 1
[sinx] - [cos x] +11 ' , (a) lim (x)log'x .vhVh- ....
2. lim — ------- = (where [•] denotes the greatest ->o+'
X- ' V
X
2 .!■ ? I... .S,i
(b) lim
x :2-9~7-
x2-6x + 9
(
integer function) l-v-ll-2 "
(a) 0 (b) 1 • t
i
(c) -I (d) does not exist
(C) lim([x]) = (where [.] denotes the greatest integer
3.
1F-11
lim------ = (where [•] denotes the greatest integer
— x function)
x-> 3
(d) none of these
function) 7. Let L. = lim (x - 6)r and L-, = lim (x - 6)4.
(a) -9 (b) -12 (c) -6 (d) 0
Which of the following is true?
1
x + -, x<0 (a) Both and L, exists
2 (b) Neither L} nor L, exists
4. If/(x) = , then pirn /(x)j = (where [■]
2x + —, (c) L} exists but L, does not exist
4 (d) L, exists but L, does not exist
denotes the greatest integer function) 1
8. Set of all values of x such that lim — 4n
3 4 tan”’(2^x)
<a)l (b) - 1+
4 n
(c) does not exist (d) none of these is non-zero and finite number when n e N is
2.2 Calculus

(a) lim/(x) = -l (b) lim /(x) = 0


(a) 0,— (b) 11 x->0+
2k 7t' k
(c) lim/(x) = -l (d) lim/(x) = 0
(c) _L _L (d) ,0 x-»0 x—>0
. 27t’27T. 2k
02 + 0-2 /(0) <
14. Assume that lim /(0) exists and <
9. lim x - log,
^e* + e 0+3 e2
02 + 20-l
(a) log,4 (b) 0 (c) log, 2 (d) *» holds for certain interval containing the point
0+3
. 1
10. lim {(<?* + K*)1 = (where {.} denotes the fractional part
0 = -l then lim /(0)

of x) is equal to (a) is equal to/(-I) (b) is equal to 1


(a) K-e (b) k-3 (c) e-2 (d) 3-e (c) is non-existent (d) is equal to -1
cos X
11. If is a periodic function, then ic I . j(x)= tan (tanx)
rz \ I-lim------------- 71
sin ax 15. Let x*(2m + 1)- then
n-1 + (loge x)"
lim lim (1 + cos2"1 nlna) is equal to
m—>°° n—
K
(a) 0 (b) 1 (c) 2 (d) -I (a) V 1 < x < —, /(x) is an identity function
2
Multiple Correct Answers Type (b) v — < x < n, the graph of/(x) is a straight line having
2
n. If iim si"''<sin x) y intercept of -n
and 5 = lim , then
•r->0cOS (COSX) x—»0 X k
(c) V — < x < e , the graph of/(x) is a straight line having
(a) A=1 (b) A does not exist
y intercept of -k
(c) 5 = 0 (d) 5=1
' gM+kl2
(d) Vx>e,/(x) is a constant function
13. If/(x)=x then (where [•] represents the greatest
k w+w
integer function)

Answers Key
Single Correct Answer Type Multiple Correct Answers Type
1. (c) 2. (a) 3. (c) 4. (c) 5. (a) 12. (b, c) 13. (a, b) 14. (a, d) 15. (a, c, d)
6. (c) 7. (d) 8. (c) 9. (c) 10. (b)
11. (c)
Limits 2.3

DPP

Limits of Algebraic and Trigonometric Functions

Single Correct Answer Type 3 tan 3x - 4 tan 2x - tan x


8. lim
x->0 4x‘ tan x
- cos x2 (a) 0 (b) 1 (c) 3 (d) 4
1. The value of lim is
x-»6 1 - COS X sin ‘x
9. lim = (where [.] denotes the greatest integer
■*-*o[tan‘ x.
(a) |
(b) 2 function)
(a) 0 (b) 1
(c) 2 (d) None of these (c) -1 (d) none of these
•Jl - ^/sin 2x
2. lim (1-sinx) tanx = 10. The value of lim is
n £ zr-4x
X->2 X->4

(a) 2 (b) 1 (c) 0 (d) ©o


(a) I «»4
(0 1 (d) does not exist
1
3. The value of lim x2 1- cos—
X
is 11. The value of lim is
X—>°°

(b) 1/4 (c) 1/2 (a) 0 (b) e (c) l/e (d) —oo
(a) 0 (d) 1
tan3 x - tan x
4. lim^(^(x+l)2 -V(x-1),22] = 12. The value of lim is
x—>rt74
X->«> \ / cos I x + —
I 4
(a)| 4 (a) 8 (b) 4 (c) -8 (d) -2
(c) 1 (d) -
3
(1 - sin x) (8x3 - tt3)cos x
13. lim
£ (^-2x)4
3-2”+1 - 4-5"+l X->2
5. lim
5-2" + 7-5" ar (b)^ 2
7C 2
(a) —— (c) (d)
(a) 0 (b) 3/5 (c) -4/7 (d) -20/7 16 16 L6 16
sin(x- 2) io
6. lim {x} = (where {.) denotes the fractional part ,2010
(x-2)2
function) 14. lim —
(x 1006 +1) (2x1004 + 1)
(a) 0 (b) 2
(c) 1 (d) does not exist 502
(a) 5 (b) 2010 (c) (d) 0
1005
cot-
7. lim----
1 -Jx]
/(I - cos x)
X->“>
sec
! [px+iyl 15. If lim
/(*)
= a and lim , b (where b^0),
x-»0 x2 x-»o g(x)sin‘ x
H x-i J
(c) 71/2 then litn^i^x) is
(a) 1 (b) 0 (d) nonexistent
x->0 x
2.4 Calculus

4a 2n 2n
(a) — (b) — kit kit
b 4/>
17. If k e / such that lim cos— cos— ! =0, then
4 6
(0 _ - ------ (d) None of these (a) k must not be divisible by 24
(b) k is divisible by 24 or k is divisible neither by 4 nor
by 6
X (c) A' must be divisible by 12 but not necessarily by 24
! x>0 .. .■»

: 16. Ifflx)= <{ sin x andg(x)= x2 - 2x -2, !<x<2 (d) None of these ' ' ' C
2-x, x< 0 x-5, x>2 18. If an and bn are positive integers and an + \[2bn = (2 + V2)M,

Then the value of lim g(/(x)) then lim


.v—>0 n—>«
b,J
(a) is-2 (b) is —3 '.; :<■

(c) is 1 (d) does not exist (b)‘ 2j!’nfX (c)V'e^f •• ((d)


‘ e2
(a) 2
.■) ( •«

- Answers Key
Single Correct Answer Type - .in
11. (d) 12. (c) 13. (d) 14. (a) 15. (c)
1. (c) 2. (c) 3. (c) 4. (d) 5. (d) - 16. (b) 17. (b) . 18. (a)
6. (c) 7. (a) 8. (d) 9. (b) 10. (d) (><1

. ’.a. ?:
( j)

I
I

0
; (d >
t •

£
)} p i
( • ’ t 'Z
’ !)

---- r.;:. .-..I

■ m!

' 7.
■);
r-t (•/> (d;

t£--?;/>•■
h: rjjjjM ,ii! • I . j '.''J!'.- ■ mn
;m! .i-J

(• K)'l i <•/!
I
A mi!
.(0 . v - — •_ — Qjd V;-. 11 z?
<• ' n i-
Hf /
■I
non (b) 0 (ci; ! i if >
I
Limits 2.5

i
DPP 2.3
Limits of Exponential and Logarithmic Functions,
Limits of Functions Type

1/x 1/x
Single Correct Answer Type /(*)' f(x)'
9. If lim l + x + = e,3 , then lim 1 +
1
x—>0 X . x—>0 x .
tan x5 log (l + 5x) (a) e (b) e2
1. The value of lim11------ -i r i is (c) e1 (d) none ofthese
o+(tan Vx)2 10. lim [1 + (cosx)cosx|2 =
rt~
X->2

(a)l (b) I
(b) -
(a) Does not exist (b) 1
(c) 1 (d) none of these (c) e (d) 4
/
(x3+27)loge(x-2) <7-1+6 n
2. The value of lim is 11.. If a > 0, b > 0 then lim
x2-9 a 7
(a) 9 (b) 18 (c) 27 (d) 1/3
(a) ba (b) a> (c) ab (d) 6*
loge(l-2x2)

3. The value of lim


9
1 - cos (sin x)
\
sin2x
12. If/(x) = lim | cos-
x ]
, then the value of lim
rw-i
is
x->0+ k ? J n—>oo 77 x-»0 X
is
(a) 0 (b) e (c) -1 (d) ~
(a) 0 (b) 1 (c) 2 (d) 3/2
1 - cos3x logc(l +4x) .2
4. hm — is equal to log J ex + 2
x^o x- sin-1 (ex - 1) tan-1 (2x)
13. lim is equal to
(a) 2 (b) -4 x—>0 tanVx
(c) 6 (d) 4
5. If graph of the function y =flx) is continuous and passes (a) 0 (b) 1 (c) e2 (d) 2
loge(3/(x)-2) 14. Let/: R -» R be such that/(a) = l,/(a) = 2. Then
through point (3, 1) then lim is equal / xt/x
x—»3 2(1-/W)
lim is

(«) 4
x—>0 /(*) 7
• (a) 43 (b) > (c) 3
t \ --
(a) e2 (b) e4 (c) (d) 1/e
2 2
6. Let /(x) be defined for all x g R such that yjn2 + n - 1
1 15. The value of lim is
lim /(x) + log 1- -log(/(x)) = 0.Then/(0)is n 7
x—>0 I e/(x) (a) e (b) 1/e (c) e2 (d) e-2
(a) 0 (b) 1 (c) 2 (d) 3 /
■ • X . • X I ( , .X x X

I It 16. If f(n) = lim 1 + sin — 1 + sm — ... 1 + sin —


7. lim x2 sin logt, cos— x->0 2 22 2”
X—>°° X
then lim J(n) =
2
(a) 0
™4 (c)--- --
7T
K
4
(d) —
7T
(a) 1 (b) e (c) 0 (d) <*

x+c 17. lim (1 - x + x- ^e)n is equal to


8. If lim = 4 then the value of ec is
X—>°° x—c
(a) e' (b) e~x
(a) 1/4 (b) 1/2 (c) 1 (d) 2 (c) e~x (d) none of these
2.6 Calculus

Answers Key
Single Correct Answer Type 11. (a) 12. (a) 13. (d) 14. (b) 15. (b)
1. (b) 2. (a) 3. (d) 4. (b) 5. (c) 16. (b) 17. (a)
6. (a) 7. (c) 8. (d) 9. (b) 10. (d)
Limits 2.7

DPP
i Limits Using L'Hospital's Rule and Expansion Formula

-----•*
JL
L Single Correct Answer Type l + 2x\2 i
6. The value of lim •e1 is
13/
x-1 + 3xy
x - iTx a „ „
1. The value of lim is (a) e2 (b) e2 (c) e~2 (d) 1
x
7. If/: /?—>/? be a differentiable function at x = 0 satisfying
z x 44 45 40 fiO) = 0 and/(0) = 1, then the value of
(a) — (b) (c) — (d) 77
91 91 89 93
liml£(-ir/f-U
2. lim^l0gx7
*->i l-2x + x2
(a) 0 (b) -log 2 (c) 1 (d) e
(a) 1 (b) -I (c) 0 (d) -1/2 q tv, i c r 1 + ^/tanx
8. The value of hm ----- -—z— is
1 - cos 2x X^L 1 - 2 cos" x
3. The value of lim 4
x->0 e - ■
(a) -1/2 (b) —2/3 (c) —3/2 (d) -1/3
(a) 1 (b) 2 (c) 4 (d) 8
9. Letg(x) =
(x-ir ; 0 < x < 2 m and n are integers,
log cos'"(x-l)’
1 ■a f(a + 2h2)-f(a-2h2)
4. If/(a) = —, then lim m * 0, n > 0 and. If lim g(x) = - I, then
4 f(a + h3-h2)- /(a -h3 + h2)
(a) n=l,m=l (b) n=l,/n = -l
(a) 0 (b) 1 (c) n = 2, m = 2 (d) n > 2, rn = n
(c) -2 (d) none of these

1 i i Vx _i Vx |
i Comprehension Type
5. lim —a tan ——-Man — has the value equal
-»0+ xfx a b For Questions 10 and 11
to Let fix) be the fourth degree polynomial such that /(0) = -6,

(a)
a-b
(b) 0
— . .. t f(x)
/0) = 2 and lim = 1
«I(X-1)2

(a2~b2) a2 - b 2 10. The value of/2) is


(c) (d) (a) 1 (b) 0 (c) 2 (d) 3
6a2b2 3a2b2
11. The value of/(2) is
(a) 4 (b) 5 (c) 6 (d) 7

Answers Key
Single Correct Answer Type Comprehension Type
1. (b) 2.
2. (d)
(d) 3. (c) 4. (c) 5. (d) 10. (c) 11. (c)
6. (a) 7. (b)
7. (b) 8. (d) 9. (c)
2.8 Calculus

DPP

Finding the Unknown

r: Single Correct Answer Type

1. Number of integral values of A for which


□ Multiple Correct Answers Type

aex + b cos x + ce X
f A2 A2 5. If lim — = 4, then
Hm sec does not exist is
x—>0 elx~2ex + \
, log,, X X -1
(a) a = 2 (b) b = -4
(a) 1 (b) 2 (c) 3 (d) 4 (c) c = 2 (d) a + b + c = -8
eax -e -x tan ([x3]-[x]3)
2. If lim — = b (finite), then 6. If a e I, then value of a for which lim
x-»0 .r~ (x-a)3
exists finitely, is/are
3 (a) 0 (b) 1 (c) -1 (d) 2
(a) a = 2, b = 0 (b) a = 0, b = |

(c) a = 2, b = —
3
2
(d) a = Q,b = 2

For Questions 7 and 8
Comprehension Type

sin (sin x) - sin x 1


v3 L = lim
3. If lim -y— ■ ------------ = 1, a > 0. then a + b is equal x->0 ax5 + bx3 + c 12
yja + x (bx - sin x)
7. The value/values of a is
to
(a) e R (b) 2 (c) 0 (d) 1
(a) 36 (b) 37 (C) 38 (d) 40
8. The value/values of b is
a/x 1 Y (a) e R (b) 2 (c) 0 (d) 1
4. If lim x log,>e Q 1/x P = -5 , where a, p, yare For Questions 9 and 10
1 0 1/x 7
(x2 + ox +1) + x2”(2x2 + x + 6)
finite real numbers, then If /(x) = lim and lim /(.r)
)oo 1 + x2" x—>±1
(a) a = 2,p = 1, ye R exists, then
(b) a = 2, p = 2, y= 5 9. The value of a is
(a) -1 (b) 1 (c) 0 (d) 2
(c) ae R.p= 1, ye R
10. The value of b is
(d) ae R, p = 1, y= 5 (a) -1 (b) 1 (c) 0 (d) 2

Answers Key ----------


Single Correct Answer Type Comprehension Type
1. (c) 2. (c) 3. (b) 4. (d) 7. (a) 8. (b) 9. (b) 10. (c)
Multiple Correct Answers Type
5. (a, b, c) 6. (a, b)
L© A IP i f il LB

Continuity and
Differentiability

DPP 3.1
Continuity (Part I)

4. For which of the following functions fiO) exists such that


Single Correct Answer Type
L__ fix) is continuous at x = 0?
ab/fi 1 (sin x|
( 2x2 (a) fix) = (b) fix) = cos
sin---- + cos i , x*0 . lOge W x
1. If/W = \ a I , is continuous
7t 1__
e\ x=0 (c) /(x) = xsin — (d) fix) =
x I + 2C0U
at x = 0 V b e R then minimum value of a is /(-V)
5. Let/x) = x3 - x2 - 3x - 1, g(x) = (x + I )a and h(x) =
(a) -1/8 (b) -1/4 g(x)
(c) -1/2 (d) 0 where h is a rational function such that
2. Let/: R -» R be any function. Also, g: R -» R is defined by (i) it is continuous everywhere except when x = - I,
g(x) = l/(x)l for all x. Then g is (ii) lim /z(x) = o° and
(a) onto if/is onto X—

(b) one-one if/is one-one 1


(c) continuous if/is continuous (iii) lim h(x) = -
' x->-i 2 ‘
(d) None of these
The value of /i(l) is
1 - sin nx (a) 1/2 (b) 1/4 (c) -1/2 (d) 1
1 + cos 2zrx X 2
3x2 + ax + a + 3
1 6. If the function fix) = is continuous at
x2 + x-2
3. Letflx) = p' * “ 2 • Then the value of
x = - 2, then the value ofy(-2) is
J2x -1 (a) 0 (b) -1 (c) 1 (d) 2
X 2 1
+ 2x - 1 - 2
7. Let/x) = 8 ’ 'r<0 (where [.] denotes the
1 a[x], a e R - {0}, x > 0,
p for which the function is continuous at x = — is
2 greatest integer function).
(a) 1 (b) 1/4 Then/>) is
(c) 4 (d) none of these (a) continuous only at a finite number of points.
(b) discontinuous at a finite number of points.
3.2 Calculus
(c) discontinuous at an infinite number of points. 3
(d) discontinuous at x = 0. (a) -7(0) (b) 28/5/(0)
2 cos x - sin 2x .n
’ A"2 64
U-2x)2 (c) y/0) (d) none of these
8. If Av) = then which of the
it I 1 x
8x-4tt ' A> 2 1 - cos 1 — cos —
14. If /(x) = -
V 2 , x^O is continuous al
following holds? 2"'x"
(a) /is continuous at x = n!2 1, x=0
(b) /has an irremovable discontinuity at x = nil x = 0 then the value of in + n is
(c) /has a removable discontinuity at x = nil (a) 2 (b) 3 (c) -3 (d) 7
(d) None of these
sin^|(x-[x])j, x< 5 Multiple Correct Answers Type

9. If Av) = p(/>-l), v = 5 is continuous at 15. Which of the following functions is/are discontinuous at
2 |x2-11X +241 x=l?
ab , x>5 1
x-3 (a) /(x) =
1 + 2,anr
x = 5, a. b g R then ([■] denotes the greatest integer
function) 1
(b) g(x)= lim
, x 25 L 6 1 +/? sin2(7rx)
zux 6 . 17
(a) a = —,b = - (b) a =—,b- —
108 5 13 29
(c) /?(x) = 2 ,x 1 and /z(l) = 1
. . I . 25 23 j 6
(c) a = —,b = — (d) a----- ,b = —
2 36 100 5
(d) 0(x) = ------- —— ----- 7, x 1 and 0(1) = 1
10. The function Av) is discontinuous only at x = 0 such that |x —1| 4-2(x — I)2
/(x) = 1 V x e R. The total number of such functions is
16. / is a continous function in [a, b] and g is a continuous
(a) 2 (b) 3
functin in \b, c]. A function /i(x) is defined as
(c) 6 (d) None of these
/(x), xg[a,6)
*(x) =
n. ao = . ( v2 + e2~ ' '
r 7
is continuous from right at g(x), xe(b,c]
k, x=2 Now, iff(b) = g(b), then
the point x = 2, then k equals (a) /r(x) may or may not be continuous in [a, c]
(a) 0 (b) 1/4 (b) h(b+) = g(b~) and /?(/?“) =f(b+)
(c) -1/4 (d) none of these (c) h(b~) = g(b+) and h(b+) = fib-)
. 12. The number of points of discontinuity of g(x) =//x))where (d) Zz(x) has a removable discontinuity at x = b
1 + x, 0<x<2 3, x=0
A0 is defined as, /(x)
3 - x, 2 < x < 3 17. Ifthefunction/x) defined as/x) , 3\
ax + bx
1+ , x>0
(a) 0 (b) 1 (c) 2 (d) >2
(1 28a + ar)1/8 - 2 is continuous at x = 0, then
13. If the function AO = is continuous at
(32 + Z>x)1/5-2 (a) a = 0 (b) b = e3
(c) a = 1 (d) b = log, 3
x = 0, then the value of — is
b

Answers Key
Single Correct Answer Type Multiple Correct Answers Type
1. (b) 2. (c) 3. (d) 4. (d) 5. (c) 15. (b,c,d) 16. (c,d) 17. (a, d)
6. (b) 7. (c) 8. (b) 9. (a) 10. (c)
11. (b) 12. (c) 13. (c) 14. (c)
Continuity and Differentiability 3.3

DPP
Continuity (Part II)

(d) / is continuous for rational x and discontinuous for


Single Correct Answer Type
irrational x
acotx 6. If/x) = [x] (sin kxf is continuous for real x, then (where
+ 4-, 0<|x|<l [•] represents the greatest integer function)
x xz
1. Let/x) = * (a) ke {nn, ne
x=0 (b) ke. {2nn, ne /},p>0
.3’
(c) k g n g I],p g R- {0}
If/(x) is continuous at x = 0, then the value of cr + (f is
(d) k g {n/r, n g I, n 0}, p g R - {0}
(a) 1 (b) 2 (c) 5 (c) 9
7. Statement 1: Minimum number of points of discontinuity
2 of the function fix) = igix)) [2x - 1 ] V x g (-3, -1), where
x is irrational
2. If/W= 1 + x 2’ has exactly two points [.] denotes the greatest integer function and g(x) = ax3 +
x2 + 1 is zero.
A x is rational
Statement 2: fix) can be continuous at a point of
of continuity then the values of b are discontinuity, say x = c, of [2x - 1] if g(c() = 0.
(a) (0, 3] (b) [0, 1]
(a) Statement 1 is True, Statement 2 is True, Statement 2
(c) (0,2] (d) 0
is a correct explaination for Statement 1.
a-x 7tX (b) Statement 1 is True, Statement 2 is True, Statement 2
sin tan — for x > a is NOT a correct explanation for Statement 1.
2 _2a.
(c) Statement 1 is True, Statement 2 is False.
3. Iffa) = 7tX
cos (d) Statement 1 is False, Statement 2 is True.
2a
for x < a 8. Number of points of discontinuity ot fix) = [sin" *x] - [x]
a-x
in its domain is equal to (where [•] denotes the greatest
(where [x] is the greatest integer function of x), and a > 0, integer function)
then (a) 0 (b) 1 (c) 2 (d) 3
(a) fia)<0 xm fjx) + hjx) + 3
(b) /has a removable discontinuity at x = a 9. Ifg(x)= lim whenx# 1 and g(l) =
m—>oo 2xM + 4x + l
(c) /has an irremovable discontinuity at x = a
(d) /(«+)<0
e3 such that/x), g(x) and /i(x) are continuous functions at
x= 1 then the value of 5/1)- 2/i(l) is
7T 7T i z « r i j
4. Let/(x) = [tan x [cot x]], xe —, — I, (where [•] denotes (a) 7 (b) 6 (c) 9 (d) 8
10. The number of points of discontinuity of/x) = [2r]2 - {2x}2
the greatest integer less than or equal to x). Then the number (where [ ] denotes the greatest integer function and { } is
of points, where fix) is discontinuous is fractional part of x) in the interval (-2, 2), is
(a) one (b) zero (a) 1 (b) 6 (c) 2(c) 2 (d) 4
(c) three (d) infinite
-3 + |x|, - 00 < X< 1
5. Let/: [a, b] —> R be any function which is such that/x) 11. Let J(x) = and
is rational for irrational x and/x) is irrational for rational a + |2-x|,, 1 < X < «>
x. Then in [a, 6] 2-|-x|, •-oo<X<2
(a) /is discontinuous everywhere -b + sgn(x), 22<x<
S x < °°
(b) / is continuous only at x = 0 and discontinuous
where sgn(x) denotes signum function ofx. If /i(x) =fix)
everywhere
+ gix) is discontinuous at exactly one point, then which of
(c) / is continuous for all irrational x and discontinuous the following is not possible?
for rational x (a) a = -3,b = 0 (b) a=0,b=l
(c)a = 2,b=l (d)a = -3,b=l
3.4 Calculus

v3
12. The function y(x) =------ sin o + 4 in [-4. 4] does not x — + x[x] if x * 0
15. Lct/(x) = - _xj (where [x] denotes the
take the value
(a) -4 (b) 10 (c) 18 (d) 12 0 ifx = 0
13. Let /x) be continuous functions f: R -> R satisfying greatest integer function). Then the correct statements is/
/()) = 1 and/(2v) -/x) =x. Then the value of/3) is are
(a) 2 (b) 3 (c) 4 (d) 5 (a) Limit exists for x = -l.
(b) /(x) has a removable discontinuity at x = 1.
(c) /(x) has a non removable discontinuity at x = 2.
Multiple Correct Answers Type J (d) /(x) is discontinuous at all positive integers.
16. A function/:/?—»/? is defined as
3- cot for x > 0 ax2+ bx + c + em
14. Given f{x) = < (where /x) = lim where/is continuous on R,
{x2}cos(el/x)
n—><~ \+ce'u
for x < 0
then
{} and [1 denotes the fractional part and the integral part (a) point (a, b, c) lies on line in space
functions respectively). Then which of the following (b) point (a, b) represents the 2-dimensional Cartesian
statements do/does not hold good? plane
(a) /0') = 0 (c) Locus of points (a, c) and (c, b) intersect at one point
(b) /0+) = 3 (d) point (a, b, c) lies on the plane in space
(c) IfflO) = 0. then/x) is continuous at x = 0
(d) Irremovable discontinuity of/at x = 0

I
Answers Key
Single Correct Answer Type Multiple Correct Answers Type
1. (b) 2. (c) 3. (b) 4. (c) 5. (a) 14. (b, d) 15. (a, b, c, d) 16. (a, b, c)
6. (a) 7. (d) 8. (d) 9. (b) 10. (b)
11. (d) 12. (c) 13. (c)
Continuity and Differentiability 3.5

DPP
Differentiability

7. Number of points where function f(x) defined as


Single Correct Answer Type
„3 - cos x — 11 , .|sin
• x|. < —1f=
3x2 - 4x[x + 1 for x < 1 '2 v2 .
V2
1. Let A*) = A [0,27r]->AA-v) = is
ax + b for x > 1 I
2 + cos x + , | sin x| >
IfAx) is the continuous and differentiable for all real values V22 V2
in its domain then non-differentiable is
(a) a = b = 4 (b) a = b = -4 (a) 2 (b) 4 (c) 6 (d) 0
(c) a = 4 and b- -4 (d) a = - 4 and b = 4
[x] Xi I
8. Let f(x) = ■ (where, [•] denotes the greatest
[x] + y[{x}, x < 1 x -1 x e I
2. If^x) = 1 ., then integer function) and
x>l
,w+w2’ sinx + cosx, x<0
j > • Then for f(g(x)) at x = 0
[where [•] and {•} represent the greatest integer and
fractional part functions respectively]
(a) Av) is continuous atx = 1 but not differentiable (a) lim f(g(x)) exists but not continuous
jr—>0
(b) Av) is not continuous at x = 1 (b) continuous but not differentiable at x = 0
(c) Av) is differentiable at x = 1 (c) differentiable at x = 0
(d) lim Av) does not exist (d) lim f(g(x)) does not exist
x—>0

3. Iff is an even function such that lim /W-/(0) has some sin(cos-lx) + cos(sin-lx), x<0
/i->0 h 9. If f(x) then at
finite non-zero value, then sin(cos-1 x) - cos(sin-1 x), x > 0
(a) f is continuous and derivable at x = 0 x=0
(b) f is continuous but not differentiable at x = 0 (a) A-v) is continuous and differentiable
(c) f may be discontinuous at x = 0 (b) Av) is continuous but not differentiable
(d) None of these (c) Av) not continuous but differentiable
4. Let Av) be differentiable for real x such that (d) Av) is neither continuous nor differentiable
f'(x) > 0 on (-oo, -4),
f'(x) < 0 on (-4, 6) and n 3k
10. IfAx) = max{tan x, sin x, cos x} where x e then
f'(x) > 0 on (6, «•).
If g (x) =f (10 - 2x), then the value of g'(2) is the number of points, where Av) is non-differentiable, is
(a) 1 (b) 2 (c) 0 (d) 4 (a) 2 (b) 3 (c) 4 (d) 5
5. Number of points where A-v) = x2 — lx2 — 11 + 211x1- 11 +
21 x I - 7 is non-differentiable is 11. The number of points at which g(x) = is not
(a) 0 (b) 1 . (c) 2 (d) 3
6. If A*) = lx - II • ([x] - [-x]), then (where [•] represents 1
greatest integer function) differentiable, where A-v) = . is
1
(a) A-v) is continuous and differentiable at x = 1 1+-
x
(b) A-v) is discontinuous at x = 1
(a) 1 (b) 2 (C) 3 (d) 4
(c) Ax) is continuous at x = 2
(d) A-v) is continuous but non-differentiable at x = 1
3.6 Calculus

"-1 (a) r<0*) = 0


12. Let f(x) = lim V----------- :------------ .Then
"-♦«^(rr+l){(r + l)x + l} «>) r<o-)= -f-
1-k
(a) is continuous but not differentiable at x = 0
(c) fix) is differentiable at x = 0
(b) fix) is both continuous and differentiable at x = 0
(d) fix) is non-differentiable at x = 0
(c) fix) is neither continuous not differentiable at x = 0
16. Ifflx) = x,/3(x - 2)2/3 for all x, then the domain off is
(d) fix) is a periodic function
(a) x 6 R - { 0 } (b) {xlx>0}
13. Let the given function is differentiable at x = 1.
(c) xe R- {0,2} (D) x e R
/ \n
Jtx | +(px.22+2)
av(x-l)|cot^
4 J ' ,xg(0,1)u(1,2)
lim Comprehension Type
/(x) /?—><»

cot — +1
4 ) For Questions 17 and 18
0, X=1 Consider two functions y =fix) and y = g(x) defined as

Then the value of Io + p\ is ax2 + b\ 0<x<1


(a) 4 (b) 6 (c) 8 (d) 10 fix) = ■ 6x + 26; 1<x<3
(a - l)x + 2c - 3; 3 < x 4
Multiple Correct Answers Type cx + <7; 0<x<2
14. Let/be a function with continuous second derivative and and g(x) =. ax + 3 - c; 2<x<3
fO) = 0,/'(0) = 0. Define a function g by x2 + b + 1; 3<x<4

£(*)
M
X
x*0 17. Let/be differentiable at x = 1 and g(x) be continuous at
x = 3. If the roots of the quadratic equation x2 + (a + b +
0, x=0 c) ax + 49(fc + ka) = 0 are real distinct for all values of a
Then which of the following statements is correct? then possible values of k will be
(a) g has a continuous first derivative (a) k e (-1,0) (b)fee(~,0)
(b) g has a first derivative (c) fee (1,5) (d) fee (-1,1)
(c) g is continuous but g fails to have a derivative
18. lim exists and /is differentiable at x = 1. The
(d) g has a first derivative but the first derivative is not X_>2lg(x)l + 1
continuous
value of limit will be
15. Let/x) be a function defined on (-a, a) with a > 0. Assume
(a) -2 (b) -1 (c) 0 (d) 2
that f(x) is continuous at x = 0 and lim
/(x)-/(fcr) ~a,
where k e (0,1) then x

Answers Key
Single Correct Answer Type Multiple Correct Answers Type
1. (c) 2. (a) 3. (b) 4. (c) 5. (a) 14. (a, b) 15. (b, c) 16. (c)
6. (d) 7. (b) 8- (c) 9. (d) 10. (b)
Comprehension Type
11. (c) 12. (c) 13. (b)
17. (a) 18. (c)
Ln] IPTr IB. l&

Methods of
Differentiation

DPP

Differentiation: Definition, Different Rules of Differentiation

EZ Single Correct Answer Type (a) 1-


1
Jl - x'.2
(b) I-
>]\ + x2
1__

1. The right hand derivative of/(x) = [x] tan jdc at x = 7 is


(where [.] denotes the greatest integer function) __1 1__
(c) 2- (d) 2-
(a) 0
(c) -In
(b) 7tf
(d) none of these
7T-x2 Jl + x2

2. Let Xx - y) = f{x) ■ g(y) - f(y) ■ g(x) and g(x - y) = 9 , ___________ j--------------- - ■

g(x) ■ g(y)-J{x) ■ g(y) for all x, ye R. If right hand derivative 7. If y = — + -xyjx2 +1 + In yjx + Jx2 +1 then the value of
al x = 0 exists fory(x), then derivative of g(x) at x = 0
xy' + log y is
(a) -1 (b) 0
’ (a) y (b) 2y (c) 0 (d) -2y
(c) 1 (d) none of these
8. Let g(x) = » sin x, where/(x) is a twice differentiable
function on (-«>, o°)such that f\-n} = 1. Then value of
3. If xe9' - y = sin2x then — at x = 0 is
dx g"(-n) equals
(a) 0 (b) 1 (a) 1 (b) 2 (c) -2 (d) 0
(c) -1 (d) None of these 9. If/(x) = logr (logr x) then f (x) at x = e is
4. Let f, g and h be differentiable functions. If /(0) = 1; (a) 0 ' (b) 1 (c) e (d) \/e
g(0) = 2; /?(0) = 3 and the derivatives of their pair wise 10. Let g(x) = e^x) and f(x + 1) = x + /(x) Vx G R. If
products at x = 0 are (/g)'(0) = 6; (g h)'(0) = 4 and (/z/)'(0) 1
g'| w + 7 g
= 5 then the value of ((/g/?)'(0))/2 is 2
n g I+, then
(a) 2 (b) 4 (c) 8 (d) 16 T <2 fl
gl ” + -
5. If for a continuous function/,/(0)=/(l) = 0,/'(I) = 2 and 2.
y(x) then y'(0) is equal to
( . J, 1 1 1 1)
(a) 1 (b) 2 (a) 2 ! + - + - + ••• + -
I 2 33 nJ
(c) 0 (d) none of these
f I
T^x" 1-x (b) 2 ( 1+1A + 11 + ...
6. The derivative of cos 2 tan - 2 cos I 3 5 2n-l
1 + xy 2
w. r. t. x is (c) n
(d) 1
4.2 Calculus

^[cos-'CvV? - 7(1--')(!= Multiple Correct Answers Type


11. L.
1 7i+x2+1 dy
(a) 12. y = cos , then — is equal to
2<Jx - x2 2yl\+X2 dx
-1 1
(b) 1 1
2-Jx-x2 (a) ,xeR (b) ,x>0
2(1 + x2) 2(1+x2)
1
(c) +---- ■
2dx - x2 (c) ----- —t-,,x<0
x<0 (d) --------- =- ,x<0
(d)
2(1+ x2) 2(1+ x2)
1__ 13. Suppose that fix) is a differentiable invertible function
(d)
7? f\x) * 0 and h'(x) =ftx). Given that/(I) =/(!)= 1,
/?(1)=0 and g(x) is inverse of/(x). Let G(x)=x2g(x)-xh(g(x))
V x e R. Which of the following is/are correct?
(a) Gz(l) = 2 (b) Gz(l) = 3
(c) G"(l) = 2 (d) G"(l) = 3

Answers Key
Single Correct Answer Type Multiple Correct Answers Type
1. (b) 2. (b) 3. (b) 4. (c) 5. (b) 12. (b.d) 13. (a, d)
6. (a) 7. (b) 8. (c) 9. (d) 10. (c)
11. (b)
Methods of Differentiation 4.3

Differentiation of Function in Parametric Form, Differentiation


of One Function w.r.t. Another Function, Differentiation Using
Logarithm

(a) y (b) xy (c) 0


L.. ' Single Correct Answer Type (d) 1
4 sin 2x + 3 cos 2x
7. The differentia! coefficient of sin
1. If/(I +X2) = x and x2 + t2 = y then at x = 2, the value of
dy . 5 cos x - 4 sin x
— is with respect to cos is
dx V41
24 z x 488
(a) —
5
(b) 151
125
(c) -----
125
(d)
125
(a) -2 (b) -1 (c) 1 (d) 2

( dy V dy 8. xy = (x + y)n and — = — then n =


\+t dy dx x
2. Ifx =y = + —satisfy/(x) — = 1 + —, then
/3
V 2/2 t \dx) dx
dx
(a) 1 (b) 2 (c) 3 (d) 4
J(x) equals
X2 d
9. If x + y = 3e2 then —(x3) = 0 for x =
(a) x (b) dx
1 + x2
(a) e (b) e2 (c) e* (d) 2e2
Z X 1
(c) X + x + - (d) x-- 10. Ifflx) = (x-1) 100 (x - 2)2(99> (x - 3)3 (98)... (x - 100) 10°,
X X
u u , r Z'dOl) .
then the value of - -------- is
3. Let y = x3 - 8x + 7 and x If — = 2 and x = 3 at /(101)
A a dx dx dt dt
r = 0, then — at t = 0 is given by (a) 5050 (b) 2575 (c) 3030 (d) 1250

Multiple Correct Answers Type I


(a) 1 (b) '4
11. A curve in thexy-plane is parametrically given by x = t + r3
2
(c) — (d) none of these and y = t2, where te R is the parameter. For what value(s)
ofris ^ = 1?
4. If x + cos 0 = sec 0, y + cos8# = sec8# then
2 dx 2
fx2 + 4> dy
■2 + 4y dx (b) 2 (c) 3 (d) I
(a)I
(a) 8 (b) 16 (c) 64 (d) 49 jbsdogx) d\>
dv .
5. The derivative of the function represented parametrically 12. Ify = x.(log.r)' , then — is
dx
as x = 2t -111, y = t2 + t2 111 at t = 0 is
(a) -1 (b) 0 (a) (!n.r1°8r-,) + 21nxln(lnx))
(c) 1 (d) does not exist x
M I (b) —(log x)'|i°g(|°g<)(2 l0g(logx) + l)
6. Ify = tan andx= sec
2m2-1’ x
(c) [(In x)2 + 2 In (In x)]
A 1 1 ( 1 . dv
ue 0,-7= U -7=, 1 then the value of 2 -7- + 1 is i^SZ(21og(logx) + l)
V2j
2 lV2 dx (d)
X log X

Answers Key -------------------


Single Correct Answer Type Multiple Correct Answers Type
1. (c) 2.2. (a)
(a) 3. (c) 4. (c) 5. (b) 11. (a, d) 12. (b.d)
6. (c) 7.7. (d)
(d) 8. (b) 9. (b) 10. (a)
4.4 Calculus

DPP

Higher Order Derivatives

Single Correct Answer Type


(b)
1. The function f:R-*R satisfies A-v2) f"M = f'(x) fix2) \ dx- J
for all real x. Given that Al) = 1 and = 8, then the 2

value of/(1) +/'(1) is (C)


(a) 2 (b) 4 (c) 6 (d) 8 I dx2 )
2. The second derivative of a single valued function 2
parametrically represented by x = 0(0 and y =^¥(1) (where (d) fy.
0(0 and W) are different functions and 0'(z) 0) is given dx2
by 5. If Al) = 3,/'(!) = 2,/"(I) = 4, then (f l)"(3) =

'fy ^2xY dy\


(a) —4 = (5)
(a) I <b)-l (c) -2 (d) noneof th
\dt2 dt2 Kdt)
dx2 xA
6. If the third derivative of is
,(x-l)(x-2)
r d2x -Ylk 6
+ (x-l)7, then the value of k is
Vdtkdl 2 (x-2)4
<b) 4=
dx2
er (a) 2 (b) 4
d 2y
7. If (a + bx) ey,x = x then x3 —y equals
dx
(C) 8 (d) 16

<(c)\ -4-=
d~y
dx2
d 2x
dt M (!)
dx f d2y
It dt2 >
3 (a)
dx
2
(b)
dy V

2 2
dy dy
(C)
~r~y (d) x— + y
dx dx
</2*Y dy\ p2?Y
t/2y dt2 )\dt J I dt2 A dt )
(d) 1?-
(fl
3. For the curve sin x + sin y = 1 lying in first quadrant. If
8. If/?=L—\dx) J. , then Z?273 can be put in the fonr

dx2

Jim xa exists and non-zero then 2a = 1 1 1 1


(a) (b)- ,2 \2/3
x->o dx~ / .2 ?'’ + 'd2x x2/3 f +
d y d y a x
(a) 3 (b) 4 (c) 5 (d) 1
k dx2 > <dy\ < dx2 , [dy\
ax + p .. ~dy-----d-r
2y .
is
4. ify= yx + 8 , then 2—
dx dx (c)
2
+-
2___
(d)-
1 1
^2 \2/3 / / 2/3
J2xV/3
(a) 7|4
Vdx2 )
dx2 > \dx2 <dy2 ?
Methods of Differentiation 4.5

9. If x ~ 2 cos t - cos 2t and y = 2 sin t - sin 2t, then the value (a) *4 = 4 (b) *8 = -16
of (cfi-yldx?-) when t = nJ2. (c) *12 = 20 (d) *16 = -24
(a) 1/2 (b) 5/2 (c) -3/2 (d) 2 12. If y = y(x) and it follows the relation + y cos x = 2, then
1 d 2y cfy_ (a) y'(0) = -I (b) y"(0) = 2
10. If y3 - y = 2x, then x2 — —T + x~ =
27 J dx2 dx ' (c) y'(0) = 3/2 (d) y"(0) = -2
13. A twice differentiable function fix) is defined for all real
(a) y (b)f (0 y-
9
(d)
27
numbers and satisfies the following conditions: flQ) = 2;
f(0) = -5 and/"(0) = 3. The function g(x) is defined by
g(x) = e™ +/W V x e /?, where ‘a’ is any constant.
Multiple Correct Answers Type If g'(0) + g"(0) = 0. Then the value/values of a \sJare
(a) I (b) -1 (c) 2 (d) -2
11. If y = e_Jtcos x and yn + kn y = 0 where yn = and k ,
dxn
are constants V ne N, then

Answers Key
Single Correct Answer Type Multiple Correct Answers Type
1. (c) 2.2. (a)
(a) 3. (a) 4. (c) 5. (b) 11. (a,b) 12. (a, b) 13. (a,d)
6. (c) 7.7. (b)
(b) 8. (a) 9. (c) 10. (c)
se
4.6 Calculus

dpp 4.4
Differentiation Using First Definition of Derivatives and
Miscellaneous Problems

[ Single Correct Answer Type


. _ . Ji
(x-o)4 (x-a)2
f\x) = A- (x-b)4 (x-b)2
1
1 . The value of A is
1. Let /(,)=*« when x * 0 and/0) = 0. (x-c)4 (x-c)2 1
X
If g(0) = g'(0) = 0 and g"(0) = 17 then/(0) = (a) 1 (b) 2 (c) 3 (d) none ofthese
(a) 3/4 (b) -1/2 (c) 17/3 (d) 17/2
2. Let f: (-*>«. «>) -» [0. °°) be a continuous function such that /'(x) /(x)
9. Suppose = 0 where /(x) is continuously
/x + y) =/x) +AV) +/x Av)< Vx, y e R. Also/(0) = 1. Then /"(x) /'(x)
(/2)] equals ([•] represents the greatest integer function) differentiable function with/(x) * 0 and satisfies/O) = 1
(a) 5 (b) 6 (c) 7 (d) 8
and/ (0) = 2 then fim —- is
3. Let/: /?->/? be a function satisfying/x + y) =/x) + Ary x—>0 X
+ 3xV for all x. y e R. IfA3) = 4 and/5) = 52 then/(x)
is equal to (a) 1 (b) 2 (c) 1/2 (d) 0
(a) lOx (b) -lOx (c) 20x (d) 128x 10. A non zero polynomial with real coefficients has the
4. A function/:/?—>( 1. <*>) satisfies the equation Axy) =/x) properly lhat/x) = /(x)/'(x). If a is the coefficient of
I Av)-fix)-fiy) + 2. If/is differentiable on R - {0} and highest degree term of/x), then the value of a is
(a) 1/3 (b) 6 (c) 12 (d) 1/18
/2) = 5,/(x) = —- • A then A = 11. If */ is an increasing function from R —> R such that
x
. ^(/-'(x))
f"(x) > 0 and f 1 exists then is
(a) 2/(1) (b) 3/(1) (c) (d)/'(l) dx~
(a) <0 (b) >0
5. I.M f^x + y) - /(*) _ /(t) - a +xy for all real x and y. If (c) =0 (d) cannot be determined
2 2 12. Vertices of a variable acute angled triangle ABC lies on a
Ax) is differentiable and/(0) exists for all real permissible fixed circle. Also, a, b, c and A, B, C are lengths of sides
values of a and is equal to ^5a -1 - a2 • Then and angles of triangle ABC, respectively. If xp x, and x,
(a) Av) >s positive for all real x are distances of orthocentre from A, B and C, respectively.
(b) Ax) is negative for all real x
(c) /x) = 0 has real roots then the maximum value of is
da db de
(d) nothing can be said about the sign of Ax)
6. Let A3) = 4 and /(3) = 5. Then lim [/(x)] (where [.] (b) -3x/3 '3 (d) 3^3
(c)
denotes the greatest integer function) is 13. In a question a student was given to find the derivative ol
(a) 3 (b) 4 the product of two functionsfand g. The student by mistake
(c) 5 (d) non-existent thought (fg)' = f'g' for his question /(x) = x3 and he go
7. Let/x) be a function which is differentiable any number the correct answer. Given that g(4) = 1. Then which of the
of limes and / (2x2 - 1) = 2x3/(x), V x e R. Then following is false?
/20i0)(0) _ (Here f’fix) = /Ith order derivative off at x)
(a) -1 (b) 1 (a) g(5) = |
(c) 0 (d) data is insufficient o

(x-a)4 (x - a)3 1 (b) /(x)<0


(c) /0)<0
8. If/x) = (x-b)4 (x-b)3 1 then
(d) None of these
(x-c)4 (x - c)3 1
Methods of Differentiation 4.7

14. / is a strictly monotonic differentiable function with 15. Suppose/: R —> R* be a differentiable function such that
J—— If g is the inverse off, then g"(x) =' 3/x + y) =fM fly) V x,ye R with/; 1) = 6. Then the value
/zw= , -3 of/2) is
(a) 6 (b) 9 (C) 12 (d) 15
3x2 3g2(x)
(a) (b)
2yl\+X3 2yl\ + g\x)
x2
(c) |s2W (d)
Jl + x3

Answers Key
Single Correct Answer Type 6. (d) 7. (c) 8. (c) 9. (b) 10. (d)
1. (d) 2. (b) 3. (b) 4. (d) 5. (a) 11. (a) 12. (b) 13. (a) 14. (c) 15. (c)

i
CL' BO
1

I Applications of
Derivatives

DPP 5.1
Tangent and Normal

Single Correct Answer Type (a) x = ±y/2(y-2) (b) x = ±V3(y-2)


(c) x = ±>/2(y-3) (d) x = ± V3 (v - 3)
1. The equation of the normal to the curve parametrically
represented by x = t2 + 3/ - 8 and y = 2t2 - 2t - 5 at the 7. If the tangent at (1, 1) on y2 = x(2 - x)2 meets the curve
point P(2, -1) is again at P, then P is
(a) 2x + 3y-I=0 (b) 6x-7y-U=0 (a) (4,4) (b) (2.0)
(c) 7x + 6y - 8 = 0 (d) 3x + y - I = 0 (c) (9/4, 3/8) (d) (3, 31/2)
2. If the curves y = x3 + ax and y = bx2 + c pass through the 8. If a curve with equation of the form y = ox4 + bx? + ex +
point (-1,0) and have a common tangent line at this point d has zero gradient at the point (0, I) and also touches the
then the value of a + b + c is x-axis at the point (-1, 0) then the values of x for which
(a) 0 (b) 1 (c) -3 (d) -1 the curve has a negative gradient are
3. If the function /(x) = x4 + bx2 + 8x + I has a horizontal (a) x>-l (b) x > 1
tangent and a point of inflection for the same value of x (c) x<—1 (d)-I<x<l
then the value of b is equal to 9. From the point (1, 1) tangents are drawn to the curve
(a) -2 (b) -6 (c) 6 (d) 3 represented parametrically as x = 2t -12 and y = t +12. The
4. Let /(x) = x3 + x + 1 and let g(x) be its inverse function distance between the points of contact is
then equation of the tangent to y = g(x) at x = 3 is 2x/43 2\/53
(a) x - 4y + 1 = 0 (b) x + 4y - 1 = 0 (a) — (b) 2 (c) 3 (d) -~
(c) 4x - y + 1 = 0 (d) 4x + y - 1 = 0 10. The value of parameter t so that the line (4 - r)x + ty +
5. A curve is represented parametrically by the equations (a3 - 1) = 0 is normal to the curve xy = 1 may lie in the
x = t + e01 and y = -t + eal, t e R and a > 0. If the curve interval
touches the axis of x at the point A, then the coordinates (a) (1,4) (b) (-«>, 0) u (4.«»)
of the point A are
(c) (-4,4) (d) [3,4]
(a) (1,0) (b) (2e, 0)
11. The portion of the tangent at any point on the curve x = at2,
(c) (e,0) (d) (1/e, 0)
y = at4 between the axes is divided by the abscissa of the
6. The equation of the straight lines which are both tangent
point of contact externally in the ratio
and normal to the curve 27a-2 = 4V3 are
(a) 1/4 (b) 3/4 (c) 3/2 (d) 2/5
5.2 Calculus

(a) equation of tangent at t = 0 is x - 2y = 0


Multiple Correct Answers Type
(b) equation of normal atr = 0is2x + y = 0
12. Equation of a line which is tangent to both the curves y = (c) tangent does not exist at t = 0
x2 + 1 and y - -x2 is (d) normal does not exist at t = 0
(a) y = V2x--i (b) y = V2x + | 14. For the curve x = 2asin t + a sin t cos2V, y = -a cos31
(a) normal is inclined at an angle — +1 with x-axis.
(c) y = -\[2x + (d) y = -J2x - i (b) normal is inclined at an angle t with x-axis.
(c) portion of normal contained between the co-ordinaU
13. For the functions defined parametrically by the equations axes is equal to 2a.
(d) portion of normal contained between the co-ordinaU
2r + /22sin- ;r*0 and
/(') = * = t axes is equal to 4a.
0 ;/ = 0 15. The curve y = ax3 + bx1 + ex is inclined at 45° to x-axis al
(0,0) but it touches x-axis at (1,0), then
[1 • ,2
-sm/ ;/*0 (a) r(l) = 0 (b) /'(I) = 2
g(t)=y= w (c)/"'(2) = 12 (d)/(2) = 2
o ;/ = 0

Answers Key
Single Correct Answer Type Multiple Correct Answers Type
1. (c) 2.2. (d)
(d) 3. (b) 4. (a) 5. (b) 12. (b, c) 13. (a, b) 14. (a, c) 15. (a, b, d)
6. (a) 7.7. (c)
(c) 8. (c) 9. (d) 10. (b)
11. (b)
Applications of Derivatives 5.3

DPP

Applications of Tangent and Normal

£ Single Correct Answer Type j 7. Distance of point P on the curve y = x372 which is nearest
to the point M (4, 0) from origin is
1. The length of the sub-tangent to the hyperbola x2 - 4J2 = 4
corresponding to the normal having slope unity is -4=-, then (a)
112 (Too
(b)
27 V 27
the value of k is
(a) 1 (b) 2 • (c) 3 (d) 4 fioT
(c) (d)
2. Cosine of the acute angle between the curves y = 3X~ Uog^ V 9
and y = xx - 1 is
8. If the equation of the normal to the curve y =J(x) at x = 0 is
(a) 0 (b) 1 (d) 1 3x-y + 3 = 0 then the value of
(c) T x2
hm----- =-=------------— is
3. Acute angle between two curves x2 + y2 = a141 andx2 x-»0{/(?) - 5/(4?) + 4/(7?)}
- y2 = a2 is
(a) -3 (b) 1/3 (c) 3 (d) -1/3
(a) £ (b)
(b) t (c)
(c) T (d) none of these
6 3 4
Multiple Correct Answers Type
4. The minimum distance between a point on the curve y =
ex and a point on the curve y = log^x is 9. If Lp Ln, and LSN denote the lengths of tangent, normal,
(a) (b) 42 (c) 3 (d) 2V2 sub-tangent and sub-normal, respectively, of a curve y =
/(x) at a point P(2009, 2010) on it, then
5. Tangents are drawn from origin to the curve y = sin x +
cos x. Then their points of contact lie on the curve ^ST _ 2010 Ksn
(a) (b) = constant
, x 1 2 , 2 1 1 2010 Lsn Vsr
(a) -y + -2=1 (b) — - — = 1
x y x y
2000 ' Lj- + Ln _ ^ST
,2 1 , 2 1 , (c) 1 ~ (d)
(c) — + — = 1 (d) —-- = 1 2010 \Lj-~ LN,
x y y x
10. Which of the following pair(s) of family is/are orthogonal?
6. If 3x + 2y = 1 is a tangent to y = /(x) at x = 1/2,
(a) 16x2 + y2 = candy16 = kx
x(x-l)
then lim (b) y = x + ce~x and x + 2 = y + ke~y
x->0 e2x>|
(c) y = ex2 and x2 + ly3- — k
2 J J 2 J (d) x2-y2 = candxy = k
(a) 1/3 (b) 1/2 (c) 1/6 (d) 1/7
where c and k are arbitrary constant.

Answers Key
Single Correct Answer Type Multiple Correct Answers Type
1. (c) 2.2. (b)
(b) 3. (c) 4. (b) 5. (d) 9. (a, b) 10. (a,b,c,d)
6. (a) 7.7. (a)
(a) 8. (d)
5.4 Calculus

DPP

Measure of Rate

i 160
Single Correct Answer Type J (a) 30 cm/sec (b) cm/sec

X (c) 10 cm/sec (d) 160 cm/sec


1. The rate of change of ^x2 +16 with respect to ----- at
x-1 8. Let the equation of a curve be x = a(0 + sin 9).
x = 3 is
y = a( 1 - cos 9). If 0 changes at a constant rate k then the
11 ( x 12 rate of change of the slope of the tangent to the curve at
(a) 1 (b) ? (0 — (d) -3
0=^/3 is
2. The eccentricity of the ellipse 3X2 + 4}12 = 12 is changed at (a) 2kN3 (b) ^3
the rate 0.1/sec. The time at which it will touch the auxiliary (c) k (d) none of these
circle is
(a) 2 seconds (b) 3 seconds Comprehension Type
(c) 5 seconds (d) 6 seconds
3. A particle moves along the parabola y = x2 in the first
For Questions 9 and 10
quadrant in such a way that its x-coordinate (measured
A lamp post of length 10 meter placed at the end A of a ladder
in metres) increases at a rate of 10 m/sec. If the angle of
AB of length 13 meters, which is leaning against a vertical wall
inclination 0 of the line joining the particle to the origin
as shown in figure and its base slides away from the wall. At the
changes, when x = 3 m, at the rate of k rad/sec., then the
-instant base B is 12 m from the vertical wall, the base B is moving
value of k is
at the rate of 5 m/sec. A man (M) of height 1.5 meter standing at
(a) 1 (b) 2 (c) 1/2 (d) 1/3
a distance of 15 m from the vertical wall.
4. If the rate of change in the volume of sphere is equal to
1
the rate of change in its radius when the radius is
ky/ft
The value of k is
>4
(a) 1 (b) 2 (c) 0.5 (d) none of these
13 m
5. Water is dropped at the rate of 2 m3/s into a cone of semi­
vertical angle of 45°. The rate at which periphery of water
surface changes when height of water in the cone is 2 m,
is IV B+M
(a) 0.5 m/s (b) 2 m/s
(c) 3 m/s (d) 1 m/s 9. Rate at which 0 decreases, when the base B is 12 m from
6. Suppose that water is emptied from a spherical tank of the vertical wall, is
radius 10 cm. If the depth of the water in the tank is 4 cm (a) 1 rad/sec (b) 2 rad/sec
and is decreasing at the rate of 2 cm/sec, then the radius (c) 5 rad/sec (d) 1/2 rad/sec
of the top surface of water is decreasing at the rate of 10. The rate at which the length of shadow of man increases,
(a) 1 (b) 2/3 (c) 3/2 (d) 2 when the base B is 12 m from vertical wall, is
(a) 15 m/sec (b) 40/27 m/sec
7. The altitude of a cone is 20 cm and its semi-vertical angle (c) 15/2 m/sec (d) 5 m/sec
is 30°. If the semi-vertical angle is increasing at the rate
of 2° per second, then the radius of the base is increasing
at the rate of
- Answers Key
Single Correct Answer Type Comprehension Type
1. (c) 2. (c) 3. (a) 4. (b) 5. (d) 9. (a) 10. (b)
6. (c) 7. (b) 8. (d)
Applications of Derivatives 5.5

DPP

Mean Value Theorem

E Subjective Type
"T
(c) more than
n3
(d) more than —
4/T
1. Prove that for A > 1, the equation x log x + x = A has at
7. Given /'(!)= 1 and /(2x) = fix), V x > 0. If fix) is
least one solution in [1, A].
differentiable then there exists a number c e (2, 4) such
2. If fix) and gix) are continuous and differentiable
that f'ic) equals
functions, then prove that there exists c e [a, b] such that
(a) 1/4 (b) -1/2 (c) -1/4 (d) -1/8
f'ic) , g'jc)
fia) -/(c) gib)-gic)
Multiple Correct Answers Type

Single Correct Answer Type 1 1 1

3. Consider fix) = 11 - xl, 1 < x < 2 and gix) = fix) + 8. Let/x) = 3-x 5-3x2 3x3-l . Then which of the
n 2x2-I 3x5-1 7x8-1
b sin — x, 1 <x < 2 then which of the following is correct?
following is/are correct?
(a) Rolle’s theorem is applicable to both / and g with (a) fx) = 0 has at least two real roots
b=3
-. (b) fix) = 0 has at least one real root.
2 (c) fix) is many-one function
(b) LMVT is not applicable to/and Rolle’s theorem is (d) none of these
applicable to g with b = . 9. Which of the following is correct?
tan-1x- tan-Iy
(c) LMVT is applicable to / and Rolle’s theorem is (a) < 1 Vxje fl,(x*y)
x-y
applicable to g with b = 1.
(d) Rolle’s theorem is not applicable to both/and g for sin-1 x-sin y
any real b. (b) > 1 Vx,ye [-1, l],x#y
x-y
1 1 jv x « 0 ., • 1 r • ’ * '1

4. If c = — and fix) = 2x - x2, then interval of x in which cos x - cos y


2 J' "' ....... ............... (C)
LMVT is applicable, is y-x
(a) (1,2) . (b) (-1,1) cot- x - cot- y
(c) (0, 1) (d) (2,1) (d) y-x
5. If a twice differentiable function fix) on (a, b) and
continuous on [a, b] is such that/"(x) < 0 for all x g (a, b)

then for any c e ia, b), —y— >


Comprehension Type ]
_____ J
fib) ~ fic)
b-c c-a For Questions 10-12
(a) ------ (b) Let/x) be a function such that its derivative/'(x) is continuous
c-a b-c
1 in [a, b] and differentiable in (a, b). Consider a function 0(x)
(c) (b-c)(c-a) (d) = fib) - fx) - (b - x) f(x) - ib - x)2A. If Rolle’s theorem is
ib - c)(c - a)
applicable to 0(x) on [a, b], answer following questions.
6. Let a,ne N such that a > n3. Then l]a + \ - l[a is always 10. If there exists some number cia < c < b) such that
<fic) = 0 and/b) =fa) + ib - a) fia) + A(b - a)2 f'ic),
(a) less than (b) less than then A is
3n£
5.6 Calculus

(a) I (b) 0 (c) | (d) 12. Let fix) = sin x, a = a and b - a + h. If there exists
real number t such that 0 < t < 1, 0' (a + th) = 0 an
11. Let./(x) = x3 - 3x + 3, a = 1 and b = 1 + h. If there exists
si" (« + *)-sin «-Acosa = A sin (o + (fc), then A=
Z(1 + /I)-Z(1) h2
c e (1, 1 + /») such that 0'(c) = 0 and
h2
Ar, then A =
(a) 1/2 (b) 2
(a) |
(b) 4 <c) Z
(d)l
(c) 3 (d) does not exist

Answers Key
Subjective Type Multiple Correct Answers Type
2. 1 8. (a,b, c) 9. (a, b)
Single Correct Answer Type Comprehension Type
3. (c) 4. (c) 5. (b) 6. (a) 7. (d) 10. (c) 11. (c) 12. (b)
D
11: uL
si *

1
Monotonocity and
Maxima-Minima
SI
of Functions
DPP 6.1
Monotonocity and Its Applications (Part I)

MOIRjy ---- --------- ------- ----------1—~~


6. The minimum value of k for which/» = 2e*- ke~x + (2k
Single Correct Answer Type J
+ 1) x - 3 is monotonically increasing for all real x is
1. If x g (0, n/2), then the function/(x) = x sin x + cos x + (a) 2 (b) 1 (c) 0 (d) -1
cos2 x is 7. If the function fix) = 2 cot x + (2a + 1 )logc|cosec x| +
(a) Increasing (2 - a)x is strictly decreasing in ^0, y), then range of‘a’ is
(b) Decreasing
(c) Neither increasing nor decreasing (a) [0,00) (b) (-00. 0]
(d) None of these (c) (—,~) (d) none of these
2. The function /: [a, 00) —» R where R denotes the range

F
- ---------~ " ■■■ MV I JI.>

corresponding to the given domain, with rule/(x) = 2X3 -


Multiple Correct Answers Type
3X2 + 6, will have an inverse provided
(a) a < 1 (b) a > 0 (c) a < 0 (d) a > 1 8. Which of the following statement(s) is/are true?
3. Lety(x) = 1 - x - x3. In which of the following intervals, (a) Differentiable function satisfying 7(-l)= 7U) and
inequality 1 -fix) -f(x) >7(1 - 5x) is not satisfied? f(x) > 0 for all x must be a constant function on the
(a) (-2,0) (b) (2,oo) interval [-1, 1].
(c) (0,2) (d) none of these (b) There exists a function with domain R satisfying/(x)
4. If g(x) = 27(2x3 - 3X2) +7(6x2 - 4X3 - 3), V x g R and < 0 for all x ,/'(x)>0 for all x and f" (x) > 0 for all x.
j"(x) > 0, V x g R, then g(x) is increasing for x belonging to (c) If/"(c) = 0 then (c,f(c)) is an inflection point.
-~,-|lu(0,l) (d) Suppose fix) is a function whose derivative is the
(a) (b) -|,0ju(l,~) function//(x) = Zr2 + 2x- 12. Then/(x) is decreasing
for -3 < x < 2 and concave up for x > -^.
(c) (0, °°) (d) (-00, 1)
5. Ifflx) = sin 2x- 8(a + 1) sin x + (4a2 + 8a - 14)x increases
for all x g R and has no critical points for all x e R, then 9. Let /: R -+ R,fix) =x + log/1 + x2). Then
which of the following values of a are not possible? (a) /is injective
(a) (-^o,-V5-2) (b) (1,°°) (b) / is suijective
(c) there is a point on the graph of y =flx) where tangent
(c) (V5, °°) (d) none of these is not parallel to any of the chords
(d) inverse of fix) exists.
6.2 Calculus

10. Let/(x)=x— then which one of the following statements -e~x + k , x<0
x 12. If /(x) - , 0<x<l is one-one and
is true? ex + l
(a) fix) is one-one function. ex2 + A , x>l
(b) fix) is increasing function.
(c) fix) = k has two distinct real roots for any real k. monotonically increasing V x e R, then
(d) x = 0 is point of inflection. (a) maximum value of k is 1
11. Letyix) be an even function in R. Iffix) is monotonically (b) maximum value of k is 3
increasing in [2. 6], then (c) minimum value of A is 0
(a) fi3)<fi-5) (b) fifi)<fi-3) (d) minimum value of A is 1
(c)fi2)>fi-3) (d) fi-3)<fi5)

Answers Key

Single Correct Answer Type Multiple Correct Answers Type


1. (b) 2. (d) 3. (c) 4. (b) 5. (b) 8. (a, d) 9. (a, b, c, d)
6. (c) 7. (a) 10. (b, c, d) 11. (a, d) 12. (b.d)
Monotonocity and Maxima-Minima of Functions 6.3

DPP

Monotonocity and Its Applications (Part II)

Single Correct Answer Type Comprehension Type

tan x2 For Questions 7-9


1. Ifxj,x2e 0, — |, then is (where Xj < x2)
2J tan X]
Let/: R -> R, y =A*) .7(0) = 0,f(x) > 0 and/"(x) > 0 . Three

(a) < — (b) points A(a,/(a)), B(p,AP)), C(y, AT)) on y = /x) such that
x2 x2
0 < a<P<y
(c) <X'X2 (d) 7. Which of the following is false?
*1
(a) oAP)>P(A(x)) (b) oAP)<PAa)
2. If fix) is a differentiable real valued function satisfying
/"(x) - 3/'(x) > 3 V x > 0 and/'(0) = -1. then/(x) + * V
(c) yAP)<PW» (d) TAoPXaAT)
x> 0 is 8. Which of following is true?
(a) decreasing function of x
(b) increasing function of x /(«) + /(/?) a+P
(a)
(c) constant function 2 2
(d) none of these
3. The roots of (x - 41 )49 + (x - 49)41 + (x - 2OO9)2009 = 0 are /(«) + /(/?) a+P
(b)
(a) all necessarily real 2 2
(b) non-real except one positive real root
(c) non-real except three positive real roots f(a) + f(P)_ a+P
(c)
(d) non-real except for three real roots of which exactly 2 2
one is positive
4. Let h be a twice continuously differentiable positive 2/(a)+ /(/?) 2a+ p
(d)
function on an open interval H. Let g(x) = loge(/i(x)) for 3 3
eachxe H
9. Which of the following is true?
Suppose (7i'(x))2 > /i"(x)/i(x) for each x g H.
Then, we can conclude that (a) yAY+P~a)>(y+P~a)AT)
(a) g is increasing on H (b) g is decreasing on H
(b) yAy+P-<*)<(Y+ P-<*)AT>
(c) g is concave up on H (d) g is concave down on H
(c) aflY+P- «)<(/+ P-a)Aa)
5. If sin x + x > K'lx2, Vx e 0, — , then the greatest value of
’ 2 (d) none of these
k is
-2(2 + 7t) For Questions 10-12
(a)
n2 Let/be a twice differentiable function such that/"(x) > 0 V x e
2(2 + 7i)
(b)
IV R. Let /i(x) is defined by /i(x) =/sin2x) +/(cos2x) where Id < y.
(c) can’t be determined finitely
(d) zero 10. The number of critical points of h(x) are
6. If4x+8cosx + tanx-2secx-41og {cosx(l +sinx)}
> 6 for all x g [0, A) then the largest value of A is (a) 1 (b) 2
(a) n/3 (b) te/6 (c) tt/4 (d) 3tz/4 (c) 3 (d) more than 3
6.4 Calculus

11. /'(sin2x)</'(cos2x) for a g 12. h(x) is increasing for x e


it it it it it it it 7t It It
it It (b) u
(a) (b) (a) 4’7
7’4 ”7’ 4 4’7 4’ 4 7’ ~4
it n 1 f 7t
it 7t
it it 7T] A 7t
11
7t
* A I n 71 ££ (c) —,0 u (d) — U 0, -
(c) —,0 kj —
4 ) 14 2
(d)
2’2 4 J l44’2 7’ 4; I ’44

Answers Key
Single Correct Answer Type Comprehension Type
1. (d) 2. (b) 3. (b) 4. (d) 5. (b) 7. (b) 8. (b) 9. (a) 10. (c) 11. (a)
6. (b) 12. (b)
Monotonocity and Maxima-Minima of Functions 6.5

DPP

Extrema and Its Applications (Part I)

(c) Two minima and two maxima


Single Correct Answer Type
(d) One minima and one maxima
1. The greatest possible value of the expression tan x + cot x 8. Ify(x) = lx-II + lx + 41 + lx-91 +... + lx-25001V x e R,
+ cos x on the interval [zz/6, itiA\ is then all the values of x where fix) has minimum values lie
in
(a) —V2 (b) —V2 (c) v3 (d) (a) (600,700) (b) (576,678)
5 6 5 6 (c) (625, 678) (d) none of these
(x + 1)3 -2<x<-l 9. Slope of tangent to the curve
2. Let/(X) = . x':2/3-l -1<x<1 . The total number of k x k X |
y = 2e* sin cos — , where 0 < x < 2k is
-(x-1)2 l<x<2 4 2 ~4 2J
minimum at x =
maxima and minima of/(x) is (a) 0 (b) k
(a) 4 (b) 3 (c) 2 (d) 1 (c) 2k (d) none of these
3. Consider the graph of the function fix) = x+ -J\x\ 10. The value of a for which all extremum of function fix) =
Statement 1: The graph of y =fix) has only one critical x3 - 3ax2 + 3(a2 - l)x + 1, lie in the interval (2,4) is
point. (a) (3,4) (b) (-1,3)
Statement 2: f'(x) vanishes only at one point. (c) (-3,-1) (d) none of these
(a) Statement 1 is true, Statement 2 is true, Statement 2
Multiple Correct Answers Type J
is a correct explanation for Statement 1.
(b) Statement 1 is true, Statement 2 is true, Statement 2
is not a correct explanation for Statement 1. 11. If the function fix) = ax has a local maximum at the
(c) Statement 1 is true, Statement 2 is false. point (2, 10), then
(d) Statement 1 is false, Statement 2 is true. (a) a = 5e (b) a = 5.
(c) *=1 (d) b = l/2
(
tan x + —
x
4. The minimum value of
1 6 f n k1 .
,X€ 0, — , IS 12. Let f(x) =
ex
and g(x) =f(x), then
tan x k 3J 1 + x2
(C)l (a) g(x) has two local maxima and two local minima points
(a) 1 (b) 0 (d) 3 (b) g(x) has exactly one local maxima and one local
minima point
x2 + 2 (c) x = 1 is a point of local maxima for g(x)
5. Let/(x) = , 1 < x < 3, where [.] is the greatest integer
TT (d) There is a point of local maxima for g(x) in the interval
function. Then the least value of/(x) is (-1,0)
(a) 2 (b) 3 (c) 3/2 (d) 1 13. If/(x) = (x-a)r010 _ £)2009 a > then
(a) /(x) has relative maxima at x = b
3-x2, x<2 (b) 7(x) has relative minima at x = b
6. Ifflx) = and if/(x) has a local
7a +14 -|x- 48|, x>2 (c) fix) has relative maxima at x = a
(d) fix) has neither maxima, nor minima at x = a
maxima at x = 2, then greatest value of a is
14. If lim/(x) = lim [/(x)] ([.] denotes the greatest integer
1 (a) 2013 (b) 2012 (c) 2011 (d) 2010 x—x—>a
7. The function fix) = x5 - 5x4 + Sx3 - I has function) and fix) is non-constant continuous function,
(a) One minima and two maxima then
(b) Two minima and one maxima (a) lim fix) is an integer
x—*a
(b) lim fx) is non-integer
x—
6.6 Calculus

(c) /x) has local maximum at x = a (a) /x) increases in the on x = (-1,
(d) Xx) has local minimum at x = a
15. Consider the function /(x) = log/^l - x.2 - x). Then 1_
(b) /has local maximum at x = -
which of the following is/are true? /2
(c) Least value of/does not exist
(d) Least value of/exists

Answers Key

Single Correct Answer Type Multiple Correct Answers Type


1. (d) 2. (b) 3. (d) 4. (d) 5. (b) 11. (a, d) 12. (b.d) 13. (b.d) 14. (a, d)
6. (c) 7. (d) 8. (c) 9. (b) 10. (b) 15. (a. b, c)
Monotonocity and Maxima-Minima of Functions 6.7

DPP 6.4
Extrema and Its Applications (Part II)

7. If the equation 2X3 - 6x + 2 sin a + 3 = 0, a e (0, k) has only


Single Correct Answer Type
one real root, then the largest interval in which a lies is
K K
x3(l-x), x<0
then which of the (a) 0,5 (b)
1. If/(x) =
xlogex + 3x, x>0’
6 7’T
following is not true? n 5k 5^ )
(c) (d) —,
(a) /x) has point of maxima at x = 0 7’T 6 )
(b) /x) has point minima at x = e'4
8. Let /be a continuous and differentiable function in
(c) /(x) has range R
(d) None of these (x,,x2). lfXx).r«ix 7l-(/(x))4 and lim J/(x))3 = 1
2. The coordinates of the point on the curve x3 = y(x - a)2
where the ordinate is minimum is and lim (/(x))2 = —, then minimum value of (x2 - x£) is
- — 27 a 2
(a) 3a, (b) (2a, 8a)
(a) - (b)
(c) (a, 0) (d) None of these
3. The fraction exceeding its pth power by the greatest number
possible, where p > 2, is (0 - (d) None of these
( 1 i V(P-1)
(a) - (b) 9. If ab = 2a + 36, a > 0, 6 > 0, then the minimum value of
\PJ P, ab is
(c) pyP-i (d) none of these (a) 12 (b) 24
x, 0<x<l (d) None of these
(c)7
4. If/(x) = 2-e- , I < x < 2 and g '(x) = /(x),
10. Let a, 6, c, d. e,f, g, h be distinct elements in the set {-7,
x - e, 2<x<3 -5, -3, -2, 2,4, 6, 13}. The minimum value of (a + 6 + c
xg [1,3], then + d)2 + (e +/+ g + 6)2 is
(a) g(x) has no local maxima (a) 30 (b) 32 (c) 34 (d) 40
(b) g(x) has no local minima 11. The perimeter of a sector is p. The area of the sector is
(c) g(x) has local maxima at x = 1 + ln2 and local minima maximum when its radius is
atx = e 1
(d) g(x) has local minima atx = 1 + ln2 and local maxima (b) -7= (C>
atx = e
7p 2
5. If g(x) = max(y2 - xy) (0 < y < 1), then the minimum value 12. Minimum integral value of k for which the equation
of g(x) (for real x) is ex = kx2 has exactly three real distinct solutions,
(a) | (b) 3-73 (c) 3 + 78 (d) | (a) I (b) 2 (c) 3 (d) 4
(a) - 13. Let/x) = x3 - 3x + 1. Then number of different real
4 2
solutions of//x)) = 0
6. If a, b g R distinct numbers satisfying la - 11 +16 - 11 = lai
(a) 2 (b) 4 (c) 5 (d) 7
+161 = la + 11 + 16+ II, then the minimum value of la-61 is
(a) 3 (b) 0 (c) 1 (d) 2

Answers Key
Single Correct Answer Type 6. (d) 7. (c) 8. (c) 9. (b) 10. (b)
1. (d) 2. (a) 3. (a) 4. (c) 5. (b) 11. (d) 12. (b) 13. (d)
VS LL<
3

Indefinite
Integration

DPP
Integration of Elementary Functions

Single Correct Answer Type (c) ^(lnx)2-x + C e


(d) £-+c
2x
cos 5x + cos 4x
1. •dx is equal to cos"-1 x
1 - 2 cos 3x 4. J sinn+l x dx, n * 0 is
sin 2x
(a) -I + cos x + C
cot”x —cot"-lx
2 (a) (b)
n 71-1
cos 2x
(b) - + sin x + C
2 -cot”x cot"-1 X
(c) (d)
cos 2x n n-\
(C) - + cos x + C
2 x27(x-l)'
5. If Jx26 ^-l)l7-(5x-3)dr = — + C where
sin 2x k
(d) -I + sin x + C
2 C is a constant of integration, then the value of k is equal
sec x • cosec x to
2. dx is equal to (a) 3 (b) 6 (c) 9 (d) 12
2 cot x - sec x cosec x
x + (cos 3x)r I------- - . ,
6. If J — dx = Ay] 1 -9x‘ + 5(cos“ 3x)3 + C,
(a) — In I sec 2x + tan 2x I + C 71-9x2
then A - B is
(b) In I sec x + cosec x I + C
(c) In I sec x + tan x I + C

(d) — In I sec x + cosec x I + C


2
<a) i (b) -1
<c) I (d) 0

7. If flan9 x dx =f(x) + log Icos xl, where/(x) is a polynomial


3. p-lnKU = of degree n in tan x, then the value of n is
Jx le'J (a) 6 (b) 7 (c) 8 (d) noneofthese

(a) —ex-lnx + C (b) ylnx-ex+C


2
7.2 Calculus

2
8. J cose*x -+ sin x +1 - x
sin x + x
dx - log,(/(x)) + g(x) + C where 3x3
(c) ----- + 6 tan x* +C
10
C is the constant of integration and/(x) is positive. Then k >
fix) + g(x) has the value equal to 2
3x3 (
(a) e* + sin x + 2x (b) e* + sin x
(d) —^— + 6 tan X* +C
(c) e'-sinx (d) er + sinx + x
k >
2
x + x3+x6 . 10. j ex(x-2) dx
9- J 7 ] equals x(x2 + ex)
V x > 0 is equal to

x 1 + x3 /
f x\ 1
7 (a) In 1H—y + c (b) In -t + ^t +c
2
7 k x 7 < 2 XJ- 7
3x3 x~6
(a) —— + 6 tan +C f
4 f p x> exk
k > (c) In 2 + -y +c (d) In x + ^r +c
2 x2)
f H k x 7 k
3x3
(b) -^— + 6 tan x‘ + C
k 7

Answers Key
Single Correct Answer Type 6. (d) 7. (c) 8. (b) 9. (b) 10. (a)
1. (d) 2. (a) 3. (c) 4. (c) 5. (c)
Indefinite Integration 7.3

DPP
Integration by Elementary Substitution

Single Correct Answer Type 1 V2sin-x _ 1 qOg|sec2x_tan2x|)+C


(b) —log
2V2
1. Let x2 * nn-1, n e N, then
1
2 sin(x2 +1) - sin 2(x2 +1) , . . (c) -j=log 1-^2 sin 2x ”i<10g'SeC2X~2XD+C
p ----- —- dx is equal to
2 sin(x2 +1) + sin 2(x2 +1)
2
(d) none of these
1
(a) In — sec(x2 +1) + C
2 5. Jx2ln(x2+1)dr is equal to
?ln(x2+l)
x2 + l (x2 +1)2'ln(x2+l)
(b) In sec +C (a) ------2------ + C (b) ------ +c
2 2(x2 + l) In 2 + 1
1
•(c) — In lsec(x2+1)1 + C (X2 + I)1"2" , c (£1D^ + C
2 (c) (d)
2(ln 2 + 1) 2(ln2 + l)
1 2
(d) —In +C
2 sec(x2 +1) sin x
6. If dx = Aflx) + log Isin x - cos xl] + c,
( n
cosec x sin
2. The value of
J—cos
2 x
1 + log tan -
dx is
then
I 4
2

(a) -tan 1 +log tan— +c


x c - ;2 - •
(a) A - —/= ,fix) = sin x
2
of X i
(b) a =-Ji.Ax) = cosx
(b) see" 1 + log tan— + c
2 (c) A= V2,/(x) = x
x .... 1 „ .
(c) tan 1 + log tan — + c (d) A= -=f(x)=x
2 y/2 ' ' '

21 X ] /
(d) sin 1 +log tan— +c cos X ' X
2 7. •sinx dx equals
cosx /
dx
3.
x^x6-16

x3 1 X
x3 (d) C - 2y]x cos x
(a) sec— + c (b) —sec— + c
4 12 4
c\ -sec
/tc; 1 —* +c (d) none of these
8. J ( 2 +(2x4 ++1)l) 3/2 dx
x x
3 4
3
<a) -T-2x------ x
(b) +C
4. f dx _
(x“ + 4x +1) “
+C
(x2 + 4x + 1)i/2
•’ cos(2x)cos(4x)
x2 1
<a) ^log ------/=--------- —(log I sec 2x - tan 2x |) + C (c) +C (d) +C
(x2 + 4x +1)1/2 (x2 + 4x + l)l/2
1-V2sin2x 2
7.4 Calculus

r (2x + 3)cZv_______ 1 Z 73+i -i"


9. If 1.
/ x -log
J x(x +1 )(x + 2)(x + 3) +1- =C- f (x) where/(a) is (c) +C
\ x3 + 1 + 1
of the form of ar + bx + c, then the value of/U) is
(a) 4 (b) 5 (c) 6 (d) none 1 77+7 + 1
(d) -log +c
10. The integral J -Jcot x Jcos x dx equals 3 'x6 + l -ly
fsm x dx 1_________ 1
yjlan x e fsinx + c 12. | =2 + c where
(a) +c (b) 2e' (l + ^)20,° _a(l + Vx)a /J(l + Vx/_
y/COS x

^/cot x e fsmx a, /3 > 0 then a-/Jis


Ismx + c
(C) (d) =—+c (a) 1 (b) 2 (c) -1 (d) -2
2^/cos x
dx sin| — -x \dx
11. equals 13. J = A tan (/(a)) + B, where A, B are
xjx6+l 2 + sin 2x
(a) sec'1 x3 + C constants. Then the range of AJ(x) is
(a) [-1,1] (b) [-72,72]
(b) ilog +C (c) [0,1] (d) [-1,0]
6 k 7x6+1+iy

Answers Key

Single Correct Answer Type 6. (d) 7. (c) 8. (b) 9. (b) 10. (b)
1. (b) 2. (c) 3. (b) 4. (b) 5. (c) 11. (b) 12. (a) 13. (a)
Indefinite Integration 7.5

dpp 7.3
Integration by Typical Substitution

Single Correct Answer Type


J 5.
71-x~.2 -x
dx is
- X2 (1 + Xyj\-X2 )
3/2
1. If
(a) 2 tan -l(x+7l-x2) + c
2
(b) tan-,(x + 71-
(c) 2tan-l(x- ^1 -x2 ) + c
then the value of 3AB is
(a) -1 (b) -2 (d) 2cot-l(x + 71--v:) + c
(C) 1 (d) 2
x2 + l 3x2 + 2x
2. dx is equal to 6. .6
x6 + 2x5 + x4 + 2x3 + 2x.2: + 5
xJx,22 + 2x -1J1 -x,22-x
( X3 + X2 + 1
/(a)x —
I tan
(a) 2 sin x-—+ 2 +c 4 I 2 J
X
(b) —tan
fx3 + x2 + f
+c
2 2 7
(b) 2 cos x-—+2 +c
X
x3 + x2 + l
(c) sin +c
2
(c) sin x-—+2+c
X .3 ,2
(d) — tan +C
2 2
(d) cos 'x - — + 2 + c 1
x 7. I x2
dx is equal to

dx
3. J-2716-
X x: .2
has the value equal to
(a) 2
/
x _1
+c
■Jl 7^,
(a) C-—tan sec (b) —tan-1 sec +C /
4 •4 X i
(b) 2
yjl-X 1-X,
7
.2 716-x' .2
V16-x: / \
(c) C- (d)
16x
-+c
(c) 2
X J +c
I6x

/
it
3 tan x — (d) 2
i x
+c
4
4. If J——f
cos2 Xyj tan3J x +. -2
tan’ x + tan x
5x8 + 7x.6i
8. If/U) = J (x2 + 1 + 2x7)2 dx, and/(0) = 0, then the value
= K tan (7tanx + 1 + cotx) + C, then the value of K is
(a) 2 (b) 3 (c) 6 (d) 8 of 1) is
(a) -1/2 (b) 1/4 (c) 1/2 (d) -1/4
7.6 Calculus

dx X2(l- log x)
9. If/= Jf—
x4
, then Z equals 11. J (log x)4 - x4 dx equals

(a) -Inf —|--ln(ln2x-x2) + C •


1 1 1 ^2 + x2
(a) — —y]a~,2 + x,2 +c 2 I In x J 4
a4 x 3x2
(b) lln r In x - x 1
—tan —I+C
1 4 k In x + x, 2 . x 7
(a2 + x2)3/2 + c
a 1 rln x + x^ 1 In x
(c) -In + -tan +C
Z V 1 1 Ll , v2 1 ,2, 2x3/2
4 k In x - x J 2 x
(C) — - a + x---- ) +c
a x 3x 1( In x-x In x
(d) - In + tan +C
In x + x x 7
o2 + x2 + C
X(x-l)
12. J (x2 + l)(x + l)>/x3 + x,22 + x
10. Jx27 (1 + x + x2)6 (6x2 + 5x + 4)dx =
X + - + 1-1
x4 (1 +x + x2)7 1, X_______
(a) +C = T>0g — A+c.
7 X
i bl? +1,
X 4-----
V X

(b) +C Then the value of A is equal to


.28 .2x7 1 1
(a) cos (b) tan x + —+ l
(0 —+ C X X
28
1
(d) None (c) cot x+- (d) sin x + -I + ,l
x X

Answers Key

Single Correct Answer Type 6. (b) 7. (c) 8. (b) 9. (c) 10. (b)
1. (b) 2. (a) 3. (c) 4. (c) 5. (a) 11. (b) 12. (b)
Indefinite Integration 7.7

DPP
Integration of Rational Functions

Subjective Type (c) —ln(l+ x4)-ln(l+ x) + c


4
(*-l)2
1. Evaluate: J dx (d) — ln(l + x4) + ln(l + x) + c
x4 + x2 +1 4
x2-4 , cos3x
2. Evaluate: j 7. The value of J dx is equal to
x4 + 9x2 + 16 sin2 x +sin x
(a) logjsin xl + sin x + C
(b) log,lsin xl - sin x + C
Single Correct Answer Type (c) -log, Isin xl -sin x + C
(d) -log, Isin xl + sin x + C
r dx3
3. equals 1
x3(x” + l) 8. Jx3 + x dx is equal to
A / f xn
(a) —In
X
(b) —In (a) x - log, lxi + log, (x2 + 1) - tan-1 x + C
n .T+i J n
Lxi + - log/x2 + 1) - tan-1 x + C
(b) x -log, 1x1
(c) — In
[x’ + ll (d) 3/i In
( x"+1
n
n \ X 7 x (c) x + log, 1*1 + ~ l°ge + 1)+ tan"1 x + C
r (x +1)2 dx (d) none of these
4. J x(x2 +1) is equal to

(a) log,x + c (b) log, x + 2 tan-1 x + c Comprehension Type


1
(c) log. —2— + c (d) log, {x(x2+1)}+c
x2 +1 For Questions 9 and 10
dx
rx3-x Letj(x) = f-------- —------ r-
5. dx is equal to J ex + 8e-x + 4e"3x e3x + 8ex + 4e
JT77
1 x4-x2 + l 1 (x2+l): 9. S(fa)-2g(x))dx
(a) -log + C (b) -tan -+C
6 (1 + x2)2 6 2
1 ex + 2e~x-2
(a) zlog +C
x 4 —x 2 + l (x2+l): 2 ex + 2e~x + 2
(c) log +C (d) tan -+C
(1 + x2)2 2
1 ex-2e~x-2>/3
(b) —-7= log +C
(*3-l) 4^3 ex + 2e'x + 2V3
6. J (x4 + l)(x + l) dx is 1 (ex-2e
(c) —7= tan +C
(a) ^ln(l + x4) + ^ln(l + x3) + c 2V3 2>/3 7

1 ex + 2e
(b) -^ln(l+ x4)-^ln(l + x3) + c (d) —tan +C
2 2
7.8 Calculus

10. /(A-v) + 2g(x)dv


ex+2e
ex (c) —tan +C
1 x -2e~x-2jl 2 2
(a) +C
ex + 2e~x+2y/3
1 ex + 2e~x-2
(d) -log +C
_] f ex - 2e 4 ex + 2e'x + 2
(b) —7=-tan +C
273 \ 2x/3

Answers Key
Subjective Type Single Correct Answer Type
. —1j^tan x-l/.v —2 2.v2+1 3. (a) 4. (b) 5. (a) 6. (c) 7. (b)
1. tan +C 8. (b)
V3 73 73 73
2. 5 Comprehension Type
9. (d) 10. (b)
Indefinite Integration 7.9

DPP
Integration Using by Parts

Subjective Type
ZJ 10. j ex(x- l)(x — In x) dx
7 is equal to
1. Evaluate: Jcos 2x Ioge (1 + tan x)dx
2 x - In x x - In x + 1
2. Evaluate: Jsin 4.tx -e,an Xdx (a) e +c (b) ex
x X
J loge(l+ sin2x)/yr
3. Evaluate:
cos2 x x - In x x - In x - 1
(c) ex +c (d) ex +c
[ *logcx
4. Evaluate: J(x2-1)3/2 dx
~7~~ X

11. J"(sin (lOlx) • sin"x)dxequals

Single Correct Answer Type 1 sin(100x)(sin x)


J (a)
100
'-+c
r (2 + sec x)sec x
5. dx =
(1 + 2 sec x)2 cos(100x)(sin x)^+c
(b)
100
(a) --------- !--------- + C
2 cosec x + cot x cos(100x)(cos x)
(b) 2 cosec x + cot x + C
(c)
100
-+c
(c) --------- ---------- + C cos(100x)(cos x)
2 cosec x - cot x (d) f-+c
(d) 2 cosec x - cot x + C 100
6. If fflx)dx = g(x) and fl(x) is differentiable, then ff~'(x) dx
is equal to Multiple Correct Answers Type
(a) g-'(x) + C
(b) xfl + C
(c) xT'(x)-g(f\x)) + C 12. If J
e4x-l
log
+r dx
(d) f'(x) + C e2x -i,
r •> ■> ■> •>

7. (2 In cosec x + sin 2x)t/r t~, r u~, u~ „ ,


sin2 x = — log t - — - — log u + — + C, then
(a) 2 ecolt In Isin xl + c (b) 2e,an t In Isin xl + c
(c) 2 eco1 x In Icos xl + c (d) 2elan t In Icos xl + c (a) u = ex + e (b) it = ex - e
[• 1-7 cos2x /(x) (c) t = ex + e~ (d) t = ex-e~
8. J • 7 1 dx = + C, theny(x) is equal to
sin x cos- x (sin x)7
(a) sinx (b) cosx 13. If j dx = f(x)J\ + ex - 2 log g(x) + C, then
(c) tanx (d) cotx
2x >
9. Ifyix) = Je' tan- x + dx.7(0) = 0 then the /1.x 71 + ex - 1
(l+x-)-J
/I
(a) fix) = x - 1 (b) g(.r) = ^——
71 + e’x +1
value ofy(l) is

(a) efM
14 27
+ 1
V '
I K I
(b) e - + - + 1
U 2
z x Vl + e* +1
(e) g(x) = -==— (d) /(x) = 2(x-2)
Vl + er -1
I n ’71
(c) <7 +1 (d) e
K 2 +1
7 2
7.10 Calculus

Answers Key
Subjective Type Single Correct Answer Type
1. | [sin 2x logf( 1 + tan x) - x + log Isin x + cos xl] + C 5. (a) 6. (c) 7. (a) 8. (c) 9. (a)
2. -2etan2jcos4x+C 10. (d) 11. (a)

3. logf (1 + sin2x) • tan x - 2x + V2 tan-1 (V2 tan x) + C Multiple Correct Answers Type
12. (b, c) 13. (b.d)
- lOge *
4. + sec x + C
ISJ A J? TF' If
1
I

Definite
Integration

DPP 8.1
Definite Integration: Definition

□Z Single Correct Answer Type


.J
5. The number of solutions of the equation
X _

I I cos x I dx = 0,0 < x < — , is


1
1. lim — +
J n2 1
+ ... + — is equal to
< 2
n—>« n (n + 1)5 + (/> + 2)3
(a) 0 (b) 1 (c) 2 (d) 4

(a) | 1
(b) -
4 6. Z’ dx = (where {.} denotes the fractional part
0

(C) I n
(d) None of these function)
(a) ee(2e-l) (b) e‘(e-l)
1____ (c) e2e(e-l) (d) none of these
2. The value of lim E • — is equal to
n—. (/7 + r)7t n
r=1sm
4n cosx . Then the value of J /(-*) dx
7. Let /(x) = lim
1 + (tan-1 x)” o
(a) 2 In (72-1) (b) 4 In (72-1)
is equal to
(c) 4 In (72 + 1) (d) In 72 (a) cos (tan 1) (b) sin (tan 1)
_1_ (c) tan (tan 1) (d) none of these
Ma' a
n a+k i
3. lim V is equal to dx
8. The value of definite integral J equals
n^k=\ na+1 7(x + l)3 (3.r + 1)
o
(a) 1 (b) 2 (c) 1/2 (d) 4
3 (b)
(a) 2-1
4. Let a, b, c be non-zero real numbers such that j (3AX2 +
3 o
2bx + c) dx = J (Sax2 + 2bx + c) dx. Then , x 5/r (d) none of these
(c) tan—
i
12
(a) a + /> + c = 3 (b) a + b + c = 1
(c) a + b + c = 0 (d) a+b+c=2
8.2 Calculus

9 ' 2ff
Isin (x + /) - sinx| . .
9. If^x) is continuous and |/(x)tZv = 4, then the value of 10. lim f -—------ --------- -dx equals
/_°J 1'1
3 0
the integral Jx • f(x‘)dx is (a) 2 (b) 4 (c) 1/4 (d) 1
o
(a) 2 (b) 18 (c) 16 (d) 4

Answers Key
Single Correct Answer Type 6. (b) 7. (b) 8. (a) 9. (a) 10. (b)
1. (a) 2. (c) 3. (a) 4. (c) 5. (a)
Definite Integration 8.3

DPP
Properties of Definite Integration,
Definite Integration of Odd-Even and Periodic Functions

r Single Correct Answer Type

i
7.
nil
J sin x sin 2x sin 3x sin 4x dx =
o
1. The value of I = J(1 + x)I/2(l - x)?/2dx is
(a) I /
(c) —
(b) - (d) —
8 16 32
(a) 7T (b) f 2013
8. j [(x-l)(x-2)...(x-2013)]4&
(c) 2tc (d) none of these
i

2. The value of I = Jx(sin2 (sin x) + cos2 (cosx))efr is (a) (2013)2 (b) (2012)(2013)(2014)
o (c) 2013! (d) 0
9 9. /: [0, 5] —> R, y =flx) such that/'(x) =f"(5 -x) V x e [0,
(a) 7? (h) y 4
5],/*(0) = 1 and/'(5) = 7, then the value of | f'(x)dx is
7T2 i
(d) none of these
(c) T (a) 4 (b) 6 (c) 8 (d) 10
a
n
7t 1 4
ex ■ sec2 xdx
3. J ln(cot a + tan x) dx, where a e
o
is 10. J elx-\
is equal to

(a) a In (sin a) (b) -a In (sin a) 4


(c) -a In (cos a) (d) none of these (a) 0 (b) 2 (c) e (d) 2e
4. If/> +/(>)) =/(x) + y V x, y e R and/(0) = 1, then 11. If/and g are two continuous functions being even and odd,
io a

j /(10 - x)dx is equal to respectively, then j


f(x)
dx is equal to (a being any
bg(x) + l
o
(a) 1 (b) 10 non-zero number and b is positive real number, b * 1)
(a) independent of/
i 1
(b) independent of g
(c) j/(x)<& (d) 10j/(x)cfr
(c) independent of both/and g
0 o
(d) none of these
mi (271 2lt sin nn (
nil A 4ff
5. Let u = J cos| — 2x dx and v = | cos —sinx k/r, 12. If J In 113 sin x + 3^3 cos x| dx = kit In 7, then the value
o 3 o I u J o
then the relation between u and v is of k is
(a) 2u = v (b) 2u = 3v (c) u = v (d) u = 2v (a) 2 (b) 4 (c) 8 (d) 16
100/r ( io
13. /is a real valued function from R to R such that/x) +/(-x)
l+x
6. j y, tan ?x dx is equal to
= 2, then J f~\t)dt =
o v=i >
(a) 0 (b) lOOzr (c) -50 it (d) 50 it (a) -1 (b) 0
(c) 1 (d) none of these
8.4 Calculus

2011
Multiple Correct Answers Type
(c) J B(x)dx = 0
14. If A(x + y) = A(x)A(y) Vx, y and A(0) 0 and -2010

A(x) 2010 2010


B(x) = , then (d) J (25(-x)-5(x))fitr = 2 j B(x)dx
l+«Y))2
-2010 0
2010 2011
n
(a) J 5(x) dx= J 5(x) dx 15. A function / is defined by />) = jcos t cos (x - t) dt,
-2010 0
o
2011 2010 2011 0 < x < 2zr. Then which of the following hold(s) good?
(b) | B(x)dx- j B(x)dx = j B(x)dx (a) /x) is continuous but not differentiable in (0,2n).
-2010 0 0 (b) Maximum value of/is n/2.
(c) There exists atleast one c e (0, 2rt) such thatf(c) = 0.
n
(d) Minimum value of/is -y — .

Answers Key
Single Correct Answer Type Multiple Correct Answers Type
1. (b) 2. (b) 3. (b) 4. (d) 5. (a) 14. (b, d) 15. (b, c, d)
6. (a) 7. (c) 8. (d) 9. (c) 10. (a)
11. (b) 12. (b) 13. (b)
Definite Integration 8.5

DPP 8.3
Definite Integration Using Substitution

(a) 3 (b) 6
Single Correct Answer Type
(c) 9 (d) 12

1. If r du
— , then e* = 8. A function/(x) satisfies/(x) = for some real number
6
4~c
(a) 1 (b) 2 (c) 4 (d) -1 c(>l) and all positive number ‘x’. If J Mi = 3, then
it2 X
2. The value of J [log^xj^logpX) (where [.] denotes f/(x)
e dx is
I greatest integer function) is
(a) 21oge^r (b) logoff
1 x

(a) 4 (b) 6 (C) 8 (d) 9


(c) 1 (d) 0
1 2
XV X ’ ,
dt 9. Let Z.= f—
_0
J VEr '
------- -dx, then
0 (l+x)6
3. The value of is (a) lx = 2L
1
1 (b) Z2 = 2Zt
J 71+7
0
dt (c) Zj=Z22 (d) IX=I2

(a) 1 (b)
(b) 22 (c) 2^3 (d) V2 F Multiple Correct Answers Type
4. Let a and b be two positive real numbers. Then the value
* exla _ (>b/x , L
of] - ---- — dx is 10. If dx =---- then
X
0
2
(a)°0 (b) ab (c) Mab (d) e°b co 00

7 log*
5. The value of J0a2 + x2 dx is (a) Je~2x2dx = 2>/2 (b) jxe'-x2 dr = -
2
0 0

2n log a n log a
(a)
a
(b)
2a (0 J A-2 dx^
4
(d) jx2e~z‘dx = ^
(c) zrloga (d) 0 0

/ •r /
X + X.2 - 1 11. Let /(x) = J—df(x>0);
6. J-ioge
J X X - X2 + 1
dx is equal to 0 1
1/3 7
then e-a[/(x + o)- /(l + a)] =
(c) 0 (d) 3
(a) /
(/ + a)
(b) JJx}'-^-dt
t+a
0
7. IfZ.
““ ~n x+a t
(c) e~a f — dt (d) J -—dt
(2n - 3) x'1"2 + (2/i - I)*"-1)dx,neN, then the value of
7^9 is
tia( 0 (* + a)

Answers Key
Single Correct Answer Type Multiple Correct Answers Type
1. (c) 2.2. (b)
(b) 3. (d) 4. (a) 5. (b) 10. (a, b, c) 11. (b,c)
6. (c)“ 7.7. (c)
(c) 8. (b) 9. (d)
8.6 Calculus

DPP

Leibnitz Rule

□ jr*(l+.r)
Single Correct Answer Type 1 (a) e
(c) 4
(b) 4e
(d) none of these
f-a+I 1
7. The maximum value of the integral JJ a-t fdx is
1. If | f(t)dt = x, then the value of/2) is j + x‘
o attained
(a) exactly at two values of a
(a) 1/2 (b) 1/3 (c) 1/4 (d) 1/5
(b) only at one value of a which is positive
2. If /(x) = J logo 5 | 2/ -8 dt, then the interval in which (c) only at one value of a which is negative
t-2 (d) only at a = 0
0 '
/x) is increasing is
(a) (-°°. 2) kJ (6, °°)
I-L
pyla + t
dt
(b) (4.6) * 8. lim = l(a > 0). Then the value of a is
x-»b x-sinx
I (c) (-°°, 2) kJ (4, co)
(d) (2.6) (a) 1/2 (c) 1/4 (c) 2 (d) 4
3. If a, b and c are real numbers, then the value of 9. Let /(x) be a differentiable non-decreasing function
fl' X
(x YV
limloge -|(1 + a sin bx)c/x dx equals
such that f(/(/))3d/ = 1
r-»o xg/?-{0} and
o x2 ko J
„ . ob x
(a) abc (b) —
c /(l)=l.If|7(/)<// = g(x) then is
o
. s bc ca
(c) — (d) — (a) always equal to 1
a b (b) always equal to -2
(c) may be 1 or -2 (d) not independent of x
J/ , 10. Let/be continuous and the function g is defined as
4. HJW= J -----
1+/4
t, then X ( t \
2 g(x) = | t2 • I f (u)du dt where/1) = 3. Then the value
Ok 1;
(a) /<3)<py (b) A3)>py of #'(1) + g"(l) is
(a) 1 (b) 2 (C) 3 (d) 4
(c)/l3)=py
X
(d) /3)>1
r Multiple Correct Answers Type

5. If J/(x) sin /dt = constant, 0<x<27rand/» = 2, then X

o 11. Let/x) = J f(t2 - 3/ + 4)dt. Then


the value offljd2) is 2
(a) 3 (b) 2 (c) 4 (d) 8 (a) /2) = 0 (b) /-2) = 0
6. A function/, continuous on the positive real axis, has the (c) /'(2) = 0 (d) /'(2) = 2
property that for all choices of x > 0 and y > 0, the integral
X

•yy 12. If \f{t)dt = e-r - ae dt, then


j /(/) dt is independent of x (and therefore depends only o 0
x e
on y). If/2) = 2, then |/(/) dt is equal to
i
Definite Integration 8.7

(a) a =
1
3-2e
(b) /(x) = et-2e2x Comprehension Type

For Questions 14 and 15



(c) a = - (d) JW^-e Consider the function
e
13. A function fix) satisfies fix) = j (5 In (1 + t2) - 10/ tan-1/ + 16 sin /) dt
X o
fix) = sin x + J/'(/) (2 sin t - sin2/) dt 14. fix) is
o (a) negative for all x g (0, 1)
Then (b) increasing for all x g (0, 1)

/g)., (c) decreasing for all x g (0, 1)


(d) non-monotonic function forxG (0, 1)
X
X
15. Which is not true for J f(t)dt 1
(b) g(x) = J./W dt is increasing on (0, n)
o
o
(a) positive for all x g (0, 1)
(c) /(0) = 0
(b) increasing for all x g (0, 1)
(d) fix) is increasing on (0, n)
(c) non-monotonic for all x G (0, 1)
(d) none of these

Answers Key
Single Correct Answer Type Multiple Correct Answers Type
1. (d) 2.
2. (b)
(b) 3. (a) 4. (a) 5. (c) 11. (a, c) 12. (a, b) 13. (a, b, c)
6. (c) 7.
7. (d)
(d) 8. (d) 9. (a) 10. (c)
Comprehension Type
14. (b) 15. (c)
8.8 Calculus

DPP

Miscellaneous Exercises

Subjective Type x ifx<0


(c) /(x)=
ex ifx>0
dx
1. Prove that Jrr------- 1
>1---- for n 6 N x ifx<0
o 1 + x" n (d) /(x) =
ex -1 ifx>0

Single Correct Answer Type co

626j e sin25 xdx


ff/2
2. Let In = J (sin x + cos x)n dx (n > 2). Then the value of n. 7. o ----------- is equal to
n
o sin xdx
4-2(«-l)/n-2 is 0
(a) 5 (b) 9 (c) 2 (d) 7
(a) 300 (b) 625 (c) 600 (d) 1200
3. Let/.n dx . Then
4
hm -— = 8. If g(x) is the inverse of /(x) and fix) has domain x e
”-*“4-2 [1, 5], where/(I) = 2 and/(5) = 10 then the value of
5 10
(a) 2 (b) 1 (C) -1 (d) -2
J7(x)^+ is
1 1 2
4. If dx = —, then Jx" e 'mdx is
o a o (a) 72 (b) 56 (d) 48
(c) 36
2n
(a)
(-1)^!
(b)
(-W-1)! 9. If/x = x + sin x, then J (x)dx is equal to
an+} o”
n
, . n\ ( . 3/r2 o (b) ^- + 2
C a"+1 (d) none of these
(a) ——2
2 2
5. Let n > 1, n e Z. The real number a e 0,1 that minimizes (c) 3/r2 (d) none of these
i
the integral J| x,nn-an\dx is 10. Given a real-valued function f which is monotonic and
f(b)
o differentiable. Then J 2x(b-f"\x))dx =
(a) | (b)
(b) 22 (c) 1 (d) i /(*)
(c) 1
b
6. Let f be a continuous functions satisfying /'(In x) = W J(/2(x)-2/2(a))A
1 for0<x<l a
and/(0) = 0. Then/(x) can be defined as
b
x for x > 1
(b) J(2/2(x)“/2(a))dx
fl if x< 1 a
(a) 7(x) = b
l-ex if x> 1
<0 J(/2(x)-/2(a))<&
1 if x<0 a
(b) /(x)
ex-\ ifx>0 (d) None of these
Definite Integration 8.9

2 It is known thaty(7i) for n > 0 is finite and g(m, n)= g(n, m) for
dx
11. If / = Jj r~i—2 then m, n > 0.
1 V2x3-9x2 + 12x + 4
7 x"-1 dx =
13. J0(i+xr+n
1_ I 2
/8 4 3 (a) g(m, n) (b) g{m-\,n)
(c) g(/n-l,n-l) (d) g(m,n-l)
(c) !</<o (d) none of these n
4
14. fx” log/ I dx-
12. Consider the function h{x) =
2
+ 3x3-5, where g(x)
i < x>
2 /(« + !) f(n)
is a continuous and differentiable function. It is given that (a) (b)
(w+ir (m + l)"+1
h(x) is a monotonically increasing function and g(0) = 4.
Then which of the following is not true? ■/•(* + !)
(c) (d) g(m + l,n+ 1)
(a) g2(l)> 10 (b) A(5)>3 (m+i)"+i

(c) <2 (d) g2(-l)<22


15.
jx'"-1 +x dx =
o (\ + x)m+n
Comprehension Type (a) g(n,m) (b) g(m- l,n+ 1)
(c) g(m- l,n— 1) (d) g(m+ 1, zi — 1)
For Questions 13-15

Let m, n be two positive real numbers and define/(n) = j x‘ e Xdx


i o
and g(tn, ti) = Jx':m“1(l-x)i dx.
o

Answers Key

Single Correct Answer Type Comprehension Type


2. (c) 3.3. (b)
(b) 4. (c) 5. (a) 6. (d) 13. (a) 14. (c) 15. (a)
7. (c) 8.8. (d)
(d) 9. (b) 10. (c) 11. (a)
12. (c)
h
is] /a\ lP it u rg
I

A
I

DPP 9.1
Area Bounded by Curves (Elementary)

r— Single Correct Answer Type j


(a) 4A + 8 cos a = 1 (b) 4A + 8 sin a = 7
(c) 4A - 8 sin a = 7 (d) 4A - 8 cos a = 7
1. The area enclosed between the curves y = sin2 x and 6. The area in the first quadrant between x2 + y2 = Tt1 and
y = cos2x in the interval 0 < x < tt is y = sin x is

(a) 2 sq unit (b) sq unit (a) <b)


4 4
(c) 1 sq unit (d) None of these (X-16) (ft3-8)
(c) (d)
2. The area of the region(s) enclosed by the curves y = x2 and 4 2
y= y/\x\ 7. The area bounded by the curves y = cos-1 x: y = sin"1 x and
y = -tex3, where -1 < x < 1, is
(a) 1/3 (b) 2/3 (c) 1/6 (d) 1
3. The area of the region bounded by y = e\ y = e~x, x = 0 and (a) — +1 - V2 sq. units
x = 1 is

(a) e + — (b) Iog(4/e) ■ - 4

e
(c) 4 log (4/e) (d) e + - -2 (c) — + 2 - V2 sq. units
e 4
4. The area bounded by the curve (d) +1 - 5/2 sq. units
y - lcos~*(sin x)l - Isin-1 (cos x)l and x-axis from
8. The area bounded by the curve y = sin2x - 2 sin x and the
— <x<2jc is x-axis, where x e [0. 2jt] , is
2
(a) n2 sq. units (b) tt/4 sq. units
(a) 4 sq. units (b) 8 sq. units
(c) 16 sq. units (d) 20 sq. units
(c) 7^/2 sq. units (d) none of these
9. Consider the functions fx) and g(x), both defined from
5. If (a, 0), a > 0, is the point where the curve y = sin 2x - /? —> R and are defined as/(x) = Tx-x2 and g(x)=x/l where
V3 sin x cuts the x-axis first, A is the area bounded by this n e N. If the area between fx) and g(x) is 1/2, then the
part of the curve, the origin and the positive x-axis. Then value of 71 is
(a) 5 (b) 6 (c) 7 (d) 8
9.2 Calculus

10. Let a function /(x) is defined in [-2, 2] as (a) |(4-s)w (b) 8(4-7r)3/2
{x}» -2£x<-l
jV) = • |sgn x|, -1 < x < 1 , where {x} and sgnxdenotes
Q
(d) |(4-7<
(c) -(4-?r)3/2
{-x}, l<x<2
fractional part and signum functions, respectively. Then the
Multiple Correct Answers Type
area bounded by the graph offix) and x-axis is
(a) 2 sq. units (b) 3 sq. units 15. If the area bounded between x-axis and the graph of
(c) 4 sq. units (d) 5 sq. units y = 6x - 3X2 between the ordinates x = 1 and x = a is
11. The area bounded by y = x2 + 2 and y = 21 xl - cos nx is 19 square units, then a can take the value
equal to (a) one value in (2, 3) (b) one value in (-2,-1)
(a) 2/3 (b) 8/3 (c) one value in (-1,0) (d) one value in (3,4)
(c) 4/3 (d) 1/3 16. Which of the following is the possible value/values of c
x2-l
12. Area bounded by/(x) = and the line y = 1 is for which the area of the figure bounded by the curves
x2+l y = sin 2x, the straight lines x = n/6, x = c and the abscissa
(a) jrsq. units (b) 2n sq. units axis is equal to 1/2?
(c) Tdl sq. units (d) none of these (a) “7 \
(b) t (d) none of these
13. The area bounded by the curve y = x e~x, y = 0 and x = c, O 3 (c) 7
where c is thex-coordinate of the curve’s inflection point, is
17. Area of the region bounded by the curve y - tan x and lines
(a) l-3r2 (b) l-2r2
y = 0 and x = 1 is
(c) 1 - e~2 (d) none of these
t tanl
16-x.2: (a) jtan(l-x)dr (b) tan 1 - J tan-1 ydy
14. Area of the region bounded by the curve y = - and
4 o o
y = sec-1 [-sin2 x] (where [.] denotes the greatest integer tanl i
function) is (c) j tan-1 y dy (d) J tan-1 x dx
o o

Answers Key
Single Correct Answer Type Multiple Correct Answers Type
1. (c) 2. (b) 3. (d) 4. (b) 5. (a) 15. (b,d) 16. (a, b) 17. (a, b)
6. (a) 7. (d) 8. (b) 9. (a) 10. (b)
11. (b) 12. (b) 13. (a) 14. (c)
Area 9.3

DPP

Area Bounded by Curves (Typical)

r Single Correct Answer Type

1. Suppose y = fix) and y = g(x) are two functions whose


□ 5. Area bounded by the min. {Ixl, lyl} = 1 and the
max. {Ixl, lyl} =2 is
(a) 4 (b) 8 (c) 16 (d) 9
graphs intersect at the three points (0,4), (2, 2) and (4, 0)
withy(x) > g(x) for 0 < x < 2 and/x) < g(x) for 2 < x < 4. COS X 0<x< —
4 4 2
6. Consider/x) = < such that /is pe-
If Jl/W - g(x)]dr = 10 and J[g(x) -fix)]dx = 5, the area n r2 n
o 2 . ------ X„. — < X < It
2 ) 2
between two curves for 0 < x < 2, is
riodic with period it. Then which of the following is not
(a) 5 (b) 10 (c) 15 (d) 20
true?
2. The ratio of the areas of two regions of the curve
Cj = 4X2 + n?y2 = 4H2 divided by the curve C2 = y = (a) The range of/is n0, — II.
-^sgn^x - cos x (where sgn (x) = signum (x)) is
(b) /is discontinuous for infinite values of x.
(c) The area bounded by y = fix) and the X-axis from
_2 n it2+ 2
it —2 (
(a) (b)
it2 + 2 it2-2^2 x = 0tox = n^is rz 1 +— for a given neN.
I 24.
it2 + 6 n2-\ (d) None of these
(c) (d) ^/2
7T2 + 3>/2 it 7. The area made by curve fix) = [x] + yjx - [x] and x-axis
3. The area bounded by the curves when 0<x<n (n e (V) is equal to {where [x] is greatest
integer function}
x^> + y = 2 loge(x - y^3) - 2 loge 2, y = >/3x,
2n n(n + 1) z. x n n(n +1)
(a) — + —----- - (b) - + —----- -

y = —1j=x + 2. is 3 2 3 2
V3 22+«(hS1) n +^1)
(a) 2 log^ 2 sq. units (b) 2 log? 2 + 1 sq. units 3 2 3 2
8. Consider the regions A = {(x, y) I x2+ y2 < 100} and B =
(c) 2 loge 2 - 1 sq. units (d) 4 logf 2-1 sq.units
{(x, y) I sin (x + y) > 0} in the plane. Then the area of the
4-x.2 region A n B is
4. Area of region bounded by the curves y =
4 + x2 ’ (a) 10/r (b) 100 (c) 100/r (d) 50zr
, 3,, 6 9. Let R be the region containing the point (x, y) on the X-Y
25y2 = 9x and y = - Ixl---- which contains (1, 0) point
, 5” 5 plane, satisfying 2 < lx + 3yl + lx -yl < 4. Then the area of
in its interior is this region is
(a) 5 sq. units (b) 6 sq. units
. t -i I ll
(a) it-4 tan — + — ? sq. units (c) 7 sq. units (d) 8 sq. units
2 lOj
W for x« Z
-1 - -'I sq. units
10. If /(*) and g(x) = {x}2 where {.}
(b) 7T-2 tan 1 for x e Z
denotes fractional part of x then area bounded by/x) and
(c) it + 4 tan - •I sq. units g(x) forxe 0, 6 is
2 2 , x 10
(a) j (b) 2 (d) 6
(d) none of these (c) T
9.4 Calculus

11. Let S is the region of points which satisfies y~ < 16x,x < 4 Comprehension Type 1
xy(x2 - 3x + 2)
and > 0. Its area is
x2-7x + 12 For Questions 14 and 15
, , 8 . 64 , \ 32 In the following figure, the graphs of two functions y =flx) and
(a) - (b) — (d) noneofthese
3 3 (c) T y = sin x are given. They intersect at origin, A(a,fla)), B(n, 0)
2 and C(2n, 0). At (i =1, 2, 3) is the area bounded by the curves
12. The area of the region {(x, y):.r i, ILxl- lyll > I is
as shown in the figure, respectively, for x e (0, a), x e (a, ri),
f
x e (jt, 2tt).
(a) 4 n - tan
I"))-’
o
If Aj = !+(«- l)cosa-sin a, then

(b) 5 fr-tan

24 ^/y - s'n -1
(c) 3 br-tan -4 C
7 O TtK 3tz/2 tin
24 ^3
(d) 2 7r-tan -1
7
13. The following figure shows the graph of a continuous
function y =7(x) on die interval [1,3]. The points /L B,
C have coordinates (I, 1), (3. 2), (2, 3), respectively, and
the lines A, and L, are parallel, with being tangent to
the curve at C. If the area under the graph of y =f(x) from
14. The functionXx) is
x = 1 to x = 3 is 4 square units, then the area of the shaded
region is (a) x2sinx (b) xsinx
y=7(x) (c) 2xsinx (d) x3sinx
(2,3)
C 15. The value of A2 is
L, (a) (tt— 1) units2 (b) (n/2 - 1) units2
A (3,2) (c) (tt—sinl — 1) units,2 (d) n/2 units2
(1,1)
4------ —t-
1 3

(a) 1 (b) 2 (c) 3 (d) 4

Answers Key
Single Correct Answer Type Comprehension Type
1. (c) 2. (a) 3. (c) 4. (a) 5- (a) 14. (b) 15. (c)
6. (d) 7. (c) 8. (d) 9. (b) 10. (b)
11. (b) 12. (b) 13. (b)
Differential
Equations

DPP 10.1
Forming Differential Equation, Differential Equations with
Variable Separable, Homogeneous Differential Equations

-------- ---------------------
Single Correct Answer Type ; 6. The general solution of -L
dx
1. The order of the differential equation whose general
solution is y = C] cos 2x + c2 cos2x + c3 sin2x + c4 is (a) 2sin ly = Xyj\-y7 + c
(a) 2 (b) 4 I
(c) 3 (d) None of these (b) sin-1 y = — sin-1 x + c
' 2
2. Order of the differential equation whose general solution
(c) cos' y = x cos'
• ax , , ,■
is yy =
= ------- , where a, b, c are arbitrary constants is x3
bx + c ... 1 . 1+/
(d) - log =x- — + c
(a) 1 (b) 2 2 1-yJ 3
(c) 3 (d) none of these 7. The solution of the differential equation e~x (y + l)dy +
3. If p and <?are order and degree of differential equation (cos2x - sin 2r)y (dx) = 0 subjected to the condition that
2 1 y = 1 when x = 0 is
I a ( dy i3 9 9 .
(a) (y + l) + e'cos2x = 2
y2 + 3x — + x y = sin x, then
< dx~ \dx J (b) y + log y = ex cos2 x
(c) log(y+ l) + ercos2x = 1
(a) p>q (b) ^ = 1 (d) y + log y + e'rcos2x = 2
7 2
8. The solution of the differential equation
(c) p = q (d) p<q
4. If m and n are the order and degree of the differential (x2 -xy2) — + y2 4-xv2 = 0 is
dx
equation (y")5+ 4 • (/y
/// + y= sin x, then
y i fx) Ii 1I My I1 i1
(a) log —=—+—+ c (b) log —=—+—+ c
(a) in = 3,n = 5 (b) in = 3, n = 1 kT 1 X y x " y-
X

(c) m = 3,n = 3 (d) m = 3,n = 2 1 1 1 1


(c) log (xv) = — + — + c (d) log (xy) + — + — = <?
5. If the differential equation _ d*. ... += o represents x y x y
3y + f px + g
a family of circle, then p =
(a) g (b) f (c) 4 (d) 3
10.2 Calculus
9. The family of curves passing through (0,0) and satisfying 2
2
(C) tan x
the differential equation 21 = 1 (where yn = (Py/dxn) is

(a) y = k (b) y = kx (d) ^ = /cotx


(c) y = A(et+l) (d) y = k(e*-V) dx
2
10. The solution of the differential equation 13. If the solution of the equation ^-^ + 4— dx + 3x = 0
y^dx + (x2-xy + y*)dy = 0 is
dt2 dt
(a) tan . dx _
given that for *t —
ivi 0, x —
= V) and — = 12 is in the form
= v0 uiiv
x = Ae~2' + Be~‘, then
(b) 2 tan + In x + C = 0 (a) A + B = 0 (b) A + B=12
(c) LABI = 36 (d) LABI = 49
(c) =0 2 ( 11 \dy
14. The solution of -2|x + -F+l = 0
,2 ,2 4xJ dx
(d) ln(x+ +y +C = 0
11. The solution of differential equation
(a) y = x2 + c (b) y = In (x) + c, x > 0
(1 - xy + x2y2)dx = x2dy is
(a) tan xy = log Icxl (b) tan(y/x) = tan log Icxl
x2
(c) xy = tan log Icxl (d) None of these (c) y=^+c (d) y=—+c

Multiple Correct Answers Type dy |2 dy


dy
15. + 2ycotx— = y 2 has the solution
dx dx
12. The differential equation for the family of curves y = c sinx
can be given by c
2
(a) y- —-—=o (b) y=
1 + COS X 1 - COS X
(a) = y2 cot2 x
/ c
(c) x = 2sin (d) x = 2 cos

“> gj'-p* I + y2=0 dy


dx
2 \?y

Answers Key

Single Correct Answer Type Multiple Correct Answers Type


1. (a) 2.2. (b)
(b) 3. (d) 4. (d) 5. (d) 12. (a, b, d) 13. (a,c) 14. (a,b) 15. (a, b)
6. (d) 7.7. (d)
(d) 8. (a) 9. (d) 10. (a)
11. (c)
Differential Equations 10.3

DPP 10.2
Linear Differential Equations

(a) x(sin y + cos y) = sin y + cd'


Single Correct Answer Type
(b) x(sin y + cos y) = sin y + ce~y
1. If y(t) satisfies the differential equation y'(t) + 2y(t) - 2e-2/, (c) y(sin x + cos x) = sin x + cd
y(0) = 2 then y( 1) equals (d) None of these
4 dy = y4 cos x is
7. Solution of differential equation x^y - x3 —
(a) -3 (b) 43 (c) - (d) — dx
e e e e (a) x2y~3 = 2 sin x + c (b) x2y-3 = 3 cos x + c
2. If ^-ylog 2 = 2sinx (cos x -1) log, 2 , then y = (c) x3y-3 = 3 sin x + c (d) x2/5 = 3 sin x + c x^
dx 8. Suppose a solution of the differential equation
(a) 2sinx+c2x (b) 2C0SX + c2x (xy3 + x1y1)^- = 1 satisfies the initial conditions
(c) 2sinx + c2-x (d) 2C0SX + c2-x dx
3. If ye? dx = (y3 + 2xd~)dy, y(0) = 1, then the value of x when 1. Then the value of — when y = -1 is
y = 0is dx
(a) -1 (b) 0 (c) 1 (d) 2 . x 3 ... 20
4. If y,(x) is a solution of the differential equation (a)----- (b)
20 3
— ~f(x)y = 0, then a solution of the differential equation / \ 5 16
dx <c) — (d) --
lo J

+fx)y = r(x) is
~ +J(x)y
— 9. The general solution of the differential equation
dx
= y tan x - y2 sec x is
1 c
(a) y= Jr(x)^,(x)«& + (a) tan x = (c + sec x)y (b) sec y = (c + tan y)x
hW
(c) sec x = (c + tan x)y (d) none of these
r(x)
(t>) >=Ji(x) J ------ dx + c
yi(x)
Multiple Correct Answers Type
(c) y = ^r(x)yfix)dx + c
dy
10. Let — + y = /(x) where y is a continuous function of x
(d) none of these dx ' ’
i,
—logx with y(0) = 1
5. The general solution of x + dog x)y = X 2 is
e if0<x<2
1,
and fix) = - . Which of the following
l--logx --logx e ifx>2
2.
(a) y = x + CX 2
hold(s) good?
i, i, (a) y(l) = 2e~x (b) y'(l) = -e’1
-logx -logx
(b) y-x2 = x2 +c (c) y(3) = -2e-3 (d) y'(3) = -2e
(logx)2
11. A differentiable function satisfies
(c) y = e' 2 (x + c) fix) = J(/(r)cosf-cos(f-x))dt. .
|(logx)
„ J2 1—(logx) 1.
—logx)
X o
(d) y = e2 (x 2 -x 2- )+c Which of the following hold(s) good?
6. The general solution of the differential equation (a) fix) has a minimum value 1 - e
(1 + tan y)(dx - dy) + 2xdy = 0 is (b) fix) has a maximum value 1 - e~l
10.4 Calculus

Jn (a) x = 1 + V2 is point of local maxima


(c) f 12 =e
(b) x = 1 - V2 is point of local minima
(d) r(0)=l
(c) lim /(x) = -«>
12. y =./U') which has differential equation y(2xy + ex)dx-exdy
= 0 passing through the point (0, 1). Then which of the
following is/are true about the function? (d) lim /(x) = 0

Answers Key
Single Correct Answer Type Multiple Correct Answers Type
1. (d) 2. (a) 3. (b) 4. (a) 5. (a) 10. (a, b, d) 11. (a, b, c) 12. (a,b,c,d)
6. (b) 7. (c) 8. (d) 9. (c)
Differential Equations 10.5

DPP 10.3
General Form of Variable Separation

7. The solution of
Single Correct Answer Type
(y( 1 + x-1) + sin y)dx + (x + log x + x cos y)dy = 0 is
1. The solution of differential equation x2(x dy + y dx) = (a) (1 + y*1 sin y) + x"1 log x = c
(xy - I)2 dx is (where c is an arbitrary constant) (b) (y + sin y) + xy log x = C
(a) xy - 1 = ex (b) xy - 1 = ex2 (c) xy + y log x + x sin y = C
1 1 (d) none of these
(c) ------- = —+ c (d) None of these dy + 1 =
xy-1 x 8. The solution of (1 + x) — is
dx
2. Solution of...the differential
.... dx y is
- fix
(x + 2y3) = — (a) e-v(x + 1) = c (b) ^(x + 1) = ex+ c
(c) ev(x + 1) = ce* (d) (x + 1) = e1 + c
(a) x = y2(c + y2) (b) x = y(c-y2) 9. The solution of differential equation
(c) x = 2y(c-y2) (d) x = y(c + y2) y (xcfy-y cfr) is
Ji
3. General solution of differential equation
y 1
xsec I — (y dx + x dy) = y cosec (—
lx X
dy ( dy 1 y 2 y
x2 x + y— + x— - y x2 + y2 = 0 is (a) xy = c cosec — (b) xy sin — = c
dx) I dx
dx lx x
1 y y
(a) +—=c
yjx2 + y2 x
(b) 7^2 + / -- = c (c) xy cosec— = c (d) .xy = c sin —
I X
x x
10. The general solution of the differential equation
(c) ^x2 + y2 +- = c (d) 2 Jx2 + y2 + — = c
x x ~ x2y2 dx = y dx +x dy is
(a) sin (xy) = x 4- c (b) sin"1 (xy) + x - c
3yx2
4. Solution of the differential y' - is (c) sin (x + c) = xy (d) sin (xy) + x = c
x3 + 2y4
11. The solution of (l + .ry)ycir + (l-xy)xrfy = O is
. \ 3 -1 23 , . 9-1 2 3
(a) x y ~-^y+c (b) x~y =-y +c . . x 1 , x 1
(a) - + — = k (b) log - = — + k
y xy \yj xy
-1 2 3
(c) xy = -y +c (d) None of these , x X 1 ( xx\
(c) — + — = k (d) log — = xy + k
y xy \y)
5. For y > 0 and x e R, ydx + y2dy = xdy where y =/(x). If
1) = 1, then the value of/(-3) is dy
12. Solution of — = is
(a) 1 (b) 2 (c) 3 (d) 4 dx e2x + y2
6. An equation of the curve satisfying x dy - y dx =
.2x2
yjx2-y2 dx and y( 1) = 0 is (a) e ~2xy2 + 21n|y| = c (b) e~xy~ - 21n |y| = c
(a) y = x2 log Isin xl (b) y = x sin (loglxl) (c) ex + In |y| = c (d) none of these
(c) y2 = x(x-l)2 (d) y = 2?(x-l)

Answers Key
Single Correct Answer Type 6. (b) 7. (c) 8. (b) 9. (c) 10. (c)
1. (c) 2. (d) 3. (c) 4. (a) 5. (c) 11. (c) 12. (a)
10.6 Calculus

DPP 10.4
Applications of Differential Equations

(a) for zi=l, equation of the curve is 2y = 3x


Single Correct Answer Type (b) for n = 1, equation of the curve is 2y2 - 9x
1. A population grows at the rate of 10% of the population per (c) for n = 2, equation of the curve is 2y = 3x
year. How long does it take for the population to double? (d) for n = 2, equationof the curve is 2y2 = 9x
(a) 2 log 10 years (b) 20 log 2 years 7. Let C be a curve such that the normal at any point P
(c) 10 log 2 years (d) 5 log 2 years on it meets x-axis and y-axis at A and B respectively. If
2. A tangent drawn to the curve y =/(x) at P(x, y) cuts the BP : PA = 1 : 2 (internally) and the curve passes through
x and y axes at A and B, respectively, such that the point (0,4), then which of the following altemative(s)
AP : PB = 1 : 3. lf/(l) = 1 then the curve passes through is/are correct?
(a) The curves passes through (V10, - 6).
| k, - | where k is
I (b) The equation of tangent at (4, 4a/3) is2x+ >/3y =20.
(a) I (b) 2 (c) 3 (d) 4
(c) The differential equation for the curve is yy'+ 2x = 0.
3. A curve C with negative slope through the point (0,1)
(d) The curve represents a hyperbola.
lies in the first quadrant. The tangent at any point P on it
8. The normal at a general point (a, b) on curve makes an
meets the x-axis at Q such that PQ = 1. Then the curve in angle 0 with x-axis which satisfies b(a2 tan 0 - cot 0) =
parametric form is a(b2 + 1). The equation of curve can be
(a) x = cos 9 + In tan(0/2), y = sin 9
(b) x = -cos 9 + In tan(0/2), y = sin 0 (a) y = ex 12 + c (b) log ky2 = x2
(c) x = -cos 0 - In tan(0/2), y = sin 0
(d) None of these (c) y = kex 12 (d) x2 - y2 = k
4. Tangent to a curve intersect y-axis at a point P. A line
perpendicular to this tangent through P passes through
point (1,0). The differential equation of the curves is Comprehension Type

/ \ dy dy I2 d2y (dy 2 For Questions 9 and 10


(a) y--j--x = 1 (b) Jc-y+l — I =1 Tangent is drawn at the point (x(, y() on the curve y =fix), which
dx dx ' dx dx)
intersects the x-axis at (xl+1, 0). Now again tangent is drawn at
dx (xi+1, y/+1) on the curve which intersects the x-axis at (xi+2,0) and
(c) y — + x = l (d) None of these
dy the process is repeated n times i.e. i = 1, 2, 3,..... n.
9. If xb x2, x3,.... . x„ form an arithmetic progression with
5, The orthogonal trajectories of the family of curves
common difference equal to log2e and curve passes through
a" "1 y = x" are given by
(a) x" + n2y = const (b) ny2 +X2 = const (0,2), then the equation of the curve passes through
(d) n2x-y" = const (a) (1,4) (b) (5, 1/16)
(c) n2x+ / = const
(c) (2,1/2) (d) none of these
10. If Xj, x2, x3, xn form a geometric progression with
E Multiple Correct Answers Type common ratio equal to 2 and the curve passes through (1.
2), then the curve is
6. Suppose a curve whose sub tangent is n times the abscissa (a) circle (b) hyperbola
of the point of contact and passes through the point (2,3). (c) ellipse (d) parabola
Then
Answers Key —?------
Single Correct Answer Type Comprehension Type
1. (c) 2. (b) 3. (c) 4. (a) 5. (b) 9. (b) 10. (b)
Multiple Correct Answers Type
6. (a.d) 7. (a, d) 8. (b,c,d)
Solutions
S.2 Calculus

CHAPTER 1

DPP 1.1 7. (a) Here, log


;_^_=0
x(y-l)
y2
or =1
Single Correct Answer Type x(y-l)
or y2 = xy-x
n or y2 - xy + x = 0
l.(c) 0<x< —
2
0 if 0<x<l _ x± 7x2-4x
y"
w 1 if 1<X< —
2
2 So y is real if
x2-4x£0
sin 0 = 0 if 0<x<l
or x (x - 4) 2:0
sin[x]
sin 1 if l<x<- x<0orx>4.
2
n Bui x > 0 for log x to be defined.
We have 0 < sin x < I when 0 < x < — • So, x>4
2
y2
[sin x] = 0 for 0 < x < y Now, x=-L->4
y-1

fo if 0<x<l
or
y2 - 4y + 4
>0
sin[x] + [sin x] = y-i
sin 1 if l<x<-
2 or (y-2)2/(y-l)>0
1 y> 1
2. (b) Here0(-x) =
l + ex But y > I for log (y - 1) to be real.
1 1 8. (d) We have,
So, 0(x) + 0(-x) = +------ y = [x2]-[x]2, xe [0,2]
1+e l + ex
i.e., y = [x2]. 0<x< 1
ex 1 y = [x2]-i, 1 £x<2
+----- —»l
ex + l l + ex ex + l y=0 x=2
S= {0(5) + 0(-5)) + ... + {0(1) + 0(-l)) + 0(0) i.e., y = 0, 0<x< 1
= 1-1=0, 1 <x< 72
1 c 1 11
= 1 + 1 + 1 + 1 + 1 + 0(O) = 5 + ----- n = 5 + — = — = 2-1 = 1, x/2 <x< y/3
l + e° 2 2
= 3-1=2, y/3 £x<2
3. (c) X1,1) =X0.XI,0)) = fl0.X0, 1)) =X0,2) = 3
=0 x=2
4. (b) f.R^Q and g : R -> Q, but V3/(x) + g(x) = 4 it is true only,
Hence, the range is [0,1,2}.
when fix) = 0 => g(x) = 4
9. (d)
(l-Xx))3 + te(x)-3)3 = 2
x2-2x
5. (c) g(n)=fin)+fin + 1) fix) - sin-1
If n is even, n + 1 is odd. 3
g(n) = 2/i - 1 + 2(/i + 1)4-2 = 4/14-3 x2-2x x2-2x
If n is odd, n + 1 is even. sin-1 is defined for -1 < <1
3 3
g(/i) = 2/i + 2 + 2(/i + 1) - 1 = 4/i + 3.
6-(c) log^.,, - 3| is defined when x - 3 * 0 and lx2 - 11 > 0 and
|x and >/[x]-[-x] is defined only for integral values ofx

lx2—11*1. => x = -l,0,1,2,3


=> x * 3; 10. (d) We have/(x) = ^-sin’l(>/iog4x) + sin 1 + x2
x2 — 1 * 0 i.e., x*lor-l; 4x ,
Lr- II * 1 -y/2 or 'J2 ; andx*0
=>x* ->J2 2
Clearly, domain of/(x)isx= 1 only,soXI)= To + sin
Hence, the points at which the function is not defined are 4 6
I ft 1
x = 0, 1,-1, >/2,->/2, 3. Hence, range of/(x) is < — >.
16 J
Solutions S.3

11. (c) We have Now, (x]>2=>x2>2 (ii)

fix) = tan'
f x2 + l ,xg R
[x]=-2=>-2<x< 1
<x2 + V3> [x]=-l =>-l <x<0
x2 + l (x] = 0 => 0 S x < 1.
Let y=- Hence [x] = -2, -1, 0 => -2 £ x < 1
X
Hence Dy = [-2, l)u[2, 3).
1
15. (c) Here,/(x) = — (sin-1 x + tan-1 x) +
=> (V3y-1) = (l-y)x2 4
* (x + l) +
(x + 1)
= g(x) + /j(x)
>0
i-y where, domain of g(x) e [-1, 1]
3
Maximum value of g(x) = g( 1) = — and minimum value of
So, 4
y-i g(x) = g(-l) = —7
4
4
ye
i-1 Also, maximum value of h(x) occurs when (x + 1) + --------- is
minimum atx= 1.
3
(x + 1)

Range offix) = Therefore, range offlx) is —, 1 .


L6 4 J . 4 .
12. (c) We must have x4-2lx2 >0 and 10--^x4 -21x2 >0
=> x2(x2-21)>0 (1) Multiple Correct Answers Type 1
and 100^x4-21x2 (2) 16. (a, b, c)
Eq. (1) gives x = 0or x^-VITor x^VIT
Eq. (2)=>x4-21x2-100<0 2 [Isin xi + Icos xi J = 2
(x2-25)(x2 + 4)<0 .*. fix) is defined if

=> x2-25<0 (as x2 + 4 > 0 always) sin2x + 2sinx + — >2


4
=> -5 < x <> 5 or (sinx+ 1)2>
Domain is given by [-5, - -721] u [TH, 5] andx = 0.
or sin x + 1 > — or sin x + 1 < -—
13. (c) fix) = log[x2] (4 - Ixl) + log2 {Vx} 2 2
1 3
For 4-bd>0=>lxl<4=>-4<x<4 or sin x > — or sin < — (which is not true)
2 2
For [x2] > 0, [x2] # 1 17. (a, b, d)
=> [x2] > 2 (a) Clearly, sinfx] + cos(x] is defined for all x.
=> 2 ^X2<oo (b) sin x is always defined and 1 + sin2 x > 1
• =$ sec-1(l+ sin2 x) is defined for all x, as 1 + sin2x £ 1
=> ^2 <x<o°or-oo<x< - 41
(c) tan(log x) is not defined if log x = (2Jt + 1) —
For {Jx} >0, Vx ^J^X*!2 ' 2
9 . . ,.
.-. x = 2, 3 (d) — + cos x + cos 2x
' ' 8
14. (a) We have /(x) = logc {sgn(9 - x2)} + 7w3-4W = 2 cos2 x + cos x + > 0, Vx e R as D < 0
We must have, sgn(9 - x2) > 0
Hence, domain is R.
9-x2>0
18. (a, c)
x2-9<0
2x
=> (x-3)(x + 3)<0 [-2, 2] has range [-1,1], hence bounded.
1 + x2 ’
=> -3 < x < 3 (i)
Also [x]3-4[x]£0 x2
fx)=----- , x e [0,2] - {1} is unbounded asfix) —> when x —> 1
[x]([x]2-4)£0 1-x
[x)([x]-2)([x] + 2)>0 x3 - 8x + 6
=>
fx) = , [0,5] is bounded because domain does not
(x] £ 2 or [x] lies between - 2 and 0, 4x + l
i.e. [x] - -2, -1 or 0 contain-1/4.
S.4 Calculus

, x sin x + sin 3x
DPP 1.2 ’•0» Z(x)=eosx7c^
2 sin 2x cos x
=---------------= tan 2x
2 cos 2x cos x
Single Correct Answer Type
it it
where x # (2n +1)—, (2n +1)—
l.(b) G(x) = M~ + |W) it
( a -1 -) f\ x + — | = tan 2| x + — = tan(2x + it) = tan 2x
I 2J I 2. 2
’ 1
G(-x) = it seems that y is a period but it is not because7(0) is defined
a“x- 1

7 a’
whereas is not defined.
\l-ox
Thus, period is it.
a*
8. (b) fix + 1) +7(x + 3) = 2 (>)
a'-} Replacing x by x + 2, we get
ox-l + l fix + 3) +fix + 5) = 2 (ii)
-J f(x) Subtracting (ii) rom (i), we get
ax-l
fix+ l)=7(x + 5)
= fl + -L- -jjf(x) or fix) =fix + 4)
ax-l
9. (a) Let7(x) = sgn([x] + [-x])
, 1 + ||F(x) = G(x) '|0; xe/
= lx-l 2) '
[-i; x<s/
G(x) is an even function.
Hence fix) is periodic with period 1.
2. (d) fix) = ar3 -fix- (tan x) sgn x
10. (c) fix) =fi-x) where fix) = xixl + sinlxl + xe\
Since fi-x) —fix)
F(x) = x2 - sin x - xe~x (i)
-ax3 + fix — tan x sgn x=ax3-fix- (tan x) (sgn x)
Also, g(x) = where g(x) = cos x + x2 - x
=> 2(ar-/3)x = O Vxe R G(x) = -(cos x + x2 + x) = -cos x - x2 - x (ii)
=> a = 0and /J = 0 F(x) + G(x) = -sin x - xe~x - cos x - x
[a]2— 5 (a] + 4 = 0and6{a}2-5{a}+ 1 =0 = —(sin x + cos x + x + xe_J[)
([a]- 1)([«] -4) = 0 and (3{x) - 1) (2{x) -1) = 0 11. (a) P(x) is an even function.
[a] = 1,4 and (a) = 1/3,1/2 So, it is symmetrical about y-axis.
i 1 i 1 A 1 A 1
o=l+ -,11 + —,4 + -,4 + —
P(-l) = P(l) = 1 and P(-2) = P(2) = -5
2 3 3 2
35
Sum of values of a = —
3
3. (c) /(x) = sin2x + (1 - sin2x)2 + 2
= 3 - sin2x + sin4x
= 3 - sin2x cos2x
n
. -2

sin22x Graph of P(x) showing minimum number of


= 3-
4 distinct real zeroes
=> T. = -, and T, = -
2 2 2
Multiple Correct Answers Type
4. (d) fi-5) = fi3) = 2,/(20) = 7(4) = 3,fi-10) = f(-2) = 7(2) = 1,
7(17)=7(l) = -2 12. (a, b)
tan’1 tan{fi-5) +7(20) + cos"1 (fi-10)) +7(17)} . g(x) = sin(sin_| J{x}) + cos (sin"’VW)-l
= tan-1 tan (2+ 3 + 0-2) = 3-x
5. (b) Since the function is periodic, a2 - 3 and b2 + 7 should be perfect = + COS(COS~l71-W) - 1
squares, which is possible only if a = 2, b = 3 in which casefix) =7w+JiTw-i
= sin x + cos 4x, which has period 2zr. Ifxe / then {x} =0
6. (d) {x} = x — [x] which is periodic with period 1. then g(x) = 0
Now, fix) = sin(3x - [3x]) = sin({3x}) Ifxe /then {-x} = 1 - (x)
{3x} has period | g(-x) = yj\-{x} + y[{x} - 1 = g(x)
g(-x) = g(x)
fix) has also period j.
g(x) is even function.
Solutions S.5

0, xeZ 6. (b)
Also, g(x) = y
g(-x), xel
Thus g(x) is periodic function
13. (b,c,d) X
We have/(x) =_/(2 - x)
Replacing x by/(I + x), we get X
0(0, 0) y = 0
/(I +x)=/(l —x)
Hence graph of/x) is symmetrical about the line x = 1. x=3
Also g(x) = 1 + x) = K 1 - x) = g(-x) Graph of y = cot-1 x
g(x) is an even function. Clearly
Further/(1 + x) = -/(! -x)
/(i)=-rd) cor* (x2 - 4x + a) > Wx e R.
/(i) = o
x2 - 4x + a > Vx e R
V3
DPP 1.3 => x2-4x+a+~7=>0Vxe/?
V3

So, discriminant <0=> 16-4 + <0


Single Correct Answer Type
1 1
1. (a) If/(x) is surjective then range of/ (x) must be [ 1,«). 4- a- —t= <0=>a>4--7=
V3 V3
Range of y]lx2-4x + k +1 +10 e (10, «>)
a elC4--71 =-,«> "I
Range of 3X2 - 4x + k + 1 e [0, «>)
D=0 Hence, minimum integral a is 4.
16- 12(jt+ l) = 0 7. (b) /(x) = sgn (x-x4+x7-x8-1)
4-3*-3 = 0
Forxe (0, l);x- 1 <0,x7-x4<0
=> x - x4 + x7 - x8 - 1 < 0
3 Forxe (l,«»);x<x4,x7<x8
2. (b) fx)= (1 +V=x)-(V=x-x) /. x - x4 + x7 - x8 - 1 < 0
= 1 - x, x < 0 Also for x = 1; x - x4 + x7 - x8 - 1 = -1
Then 1 -x£ 1. Thusx-x4 + x7-x8- 1 cOforallxe /T
Hence function is injective and surjective. Hence sgn (x-x4 + x7-x8-l) = -lVxe FC
3. (a) x, y g A and/(x) =fiy) =>ffix)) = fifty)). So,/is one-one Therefore fix) is many-one and into
Also a e A =>/Xa)) = a = a where b =fa) e A. So,/ 8. (d) I. /(x) = xandg(x) = -xor/(x)=xandg(x) = -x3
is onto. II. fix) = x and g (x) = x3
4. (a) Clearly fix) -> oo when x —> °° III. fix) = sin x which is odd but not one-one or fix) = x2 sin x
and fix) —> when x —»-°° which is odd but many one
Range is R asfx) is continuous. 9. (c)
Also for x > O./Xx) = Ixl - sin x > 0 ,sgnx + ex‘
fx) is increasing (a) fix)= e-
fx) is one-one. when X=0 then/(0) = 2
2
5.(c) when x>0 then/(x)= e + ex
'y = 2x + a2 when x<0 then fix) = - + ex

/ 0 ~2 (0,1 + e)
ax ,
y= + 10
"(0,2)
Clearly from graph ^>(o,i+-
we must have
V e
2(2)+ <?<« + 10 ■x
~6 (0, o’)
(a-3)(a+2)£0
a 6 [-2,3]
Also a>0 Hence,/(x) is many-one
«*:(0,3]
S.6 Calculus

(b) We have/(x) = e*2*1*', x e [-1


clearly /(-I) = e2 =fi\)
. °°) DPP 1.4
also x2 + Ixl > 0 V x e f-1
^=(1,00) Single Correct Answer Type
f (x) is many-one into function.
4 1
(c) /(x) = lx- ll + lx-2l + Lx-3l + Lr-4l,xe [3,4] 1. (d) We have 4cos.4 x-2cos2x--cos4x-x.7
_flx) = (x-l) + (x-2) + (x-3)-(x-4)
= 4 cos4 x - 2(2 cos2 x - 1) - - (2 cos2 2x - 1) -x7
= (3x-6)-(x-4) 2 1
fix) = 2x - 2, which is increasing function = 4 cos4 x-4 cos2x + 2 - (2 cos2 x- I)2 + — -x7
*,= [4,6] 3
--x,7
Clearly,/(x) is one-one onto function. .2
1
|7
(d) fix) = Vln(cos(sinx)) 3 7
—x
.2
For domain, In(cos(sin x)) £ 0
(cos(sinx)) > 1 g(gM) ^|-(g(*))7 3 3 ,1
— x.7 =x
=> cos(smx) = 1; 2 .2
sin x = 0
Hence g(g( 100)) = 100
=> x = nff. m e /
/?z= (0) 3 = 1f x + —1
2.(c) /(x) = x2 + x + —
since /(x) = 0 4 k 2 1 2 2
Thus.yfx) is many-one function. g(fix)) = !fix)2 + afix)+\
forg(/(x)) = 0,
Comprehension Type
a=- <-2
10. (c),ll.(a),12.(a) /W.
/(x) = (x-l)--2
If a > -2, g(fix)) = 0 has no solutions
a = l,b = -2
1
2
9 3. (a) 1 <log2(x2 + 3x-2)<3
Now, g(x)=x2 + x-2 = x + - => 2^(x2 + 3x-2)S8
2. 4
=> -5 < x < - 4 and 1 < x < 2
f | if
g(W)= |M+-J - 4
9
rx
4. (b) f(g(x)) = g(fix)) = rx.
1 + (r - l)x ’
Smin = «<°)=-2
If /(^W) = g(fix))
Now. /: [I,")-> [-2,oo), rx
=> \ + (r-\)x~rX
=> [-2, oo)->[l,<»);

=>
fW=y
x2-2x-(l+y) = 0
4——1 l + (r-l)x
=0

If this is to be true for infinitely many (all) x, then r = 0 or r-1


2 ± 74 + 4(1 + ^)
=0
X 2
ax + b
a +b .
cx + d
5. (a) fofix) = - ---- = x
So, ax + b
c +d
cx + d
/-‘(x)= 1 + 42 + x (ac + dc)x2 + (be+ 41-be- a2)x -ab-bd = 0
Graph ofy=/(lxl) It is true for all real x,
ac + de = 0, a2 - r/2 = 0, ab + bd = 0
y
so a = -d

O
x 6-(a) r*or1(23) = (gO/)->(23)
A Now gofix) = 2(x3 + 3) + 1
y=k
For 2(x3 + 3) + l=23
x=2
From graph/(lx!) = k has four roots if k e (—2, -1). «of‘(23) = 2
Solutions S.7

7.(c) {xe Rffa) =fa)} y = 1 —x


= (xe R:J(f(x)) = x} 4-
fax)) =fa(x - 1)) = [x(x - 1 )] [x(x - 1) - 1 ] 3-
= x(x-l)[x2-x-l]
2-
Now, ffa)) = x
=> x(x- l)(x2-x-l) = x
x(x3 - 2x2) = 0
-4— 4- 4-
=> x = 2(vxe [loo)) 1 2 3

8.(a) y = 2\ogax
-2-- \ y= \ -x
’ogZ=f y = -(x-i)2
-3--
x = a>’n

=> fa(y) = ayr2


1>+C b c
fl(b + c)=a1 =a2-a2=f\b)fi(c)
.2
DPP 1.5
9. (c) /(g(x)) =
Single Correct Answer Type
fafa)) = 2 => x2 + 1 = 2 and 2X2 + 1 = 2 .a
1 = 2f(x)f(-]vx,yeR
l.(b) Let /[ x + -| + fl x--] i
=> x = ±l,x = ±-U
V2 {yj\ y y
No. of solutions = 2 Given J(0) = 0
10. (d) f(g (x)) = a(bx + a) + b „ . n 1
Putting x = 0, y = —, we get
x
= abx + a2 + b (i)
fa) +fax) = 2/(0y(x)
g(f (x)) = b(ax + b) + a fa) +fax) = 0
= abx + b2 + a (ii) fa) =-fax)
From (i) - (ii), we get Putting x = l,y = -l, we get
f(gW-g(f20)) = a2-b2 + b-a /(2)+/(0) = 2(/(i)]2
(a2 - b2) + (b - a) = 28 /(2) = 2[/(l)]2
(a - b)(a + b - 1) = 28 = 1 x 28 or 2 x 14 or 4 x 7 Putting x = -1. y = -1, we get
If a - b = 1 and a + b - 1 = 28
=> fa2) = 2fa\fa\)
-/(2) = 2(AD)2
Then a = 15; b = 14
/(2) = -/(2)
If a - b = 2 and a + b - 1 = 14 (not possible)
/(2) = 0
If a - b = 4 and a + b - 1 = 7 . /U) = 0
Then a = 6 and b = 2 /(l)=/(2) = 0
11. (d) Given g(x) = 2 fa) + 5 /(l)+/(2) = 0
Replacing x by g"*(x), we get 2. (c) x2-6x + 6 = x2-4x + 4=^x=l
£«‘(x)) = 2/(g-‘(x)) + 5 Put x = 1 in the given relation to get
7(1) =1
=> x = 2fg~fa)) + 5 for x2 - 6x + 6 = 1, x = 1 or 5
x—5 for x2 - 4x + 4 = l,x = 1 or 3
fa'\x))=^ Put x = 3 and 5 in the given relation to get
x-5^ /(-3)+/(1) = 6 =>/(-3) = 5
g~‘(x) = f and 7(l)+/(9)= 10=>X9) = 9
2
/(-3)+J(9)-5/(l) = 9
-X + l, X>1 3. (c) We have,
12. (d) /’*(x) = i— the solutions of
1 + V-x, x<0 fa ~fa)) =ffa)) + -fa) +fa) ~ 1 (1)

fa)-ffa) = 0 arex = -l, 1, 0, 2. Put x =fa) = 0


S.8 Calculus
Then/0) =/0) + 0+/(0) - 1
Multiple Correct Answers Type
X0)=l
Putting/)’) = x 10. (a, b, c)
A0)=Ax) + x2+/(x)-] (x - ytffx + y) - (x + y)ffx - y) = 2y(x - y)(x + y)
x2 Let x - y = w; x + y = v. Then
Hence,/x) = 1 - — uflv) - vfl.n) = wv(v - w)
4. (a) Let/0) = k. Then/x) =/x + 0) =/0) = k Z^_/Of) =v_u
------- = V - w
3 V u
So,/is a constant function. But f\ - | =

• •
„ , = (3}
/(x)
\”7 I
k4/
- all
^J for
4 I
\4/ 4
■■ x and hence /^J = 3
4
=>
—H
r/(y)
V v
^^-x = A.Then
. w
= constant

Let
5. (c) Taking x = y = 1. we get x
/d)Al)-/l) = 2 /(x) = (Ax + x2)
/2(l)-/l)-2 = 0 Since /I) = 2 then A=1
(Al)-2)(AD+l) = 0 /(x) = x2 + x
/1) = 2 (as/(l)>0) 11. (a, c)
Takingy = 1, we get flx + y)-kxy=f(x) + 2yi (1)
/x)J(l)-/(x) = x+ 1 Put y = -x
/x) = x+ 1 => /0) + £x2=/x) + 2x2 (2)
r’(x) = x-l /(x) = (£-2)x2+/(0)
Putx = 1
/x)Jf'(x) = x2-l
/(l) = (£-2)+y(0)
6. (b) (Ax>’))2 = x(A.v))2andy(2) = 6
£-2+/0) = 2
Put x = 25 and y = 2
£+/0) = 4 (3)
(A50))2 = 25(/(2))2 = 25x36 Putx = 1
/50) = 30 )(2) = (£ - 2) • 4+/(0)
7. (b) /(x+/x)) = 4/x)and)(l) = 4 4£ +/(0) = 16
Putx= 1, then Solving we get 3k = 12 or k = 4 and/(0) = 0
/1+/1)) = 4AD /x) = 2x2 .
X5)=16 12. (a, b)
Put x = 5, then /(1000)/(f(1000))=l
/(5+/(5)) = 4rt5) /(1000)/(999)= 1
/(21) = 64 fix) 999/(999) = 1
8.(c) g(x)e [-2,-1) Vxe R 0 ■x /(999) = —
/(x) = -3x2-£x-12 999
=* /(0) = -12
999 are in the range off.
The numbers 999 and----
A5(x))>0Vxg R V12
Hence, by intermediate value property of continuous function
/(-2)>0and/(-l)>0
function takes all values between 999 and —, then there exist
/(-2)>0=*-12 + 2£-12>0=*£> 12 =*£>15 999
/(-l)>0=*-3 + £-12>0=*£> 15
a€ 999J such that/(a) = 500
9. (c) Given that/(/(n)) = n and (i)
/0)=l Then/a)/(/(a)) = 1 =*/(500) =
Put n = 0, we get 1
-^t,999 I, thus/( 199)= —
AA0)) = 0or/(l) = 0 Similarly, 199 G
199 ) J 199
Also given that/(/(n + 2) + 2) = n (ii) But there is nothing to show that 1999 lies in the range off.
Putn = -l,we get Thus (d) is not correct and (c) is also incorrect.
JVU) + 2) = -1 or/(0 + 2) = - 1 or/(2) = -1
13. (a, b, c)
For/(3), put n = -2 in (ii)
(a) -l^Ld-1^4 =>0<lxl^5 =>-5<x<5
A/(0) + 2) = -2
(b) Xlxl+1)
or /(l+2) = -2
/(3) = -2 range of/(Lrl+ 1) is [0,2]
or
Solutions S.9

y 2xfly) + 4>Ax) = 2>fix) + 4xfy)


/(*) . /(>)
-4j-2j 4y-2r
/(x) = k(4x - 21)
0 1 3 f(l) = k(4-2) = 2
k = 1.
(c) range off (-1x1) is [-1,0]
Hence, _/(x) = 4X - 2X.
y
/(4) = 44 - 24 = 240
-I 1 x
/x) = (2X)2 - 2X = (2X - 1/2)2 - 1/4
Thus,/(x) has least value as - 1/4.
-1
17. (b), 18. (a)
(d) -l<lxl<4 =>xe[-4,4] Putting x = y = 0, we get
14. (a, b, c,d) = /(0) + /(0)
/(0)
l-[/(0)]2
Comprehension Type => /(0)[/2(0)+ 1] =0 =>/t0) = 0 (since^fO) *-1).
Now putting y = -x, we get
15. (b), 16. (c)
/(x) + /(-x)
Ax + y) = 2\/(y) + 4>7(x) (i) /(0) =
l-/(x)/(-x)
Interchanging x and y, we get
Xx)+A-x) = 0
ytx + y) = 2^x) + 4"Ay) (ii) fl—x) = -fix) =>fx) is an odd function.
S.10 Calculus

CHAPTER 2

DPP 2.1 6. (c)


(a) lim (x)lo8<,x = (0+)'~ ss oo

So, limit does not exist finitely.


Single Correct Answer Type
_ x2 -9~7x2-6x4-9
1. (c) Given that, (b) lim —
x->3 |x-l|-2
/(x) = [x - 3] 4-lx - 41
x2-9-|x-3|
lim f(x)= lim ([x-3]4-|x-4|) = lim
x-3

= lim([3-A-3] 4-13-h-4|) Now,


/>-+o
= lim ([-/;] 4-1 4- h) (x2-9)-(x-3)
h-tO A3+)= lim = lim ((x4-3)~1)=5
(x-3)
= -14-14-0 = 0 Also,
(x2 - 9) + (x - 3)
[sin x] - [cos x] +1 '0-(- 1)4-1' 7(3-)= lim = lim ((x 4-3) 4-1) = 7
2. (a) lim =0 (x-3)
3 . 3
So, limit does not exist.
[sin x] - [cos x] +1 ~0-0-H'
lim =0 1
x<L 3 . 3 . (c) lim([x]) = lim x-i = 1

[sin x] - [cos x] 4-1


lim =0 7. (d) If (variable)vanable and base is negative then limit does not exist
3 .
And lim (x-6)4=16.
1 8. (c) We must have
3. (c) x < —
3 4 tan (2kx)
<1
1 k
=> —>-3=>--<3=> -- = 2,whenx->
x x lx 3
=> -— <tan~* 2kx< —
4 4
lim------- = (-3)(2) = -6 => -1 <271X< 1
-r x L x.
x-> 3
1 1
xe
. 2k'2k.
lim /(x) = lim [X4- -| = 2_
4. (c)
r-»0" x-»0 \ 2) 2 ex4-e
9. (c) lim x - loge
lim/(x)= lim^2x4-^ = 3 :2
4 14-e~2x>l
= lim x-logeex
lim /(x) does not exist,
x—>0
X-»oo
2 J
(1 «~2x
= lim x - x - loge
.. ' [x]24-15[x] + 56 X—
2
5. (a) L = lim
x-+-7 sin(x 4- 7) sin(x 4- 8)
= Hm (W + 7XW + 8)
= -10gJ|l = 108e2
x-»-7 sin(x 4- 7) sin(x 4- 8) xl/x
.X
e 1
10- (b) lim (ex + nx)1/x = n lim +1 =n
j ([x] + 7)([x]4-8) (exact 0) (1) q
Now, lim >
-7*sin(x4-7)sin(x4-8) (—»0)sin(—1)
Now, {/r} = n-3
([x]4-7)([x] + 8) =(-l)-(exact 0) = 0
lim
x-»-7_ sin(x 4- 7) sin(x 4- 8) (->0)sin(-l) Jim {(ex4- ttx)*} = — 3
L=0
Solutions S.11

cosx 15. (a, c, d)


11. (c) If is periodic then a must be rational.
sin ax
Now, a is rational and n —> «>. Thus, n! na will become an V 1 < x < — , tan_,tan x = x
2
integral multiple of it.
lim (1 + cos2"1 n!na) = 1 + (±l)2/n = 2 and 0 < loge x < log, y- < 1
J(x) = x
Multiple Correct Answers Type V- n I
v — < x < e, tan-1 tan x = x - n
12. (b,c)
and 0 < loge x < I
. sin-1 (sin x) (logexy = 0
A = lim —q--------
x-^ocos (cosx) => f[x) = x-it
.. sin-1(sinx) x , and forx > e, logex >1, (loggX)" -»~
hm —----- -=■ li m — = -l
x-»0“ cos (cosx) x-»0+—X => jV) = 0
lim -8.i."~'<8inX)
x-»0+COS (cosx)
= lim —= 1
x-»0x DPP 2.2
Thus, A does not exist.
Single Correct Answer Type
B = lim = 0 as [Ixl] = 0 when x —> 0.
x-»0 x
.. - cos x.2‘
13. (a, b) 1. (c) _...1
lim —---------------
X-*O 1 - cos X
glx|+(x]_2>
lim x J2sin2(x2/2)
|x| + [x] J = li m------ =---------
x-»o 2sin‘(x/2)
(ex+0-2>l /
= lim x - ------ - sin (x2/2)
x->0+ \ X + 0 ,
x2/2 X2/2 rr
1 rlim
= -7= X —x— = V2
= lim+(ex-2) 2 x2/4
sin (x/2)
= l-2 = -l x/2 7

>i+w_2Y 2. (c) lim (1 - sin x)tan x


lim x x-tx/2
|x| + [x] J
1 — sin x
e -1-2> = lim sin x
= lim x =0 x-»ff/2 COS X
x-»0” -x-! ?
(l-sin2x)
= lim
14. (a,d) x-»x/2 cos x(l + sin x)
cos2x
02 + 0-2</(0) 02 + 20-l = lim ------------------
x-*xi2 cos x(l + sin x)
6+3 e2 0+3 cos X
= lim ------------= 0
Putting 0 = -l, we get x-»x/2 (1 + sin x)
1-1-2 1-2-1
</H)< , 1
2 2 3. (c) lim x2 1 —cos—
X
=>-l <y(-l)<-l => - =-
x d i
= lim x2 x 2 sin2
e2+e-2 02+20 — i
Also, lim = -l= lim
0+3 0+3
2x2xsin
Using sandwich theorem = lim
1 „
—vx4x
4x2
u—y >—|

2
S.12 Calculus

4. (d) lim [^(x + 1)2 - \/(x—1) .. 1 3 cos x - (3 - 4sin2 x) cos 3x


= lim — - —
x-»ocos lx cos 3x 4xz
= Jim xl/3{(x + l)l/3 + (x - l)l/3} {(x +1)1/3 - (x - l)l/3} 3(cos x - cos 3x) + 4 sin2 x
= lim--------------------- -------------------
x-»o 4x
_ x,/3{(x+D,,3 + (x-1)'/3}2
*^m ----------- ---------- —■ —
.. 6 sin x sin lx + 4 sin2x
*-»- {(x + l)-/3 + (X2 - l)i/3 + (x -1)2/3} = lim----------------------
x->o 4xz
1/3 I/3)
2jt2/3 I + 1 +1-1 | 3 sin x 2 sin lx sin2x
= lim------------------ +
\ x \ X 4 *-♦0^2 xX lx
= lim----- x2
2/3 1/3 2/3
J'"x2/3 3
1+1 + [1-4 +1-1 = 3+1=4
X \ x . \ x
^2p' = si°’>
C .. 3-2"+,-4-5"+l
5. (d) hm------------------ y = tan x
5-2" + 7-5"

.. 62"-20-5"
9. (b) —t- + + +-
= hm---------------- -2 1 2
5-2"+7-5"
6 (2/5)"-20
= hm---------------- '' -nil-
5-(2/5)"+7
0-20 -20
z-----------=--------
0+7 7 sin"‘x
L= lim
*-»oLtan“'x.
sin(x - 2) sin"1 x
6- (c) lim {x}
(x-2)2
= lim X
tan”’x
= lim sin(* ~2)
. x
x-+2* (x - 2) (x - 2)
l(but>l)
= lim ±±1= hm =1 1 (but < 1)
x-»2+ X-l x-+2+X-2
=1
lim (Jx + 1 - -Tx) = lim 1
7. (a) r=° J1 +Jsin 2x
x 10. (d) lim —1-Vsin2x
v xA- =
=> cor'(O) = nil
ir n-4x Jl + Jsin 2x

iY
( II - sin 2x .. 1
2x + l 2+- = lim x lim . =
Also, lim = lim x = co
n (n - 4x) X~* 4 V + VS^n
J->4
1--
\ XJ n
sec-,(«>) = nil sin2 — x
= lim^
Thus, required limit is 1. ------- --1
n n-4x
x-»—
4
3 tan 3x - 4 tan 2x - tan x
8. (d) lim • (—n x
*-»o 4x tan x sin
14
= lim
K
3[tan 3x - tan 2x] - (tan 2x + tan x) x-t—
A
4---- x
K
4
x —»o 4x2 tan x I4
3sinx sin 3x
which gives RHL = -- at x = - and LHL = 1 atx= -
cos 2x cos 3x cos x cos lx 4 4 4 4
= lim
x-»0 4xztan x So, limit does not exist.
3 3-4 sin2 x
11. (d) £= lim(e^-e<?+,>)
cos 2x cos 3x cos x cos 2x x-><»
= lim
x-»0 4x.2:
cosx = lim e?+l[e^**-<?+,)-i]
Solutions S.13

Now, Um [,/x4 +1 — (x2 +1)] -lim(x + ,)20l° + (x+2)2oi9 + ...+(x + 1O)2oi°


,006 + 1)(2x,00* + 1)
x4 + l-(x4 + l + 2x2)
= lim 2010 / ?\2010 z loyolo
/x4 + l+(x2 + l)
1+1 + 1+- +...+ 1 + —
X V Xy \ XJ
-2x2_______ = lim
= lim — 1
x-»~ 2 ^l + (l/x4) + l + (l/x2) 1 + POO6 2 + x10O4
X

= -l = —= 5
2
L = °° x | — — 1 15.(c) lim/*1-*”5/
Ie g(x)sin x
= —o©
x2
f\ 2sin2- j • 2X1
2sin —
12. (c) We have, iim tan3 x-tan x = lim
I 2) X
2J
x-»*/4 COS (x + 7t/4) x-»0
. J _ . 2 x |
g(x)| 2sin2^ 41 sin2— cos 2 -
X

.. tanx(tanx-l)(tanx+l) 'A 2J I 2 2
= lim ------------------------------
x-tx/4 COS (x + 7t/4) i- a
2*
= lim----- xtan
tan x(sin x - cos x) (tan x + 1) *-*og(x) 2
= lim / \2
x-nr/4 cos x cos (x + tc/4) I x 1
tan 2£
tan x(cos x - sin x)(tan x +1) 2
= - lim = a lim x —_2 \2
X—>X/4 cosx cos (x + n/4) x^O g(x) X 1

( 1 1 . V 2)
tanxl -yjcosx--/=sinxl (tanx + 1)
= -■72 lim x2
= a lim-------
x-»x/2 cos x cos (x + n/4) -r-»0 4g(x)
tanx(tanx + 1)
= -v2 lim ------ --------- — x2 4b
x-tx/4 cos x lim----- =
x-+og(x) a
— — ^[2 x 2 x \^2 = — 8
Now, iim g(l - cos 2x)
13.(d) lim 0->i°*)(fr>-*J)co8x •
x->0 ?
„£ (x-2x)4
g(2 sin2x)
= lim------ ------
x-»0 j4
1 -cos( —-X (2x - ?r)(4x2 + n2 + 2nx) sin (y - x
= lim
u
/r f \4 x->o(2sin2x)2 x4
X”*2 .A n x1
16 —
12 J a
=—x 4=—
t a
.. (1 - cos/i)(-2A)sin h 4b b
= lim------------------------ lim (4x2 + it1 + 2?rx)
h->o 16/j4 X
*"*2
16. (b) Um g(rtO) = ^0+))=g(l*) lim —— = 1+
.. (1 - cosA)(-2/i)sinA x—»Of x-»osin x
= Iim-------------- ----------- lim (4x2 + n2 + 2nx)
*->o i6/i4 = 1 -2(l)-2 = -3
x~*2

' / x2
lim g(/U)) = g(fl(T)) = g(2+) /v lim (2 - x) = 2+
(1 -cos/i) 41 — 1 ,+ it,22 + 2n x-»0- I x->0”
= - lim
8/i2 U)
\ = 2-5 = -3
limg(/(x)) = -3
2 sin2 — x-»0
= -(3^2)lim 2
A-»0 2n
kn
32—
4
17. (b) lim I cos—I cos—
6
I =o
3n2 holds good if
16
10 Case I: cos — = cos — = 1
4 6
Z(x + r)2010
14-(a) lim—: i.e., — = 2mn and = 2ptt, m,pe Z
x-»~(x'■l006 + l)(2x1004 + 1) 4 6
S.14 Calculus

i.e., k = 8/n and k = 12 p logf(l-2x2)


i.e., k is divisible by both 8 and 12 i.e., k is divisible by 24. l-cos(sin2x) sin" i
3. (d) lim
Case II:-1 < cos —, cos — <1 x-»0+
4 6
i.e., k is not divisible by 4 and k is not divisible by 6. 1 - cos (sin2x) sin4x =1x0=0
As lim X—
Case III: cos — = -l=cos — sih\ 2
4 6
— =(2m+l)^and— = (2p+l)w ln(l-2x2)
4 6 (-2x2)
and lim -2x2 ---------= -2
k = 4(2m +1) and k = 6(2p + 1) ( sin2x
This is not possible. •x2
18.(a) We have a„ + 72d„ = (2 + 72)"
In (I—2x2)
an-726n = (2-72)"
So, lim 1-cos (sin2x)
sin2x .
= oo
Solving we get a„ = 1[(2 + 71)" + (2 - 71)"] x->0+

[(2+ 72)"-(2-72)"]
and 4'=—IT2— 4. (b) lim -V 1 - cos3x loge(l +4x)
x->0 xZ sin-1 (ex -1) tan-1 (2x)
a, _ ^[(2 + j2r+(2-j2)"]
[(2++ 72)"-(2-75)"]
'[(2 V2)"-(2->/2)"] 1 - cos 3x loge(l + 4x)
X X
f r~ \n 1 = lim
1+
[ 2-75 x-*0 sin'^e*-1) tan~*(2x)
(.2 + 75 J x x
(2-J2V 1 - cos 3x lim'°fc(1 + 4x)
1- lim----------
x-»o x x-»0 X

lim----------------
Hence, Jim |^j = 75 (— | = 75 (.2-75 x->0 x x-»0 x
1-0 1’2 + 72 ? 0 4
= -4
1 2

DPP 2.3 5. (c) Since y =/(x) is passing through the point (3, l),/(3) = 1.
Also, y =j(x) is continuous.

E lim/(x) = l

limloge(3/(x)-2)
1
tan xs log (1 + 5x) 2(1-/(x))
1. (b) L = lim loge(l + 3(/(x)-l))
o+ (tan-17x)2 (e3iix -1) = lim
-2(/(x)-l)
log(l + 5x).
tan(x5) | 3
Vx 5x
= lim I ,x5. 1 \ 2
x’ . tan'1 x J i
.3x’ 1
6. (a) lim /(x) + log 1 — - log(/(x)) = 0
k 3x5 y x-»0 e/(x)

-1 1 1Qgg 5_5
— 1’1’--------- . — — —
1 3 3 => lim /(x) + log .
fe^-1^ -log(/(x)) =0
x-»0 e/(x)
(x3+27)log,(x-2)
2. (a) lim
(x2-9)
lim /(x) + log -/(x) =0
.. (x + 3)(x2 + 9-3x)logP(l + (x-3))
= 11 m------- ----------------------------------------
x-»0
< /to ,
x-»3 (x + 3) (x - 3) /
%^-l
= lim(x2 + 9-6x) log lim =0
x-»0 < /(x)
= 18-9 = 9
Solutions S.15

1/x l.m^
lim =1 Now. lim 14-2122
x->0
< /W > x-»0
= ex-Ox =e2
X .

=> lim /(x) = 0 10. (d) L = lim [14- (cos x)"” I2


x-»0
x-» —
2
=> AO) = o
Lety= lim (cosx)CO5X
I-------- X
2
7. (c) lim x2 sin log,<e cos— (Form 0 x oo)
X—>oo \
xj log(y) = lim (cos x) log cos x

I--------- X
x2sin loge cos— z
_x) 1 n
l0gw= Bro (oo/oo)
= lim ——— Ioge. cos— sec (x)
1 I I V xj 2
logeJcos- v
V XJ Using L’Hospital’s rule, we get

[ K
1 sin x
= lim x2.\oge cos— log(y) = lira --------- X
X—>«» X
COS X sec x tan x
2
.. X2
n ( n => l°g(j)= lira (-cosx) = 0
= lim —loge cos—
x-»~ 2 x x-*2

y = e° = 1
i I K ,
10ge 1+ cos---- 1 L = (l + I)2 = 22= 4
x2 \ x -f -2sin2 —
= lim — f 1X"
x—><» 2 f n I 2x a-1 + b"
cos-- 1 11. (a) L- lim
< X )
< a >
1
-2 sin2 — Let - = X,
x2 ______ 2x n
= lim —
x—2
(tf L = lim
x->ol
a-\ + bx
a
X
I

a-l+b* 1
4 lim
a
= e'’

8. (d) lim |— =4
x->«> x-c =e
I
= e;tog'6 = ba
4
x+c-x+c n
hm 12. (a) /(x) = lim | cos-^= j
e*—LI 4 yn )
g2<? _ 4
lim ^cos-
2c = log 4 => c = log 2
TT1) ■n
e^ = 2
sin2 —~
Vx 2jn
/(x)~
9.(b) lim l + x + ^
2 lim
= e3 4 x2
x-»0 x 4-Vn .
=e
lim^l+x*1

lime = e,3
x-»0
-lx2
/(X)-1 e 2 -1
lim = lim =0
=> lime = e',3 x->0 X x—»0 X
x->0
r /(■*) o log(e*~ + 2y[x)
hmj£-V- = 2 13. (d) lim
x-»o x2 x-*0 tanyfx
S.16 Calculus

t. log (t’x2 + 2x/x) X + sin-L + .,. + $in—


11 + 1 sin- • x X
= hm---- 1—7=------- ll 22 22______ 2\/
■ + sin-sin
2 22

x->o tan Vx r-
x = lim e x
x-+0

log (f?*2 + 14x j sin- sin


__ 2+_
= lim = lim e'
x-»0 4x x-»0
_i_
= lim log(ex2 + 2Vrp* 2",
= e'

>2+2VT-iU 1/2
lim
) , J \
= loge'-*0 I--
lim f(n) = e 2 = e
n->«

lim +2
= logeI-,° 17. (a) lim (1-x + x-\/e)n
(1-form)
=2
lim n(l-x+xel/n -i)
= e"-“
W + X)^
14. (b) a lim hm 1-------- -
x-({ /(a) y = exe^“ l/B

f2(a + x)-f(a) = ex
lim
z-.C xf(a)
=e
hm /2(g-»x)-(/(fl))2

= ex-,° x
|im (/(o > x) - /(o)X/(o + x) + /(o))
dpp 2.4
I
= eI->0 x
Single Correct Answer Type
lim 2/(0)^°+x)-^fl»
_ eJ-*0 x 1
!• (b) lim ^-j— 0r
lim 2/(o)/'(.) —form
= eI-’°
X^-x3 0

= e4
Apply L’Hospital’s rule

1 ■-l+l 1 -l+|
'n +n
f 7/I2 + /7-l>| >3 - -x 7
13* 7_____
15. (b) £ = lim = lim
n 1 ”--IxF
>
J -X5 1
5 3
lim 1 1
= en^-l
=U 7 _ 45

Um '
.1 9?
5 3

2- (d) Applying L’Hospital’s rule,

|jml + 10gx-x --1


.. 1-x
= lim X = hm------------
1->l l-2x + x2 *-»i-2 + 2x x-»i2x(x-l)
lim •

_1_
= lim — =
x-»i2x 2
l+sin-
2.
16. (b) /(n)=limeJ 3.(0 Lei£= lim-~e'4-J
x-»0 l-cos2x
Solutions S.17

r — -1 + x- ex +1
= lim - (-i)V -
x—>0 „ sin2x —
7. (b) L = lim £ _kZL
2—y- •x2 x
X

e?-l x-ex+\
x2
+ lim
x->0 x2
= liray----- (Applying L’Hospital Rule)
1
_1_ ex-1
1 - lim------ (using L' Hospital Rule) - (-i)nl/'(0)
2 x-»o 2x n____
=1 n=i 1
= 11-1’
2L 2.

-ivhn? n=1
4
= -log2
=>
1 - cos 2x 1
lim -- ---------- = 4
-ex + x 1 + (tan x)3
8. (d) L = lim
x-»3ff/4 1 2 cos2 x
llld /(o + 2A2)-/(a-2A2) 1
4. (c) lim l + (tanx)3 0r
*'4o /(a + A3 - A2) - f(a - A3 + A2) = lim ----------------- —form
x-+3s/4 -COS2X 0
u f'(a + 2A2) 4A - f'(a - 2A.22)(-4A)
Applying L’Hospital’s rule
f\a+A3 - A2 )(3 A2 - 2A) - /'(a - A3 + A2)(- 3A2 + 2 A)

_______
4h f'(a + 2h2) + f'(a-2h2) -(tan x)3 x sec2x
= lim L = lim —
*4*03h2 - 2h [/'(a + A3 - A2) + /'(a - A3 + A2)
2 sin 2x
4
4
= lim —-— = -2 \2
A->o3A-2 -1 n)
-sec—
6 4J
1- If -1 Vx , _i Jx
S.(d) hm —j=| a tan------ A tan -1 . 1
->0+ xvx a T = — x2 = —
6 3
a 1 1 b (x-1)"
x' 'alfx C, x 9. (c) We have lim------------------ = -l
1 + ~2 r-»r log cos'" (x-1)
= lim — a 7
x—>0 3 A"
x => lim--------------- = -1
2 A->o/n(log cos A)
___________
a2 b2 1
= lim nh ’■ cos A
x—»0 (a2 + x)x (A2 + x) x 3 hm— =1 (Applying L' Hospital Rule)
A—>0 sinA'j
g2-A2 ~h~ J
3a2b2 nA
lim----- = 1
4r(log(l + 2x)- log(l +3x)+- A-+o m
6. (a) lim ex x n = 2 and m = 2
x->0
(log(l+2*)-log(1+3x)+x
lim-------------- z-------------
= eI-° Comprehension Type
10.(c), ll.(c)
lim-------- «-----------------------
(using expansion) Since lim ^-=U(i)=o
3X2 x-»(x-l)
lim ^-x-
/(X)
=1
5 x->l(x-l)22 x-»12(x-l)
= e2 Z(l) = 0
S.18 Calculus

lim£^ ae° - e° - 1 = 0 or a = 2
= i=>HD = 2
x-1 2 2e2x-ex~l
L=\im—---- -----
Since x = 1 is root of/(x) = 0 and/*(x) = 0, x-»o 2x
/(x) = (x - 1 )2 (ar2 + bx + 2) (v/lO) = 2) 4e2x-l
= lim—----- - I(Using L’Hopital’s Rule)
f(x) = 2(x - 1 )(a? + bx + 2) + (2ar + b)(x - 1 )2 x-+o 2
/(0)=-6=>b = -2 _3
Using/"(1) = 2, wegeta + fe = -l =>a= 1 ~2
/(x) = (x-l)2(x2-2x + 2) x3
=> /(x) = (x-l)4 + (x-l)2 3. (b) lim . ------------ = 1
x-»o yja + x (bx - sin x)
=> /(2) = 1 + 1 = 2
Also, /,(x) = 4(x-l)3 + 2(x-l) =>
x3
lim-j=—-------- = 1
f(2) = 4 + 2 = 6 *-*o >Ja (bx - sin x)
3x2
=> lim-t=----------- : = 1
DPP 2.5 h-^OyJa (b-cosx)
b=\
3x2
l=lim-^^------
Single Correct Answer Type *->0Va (1-cos x)
3x2 6
' A2 = lim
x->0 a
1. (c) L- limsec 4a 2 sin2 —
Jogex “J 2

let x - 1 = t a = 36
Thus a + b = 37
L = lim sec'
A2 £ a/x i r '
/->0 In (/ +1) t
4. (d) Let, L = lim x-logf 0 l/x p
= lim sec'
fA'l2f/-ln (/ + !)? I 1 0 l/x>
»-»o rln(/ + l) I
f
= lim x-logj ^r + P~
x->« (XJ
“'I
f
( f2 xJ
t- /---- + ... For limit to exist, P = 1
= lim sec' A2 k 2
r-»o ( I /ln(r + l)
L = lim x.loge f 1 + —
( x-*» \ x x,

= lim sec' A2 2 3
= limxf4-^
/-»0 I Uln(/ + 1) )) \x x)
X-»«»

a
= lim I — - y I = -y = -5 and a e 7?
*-»~\x2 J
= lim sec' A2 2 3
f-»0 V lln(/ + l)
Multiple Correct Answers Type
A2>|
= limsec — exists 5. (a, b,c)
t-tO .2)
aex + b cos x + ce
12 _ lim =4
=> 2 1 => I AIS 42 x-+0 e2x-2ex + l
2
x2
Thus, L does not exist for A = 0, ±1. 2
2 x
-“-ex-x => lim-------------------- =4
2.(c) £=lim-
p
x-»0 7-T
x->0 x2
a + b + c = 0,a-c = 0=$a = c
^“-ex-l
= lim----- ------- (Using L’Hospital’s Rule) a-b + c
x->o 2x => =4
:2:
Since denominator -> 0, we must have numerator -> 0 when a-b + c = 8=>2b = -8=»b = -4
x-»0
Solutions S.19
6. (a, b) 1
Let/(x) = [x3] - [x]3 = —— =>6 = 2
6b 12
fla+) = a3 - a3 = 0 a e R,b = 2,c = 0
lim ([(a-A)3] - [a-A]3) = a3- 1 - (a- 1)3 = 3a(a- 1) 9. (b), 10. (c)
A-»0
Si nee J(a~) = 0 => 3a(a -l) = 0=>a = 0ora=l
/(x) = lim

Comprehension Type x2 + ax + l, |x|<l


7. (a), 8. (b) 2x2 + x + b, |x|>l
We have s‘n (s‘n ~ s*n x 1 —a + b + 3
x = -\
*-»o axs + bx3 + c 12 2 ’
a+b+5
In L.H.S. Nr —> 0 when x —> 0 x=l
2~'
Dr must —> 0
=>c = 0
lim f(x) exists if
sin (sin x) - sin x
Now, lim
x-»0 a? + 6x3
lim /(x)= lim /(x)
sin x - x sin x + x x—1” x->-r
2 sin cos
= lim------ 2 ___ 2
x->0 ax5 + bx3 lim (2x2 + x + b) = lim (x2 + ax + l)
x-»-l‘ x->-l+
sin x - x sin x + x
2 sin cos
= lim
F sin x-x 2 2-1+b=1-a + 1
x-»0 sin x - x 2 ax5 + bx3 a+b= 1 (>)
2
lim /(x) exists if
sin x-x'I x-»l
= lim
x-»ol axs + bx3 J
,, COS X -1
lim /(x)= lim /(x)
= lim---- -------- =
•r->0 5<2X4 + 36x2
-sin x => lim (x2 + ax + l) = lim (2x2 + x + 6)
= lim-------------- (Using L’Hospital’s Rule)
*-»0 20ar + 6bx
-sin x 1 => I + a + 1 = 2 + 1 + b
= lim-----------------------
*-*o x 20ax + 6b => a-b = 1 (ii)
Solving Eqs. (i) and (ii), we get a = 1 and b = 0.
= —— for all a e R
6b
S.20 Calculus

CHAPTER 3

(
DPP 3.1 lim
’-(i)
2x-l

Single Correct Answer Type F2x-1


= lim (74 + ^T^T+2)
1+ ^"4J
1. (b) fix) is continuous at x = 0
ablx2 =4
.. f . 2x2 3x
lim sin---- + cos— ! =/(0) Since LH.L. # R.H.L., value of p does not exist.
x->0 a b. 4. (d)
.. [ . 2x2 3x Alai
hm sin— + cos---- 1 .. f(x)
=> e,33 = er-»ol a b ). I? S.(c) h(x)
g(x)
/ _ • :3x' x3-x2-3x-1
. 2x2
sin — 2sin — => h(x) = (i)
2 a 9 ____ 2b (x + l)a
3 = o61im
x—>0 a 2x2 4h2 9x2
a 4d2 > Now,

=> 3 = ab[-~ 9 x3 - x2 - 3x -1
lim
\a 2b2 (x + l)a 2
4b2-9a 2 1
=> 3 = ab\ — = -=>a = 4
2ab2 a 2

4b2-9a x3-x2-3x-1
=> 3= Mx)=
2b 4(x + l)

4b2 -6b-9a = 0 /i(l) = -l/2


Since b is real, we have 6. (b) Since the function is continuous at x = -2, then
I D>0 /(-2)= lim /(x)
36+ 144a >0
=> a >-1/4 3x2 + ax + a + 3
= lim
2. (c) (a) Since g(x) = l/(x)l x2 + x-2
gix) > 0
Range of g R. Hence, g is not onto. this limit will exist if 15 - a = 0
(b) If we take/(x) = x, then/is one-one but l/(x)l = Ixl is not => a - 15 (i)
one-one. 3x2 + 15x + 18
(c) Iffix) is continuous then l/(x)l is also continuous. So, hm f(x) = lim
x2 + x-2
_ /I
3.(d) f - =p (i) 3(x + 2)(x + 3)
= lim---------------------
2
x-»-2 (x + 2)(x-l)
1 - sin nx
limj-------------------------
i -1 + cos(2^x) -lim
x->-2 (X — 1)
2
-;rcos(7rx) A=-i
= hm---------------
(Using L’Hospital’s Rule) -3
r-2^sin(2^x)
Z"*2 Hence,J(-2) = -1.
-;rsin(7rx)
= hm------- ——- (Using L’Hospital’s Rule) 7. (c) lim /(x) = lim 8 * =0 (where h > 0)
h 4n cos(2zrx) A->0
2
lim /(x) = lim a[x] = a(0) = 0
2 x->0T x->0+

4 fix) is continuous at x = 0.
At positive integral points, fix) is not continuous.
Solutions S.21

2 cos x-sin 2x 13. (c) If/is continuous at x = 0. then


, X<-
(*-2x)2 2 (128a + at),/8-2
8. (b) fx) = ■ flu) = lim--------------------
e~cosx-l K x->0 (32 + Z>x)1/5-2
X> —
8x-4/r ’ 2 As x —> 0, the denominator —> 0.
L.H.L. at x = n!2 Thus, for limit to exist the numerator must also —> 0.
2 sin h - sin 2h 2 sin A(1 - cos h) . Thus, we have (128a)1/8 = 2 or a = 2.
lim = lim------------- =------------ 0 Now, we have
A->0 4h2 a-*o 4/i2
esinh -I /(0)=iim<256 + 2^-2 0
R.H.L.= lim x->o (32 + 6x) /J-2 0
h^>o8((n/2) + h)-4k
2
.. esmh-l sinh 1 -(256 + 2x)”7/8
= hm---------------- = — /(0)=lim-^---------------- 5 2~7 5
h-»o 8/i sinh 8 =>
x-’° —(32 + bx)-4'5 4b 2~4 32d
=> /i(x) has irremovable discontinuity at x = k/2.
9. (a) /(5")= lim sin^y(x-[x])^
=> b= —
32/(0)
= lim sin^y(x- 4)^ = 1
rr , a 64
Hence, we have — = — /(0)
b 5
ab2 |x2-1 lx + 24|
/(5+)= lim i (i x
x-3 1 - cos 1 - cos-
14. (c) /(x) = l 2 , x * 0 is continuous at x = 0
-g/>2(x-3)(x-8) 2”xn
= lim = 3ab2
7^3 1, x=0
=> 5(/>-l)= 1 =3ab2 \2
25
6 ,a =-----
lb = — I-cos 1 — cos— 1 - COS- I
=> V 2. 2) = 1
5 108 Hence, lim z \2
x-+0
I 1 - COS - I
2nxn
10. (c) (i)/x)=j1’. x<0
k 2J
-1, x>0
2
1, x< 0 1 —cos— 1 - cos Q
(ii) = lim _____ 2 =2 Using lim
-1, x>0 x->0 2mxn 0—»0 e2 2.
-1, x<0
(iii)/to = 4 sin4 -
1 *>, x>0 lim------- — = 2
x-o 2'nx"
-1, x<0
(iv) /x) = < 4x4
lb, x> 0 => lim—------- = 2
x-0 442mx"
x>0 1, For limit to exist n = 4.
(v) />) = 1, x<0
-1 x = 0 2®+m 1

-1, x>0 28 + m = 2
m=-7
(vi) fix) = • -1, x<0 => nt + n ~ -3
1 x=0

Multiple Correct Answers Type


11. (b) k = lim (x2 + e2'x)
15. (b,c,d)
k = lim[(2 + A)2 + e* ] (a) Clearly, flx) is continuous at x = 1.
A-»0
(b) g( 1+) = 0, g( 1) = I => g(x) is discontinuous at x = 1
= (4 + e“")-1
= 1/4 (c) h( 1+) = 1 and hiI-) = 0 => /i(x) is discontinuous at x = 1
12. (c) fx) is discontinuous at x = 2, (d) 1+) = 1 and <pi 1") = -1 => <p(x) is discontinuous at x = 1
So,//(x)) may be discontinuous when 1 + x = 2 (when 0 < x 16. (c, d)
< 2) or 3 - x = 2 (when 2 < x < 3). Given/is continuous in [a, d] (i)
x= 1 g is continuous in (h. c] (ii)
Hence flfix)) is discontinuous at x = 1 and x = 2.
flb) = glb) (iii)
S.22 Calculus

fix), xe[a,b) So a+P= 0


Also, p= 1 => a = -l
Also, /i(x) = < fib) = gib), x = b
=> cr + /? = 2
Six), xe(b,c]
2
x is irrational
From (i) and (ii). we can conclude that /i(x) is continuous in 2. (c) fx) = •1+x2’ has exactly two points of
[fl. b) u (b, c]. b, x is rational
Also,/(b-) =fb)-, gib*) = gib)
h(b~) =fb~) =fb) = g(b) = gib*) = hib*) 2
continuity then equation = b must have two distinct
Obviously, gib~) and/(b+) are undefined. 1 + x2
hib-) =fb~) =fb) = gib) = gib*) roots.
and hib*) = gib*) = gib)=fb)=fb~)
2
Hence, h(b~) = /»(b+) =fb) = gib) Now e (0, 2]
Thus h(x) has removable discontinuity at x = b. 1 + x2
17. (a, d) be (0, 2]
\lh
ah + bh3 cos^^-(o-b) . 7th
lim /(0 + /i)= lim 1 + sin —
h-*0 2a _
3. (b) fa~) = lim = lim = 0;
| In 1+ ah-rbh* A—>0 h h—>0 h
~hr~
= lim eh sin h 7t 7th
A—>0 /(a+)= lim tan - + — =0;
For limit to exist, we must have
A->oV 2 J L2
\_2 2J

ah + bh3 Hencey(a+) =fa~).


lim =0 Thus,/(x) has removable discontinuity atx = a.
h-»0 h2
4. (c) When x e I,
’--0 .. a + bh [cot x] = cot x
hm--------
h-^o h => tan x [cot x] = 1
fl = 0 [tan x [cot x]] = 1
So, we have Whenxg /,
lim /(0 + /i)= lim(l + b/0,M [cot x] < cot x
h-fO h—»0 0 < tan x [cot x] < 1
= lim((l+Z>b)l/6*)6 = eb [tan x [cot x] ] = 0
A-»0
1, cot X € I
For./(x) to be continuous at x = 0, we must have => fx) = [tan x [cot x]] =
0, cotxg/
Hm fix) = fi0)
x-»0* So,/(x) is discontinuous when cot x 6 I
=> ^=3 XI 71 n
Now —<x< —
b = log,3 12 2
=> 0<cotx<2+ 73
DPP 3.2 Hence, number of points of discontinuity are x = cor'3, cof'2
and cot-11 = —.
4
Single Correct Answer Type Thus three points of discontinuity.
rational if x e Q in [a, b]
5. (a) fx) =
1. (b) lim fix) = irrational if x e Q in [a, b]
x->o 3
Letce Q
x-acotx + P 1 fc) = irrational
hm--------- =------- = —
x-»o x2 3 lim fix) = lim /(c + h) = rational or irrational
x—>c h-tO
xa + Btanx 1
hm —j-2------- = - fix) is discontinuous for x = c, hence for all x in [a, b]
x->o x -tanx 3
6. (a) fix) = [x] (sin kxf>
X
a( X’ (sin kxf is continuous function V x e R,ke R and p > 0. (x]
ax + ft x+ — + .. .oo is discontinuous at x 6 /
3 1
lim For k = nn, n e I
x-»0 tanx 3
x3 fx) = [x] (sin in7tx))P
x
lim fix) = Q,ae I
x—>a
(a + p)x + and fa) = 0
1_
lim------------- So.^x) becomes continuous for all x e R
x-»0 3
Solutions S.23

-5 -3 Replacing x by x/2, we have fix) - fl — 1 = X


7. (d) Clearly, [2x - 1] is discontinuous at three points x =—,—
2 2 \2J 2
and —2
-5 -3 Replacing x by — repeatedly and adding, we get
fix) may be continuous ifg(x) = ax3 +x2+ 1 = 0atx= 2
or-2
g(x) can be zero at atleast one point
minimum number of points of discontinuity = 2.
8. (d) Domain of the function is [-1, 1] +-+(/(Fr)
[sin- *x] is discontinuous at x = -sin 1,0, sin 1
[x] is discontinuous at x = 0, 1 X
XXX X
=--- f- —r + —r + ••• +---
Now/(0+) = [0+] - [0+] = 0-0 = 0 2 22
2' 23 2n
/(0-) = [0-]-[0-]=-l-(-I) = 0 XXX X
Hence fix) is continuous at x = 0. => ---- 1---- 1- + —7 + ••• 4
2 22 23 2”
Therefore points of discontinuity are -sin 1, sin 1 and 1.
x
9.(b) lim g(x) = lim < lim
xm f(x) +h(x) + 3 x
lim /(x)-/l — = _JL
*->l+ m—>“ 2xm + 4x + l n—♦«> \ 2" 1-1
2
xw/(x) + A(x) + 3
= lim lim fix)-fiO)=x
m—>l+ 2xm + 4x +1
f x |
A(x) + 3 v /(x) is continuous => lim f — =/(0) = l
/(x) + n—>*» \ 2^ J
= lim lim xm
4x + l /(x)=x+/(0) = x + 1
2 +------- 7(3) = 4
xm
/(I)
Multiple Correct Answers Type
2
xm f(x) +h(x) + 3 14. (b,d)
lim g(x) = lim lim
M—>°° 2x'" + 4x + l '2?-3
lim 3 - cot = 3-[cot '(-“)] = 0
. x2
xCT/(x) + /i(x) + 3
= lim lim
2x'” + 4x + l lim {x2}cos(el/x) = 0 x (value between -1 and 1)
=/>(!) +3
=0
5 15. (a,b, c, d)
g(x) is continuous at x = 1
Z(l) = Z>(l) + 3
2 5
/(l+)= lim
(x[;]+xW
. = lim (x(0) + x(l))
=> 57(1) -2/t(l) = 6
10. (b) Given,/(X) = ([2x] + {2x))([2x] - {2x}) = 4x- 4x[2x} =1
2x e (-4, 4)
Hence fix') is discontinuous when 2x = -3, -2, -1,1,2.3.
At x = 0,/(x) is continuous.
11. (d) /i(x) =7(x) + g(x)
/(!")= lim
Hi?4x1
= lim (x(l) + x(0))

-1, -oo<X<l =1
= ■ a 4- 4 - 2x, 1<x<2
a-b-l + x, 2<x<«> /(2+)= Um x - +*[x]
xj
.*. We must have either a = -3, b 1 or b = 1, a * -3 = lim (x(0) + x(2))
x3
12. (c) fix) =------sin 7tx + 4 =4
8
7M) = ^U(4)=12
=> 7(x) can take value 10 as/(x) is continuous function
/(2') = lim
(X[x] + XtX)
= lim (x(0) + x(l))
x3
— + 4 < 12 and Isin tk! < 1 V x e M, 4]
8 =2
j(x) cannot take the value 18
Obviously 7-v) is discontinuous at all positive integers but at
13. (c) Given 7(2x) -fix) = x,
x = 1 it has removable discontinuity.
S.24 Calculus
16. (a, b,c) —^--1
ax2 + bx + c + enx
/x) = lim 1 + /12
\+cen' /(F) = lim A1??). = lim
h h-i0+ h
ax2 + bx + c + (cx)n -h
lim ; x<0 = lim =0
\ + c(e*)n A->o+1 + h.2
ax2 + bx + c + (ex)n /(I-/>)-/(!) = lim
lim ; x=0 /(F) = lim
\+c(ex)n h->0* -h -h
ax2 + bx + c .. 1 1
+1 = hm —,--- :— = —
(ex)n a-»o+ ^jl-h +1 2
lim x> 0
1 .'. j(x) is not differentiable at x = 1
------ + c
(ex)n . ...... .. t/(/Q-/(0)
3.0)) Let yz (0*) = lim = k (say)
h->0 h
ax.22 + bx + c\ x < 0
1; x=0 ... no-)=
h
]_
x >0 /(0)-/(A) ,
x h->0 h
since/x) is continuous function V x e R Since f'(0+) * f'(0~), but both are finite, we can say that
lim /(x)= lim /(x) = /(0) f(x) is continuous at x = 0 but not differentiable at x = 0.
4. (c) g(x)=/(10-2x)
lim f—)= lim (ax2 + bx + c) = 1 g'(x) =- 2/(10 -2x)
=> g'(2) = -2/(6)
x->0+\cJ x—»0~
Now/x) is differentiable.
-J = 1. =c=>c= 1 Also /(x) < 0 on (- 4,6)
c and/(x)>0 on (6,00)
c= 1,0.be R => /(6) = 0
=> /(2) = 0
5. (a) Clearly fix) is continuous for all real x.
fix) =^~ lx2-II + 211x1-11 + 21x1-7
DPP 3.3 x2-(x2-l) + 2(-x-l)-2x-7, x<-1

x2-(1-x2) + 2(x + 1)-2x-7, -l<x<0


Single Correct Answer Type x2-(1-x2) + 2(1-x) + 2x-7, 0<x<l
x2-(x2-1) + 2(x-1) + 2x-7, x>l
3x2-4>/x + 1 forxcl
1. (C) fix) = -4x-8, x<-l
ax + b for x > 1 2x2-6, -l<x<0
for continuity atx= l,/l+)=/r) 2x2-6, 0<x<l
=> a + b = 3 -4+ 1
4x-8, x>l
=> a+b=0
for differentiability at x = 1, -4, x<-l
r(i+)=r(i-) 4x, -l<x<0
=> o=6-2=4 4x, 0 < x < 1
=> b = -4 4, x>l
[x]+x < i Clearly, fix) is differentiable for all real x.
6.(d) /x) = lx-ll-([x]-[-x])
2. (a) fix) = i
x>l [x] - [-x] is discontinuous atx = 1
w+w2 But g(x) = lx - II is continuous and g(l) = 0
Consider the function fix) in the interval (0,2). Hence fix) is continuous atx = 1
|(l-x)(0-(-l)), xg(0,1)
Vx 0<x<l
Also/x) = 0(1 -(-D), x=l
fix) = 1
l<x<2
l + (x-l)2 (x-l)(l-(-2)), xg(1,2)

fi\) = 1 and lim/(x) = l 1-x, xe(0,l]


3(x-l), xg(1,2)
fix) is continuous al x = 1
Solutions S.25

Clearly fx) is continuous at x = I and non-differentiable at 1 x •


x= 1 as/(!-) = -1 and/(l+) = 3 11. (c) fx)~---- - =----- is not differentiable at x = 0,-1.
1 i , 1
, 14X x+l
3-cosx--^=, |sinx|<-j=-
■ V2 1 1 x
7.(b) /(x) Also. g(x) =
2+ cos x + —U |sinx|>-^ j [ 2(x + l) ~ 3x + 2
1+—
fW X
, I Thus, the points where g(x) is not differentiable are x = 0, - I,
3 - cos x —t= , ■ COS X . > —1t=- -2/3.
V2 n/2
„ 1 12. (c) rr+1 =---------- - ----------
2 + cos x + —}= , |cos x| < r+1 (rr+l){(r +l)x +1}
J2
_ (r + l)x +1 - (rr +1)
1
3-cosx + -t=, ,|COSx|>-y=r
. 1 " (nc + l)[(r + l)x + l]
V2
1 1
„ 1 |C0SX|<-^=r
2 + COSX + -7=, (rx + 1) (r + l)x + l

1 ■^n=X/r+i = ^ ~7
Thus, fix) is discontinuous at Icos xl = -j= or ,=o nx +1
V2
1
7t 37t 5n 7tc fx)=\- x*0
x= —, , , nx + r
4 4 4 4
fO) = 0 and
[x] x e 1
8. (c) /(x) = (where, [•] denotes the greatest integer I
x-1 xe/ lim Sn = lim 1 - =1
n— nx + l
sinx + cosx, x<0
function) and g (x) = . Then for f(gix)) ^0+)=l
atx = 0 1, x> 0
Thus,/(x) is neither continuous nor differentiable at x = 0.
For continuity at x = 0,/(g(0)) =/(!)= 1-1=0 ax(x -!)[ cot— I + (px2 + 2)
/(g(0+))=/(l)= 1-1=0 4 J
lim---------- i----- ------------------
fgiO-)) =Asin(0-)+cos(0-)) =fiQ-) + (!’)) =f 1")=[ 1 -]=0 13. (b) fx) = . 71X
COt---- +1
Thus/(x) is continuous atx = 0 k 4
For differentiability at x = 0, 0, x=l
/-(g(0’))=lim^(h))-^(0)) xe (0, l)u(l,2)
A-»0 h ax(x -1), 0 < x < 1
1- 0-0 A
= hm----- = 0 fx) = 0, x= l
/>->o h
pxz + 2, 1<x<2
/'( g(0-))=limU^
/'(g(0-)) = lim =0 :.fx) must be continuous at x = 1
h->0 -h
L.H.L. = R.H.L = fll)
•*. fx) is differentiable at x = 0. => p + 2 = 0 p = -2
= . 2^1-X2, x<0 a(2x-l), 0<x<!
9.(d) /(x) Also f(x) =
-4x, x>1
0, x>0
Clearly, fix) is discontinuous, hence non-differentiable at v fx) is differentiable at x = 1, so
x = 0. L.H.D. = R.H.D. => a = - 4
Hence la + pl = 6
mJ
y = tanx
2-
Multiple Correct Answers Type
y = cos x y = sin x
14. (a, b)
10. (b) h-*-x
-71/2 o x°ti/2 71 71/2
g(x) =
0,.r = 0
-2- f(x)(0 A
hm ----- 1 - form |
*-»0 x 0
From the graph, there are three points x = x0, — , n of non­ = UmZW
differentiability. “ ^0 1
= /'(0) = 0
S.26 Calculus
Thus g(x) is continuous at x = 0
Comprehension Type
—-g(0)
For Questions 17 and 18
ax2 + b; OSx^l
= lim------ Given two functions: fix) = bx + 2b\ 1 <x£3
/>->o 2
(a - l)x + 2c - 3; 3 < x < 4
zhq)
2 cx + d\ 0<x<2
Similarly and g(x) = ax + 3-c; 2<x<3
g'(o-)= no x2 + d + l; 3 < x < 4
2
15. (b, c) Condition for continuity ofy(x):/(l ) =f 1) =)(1+) and/(3 )=)(3)=/3+)
^/(x)-/^) => a + b = 3b and 5b = 3a + 2c - 6
=a
x-»0 X
=> a = 2b and c = 3 - -
lim/(x)-/(0) + /(0)-(fa)^g 2
x-»0 X condition for continuity of g(x);
/(x)-/(0) /(M-/(0)] = a g(2) = g(2") = g(2+) and g(3") = g(3) = g(3+)
lim
x-*0 x x J => 2c + d = 2a + 3- c and 3a + 3- c=10 + b
3c + d - 2a = 3 and b + c - 3a = -7
lim----------------- \k = a 2ax; 0 < x < 1 c; 0<x<2
x->0 x x-»o kx
Also f(x) = b\ l<x<3 andg'(x) = a; 2<x<3
/(*)-/(Q) - lim /(fa)-/(Q) ■k = a
lira a-1; 3<x<4 2x; 3 < x < 4
x kx
/(x)-/(0) 17. (a) /is differentiable at x = 1 and g(x) is continues at x = 3.
lim - lim /(fr)-/(Q) ■k = a i.e. a = 2b and 2a = b
X kx
Also 3a + 3-c=10 + b
/'(O’)-W) = a => a = /> = 0 and c = -7
/'(0+)-A/'(0+) = a x2 - lax + 49A (1 + a) = 0 has real and distinct roots for
Vae R.
r(l-*)/'(0’) = a 49a2 - 4(49£) (1 + a) > 0 Va e R
(l-Ar)/'(O+) = a o?-4ka-4k>O Vae R
=> 16A2 + 16Jk < 0
1) <0
ke (-1,0)

18. (c) LHL=lim


„..i-------------------= lim-------------------------
*->o|g(2 —/j)I +1 *->o|c(2-A) + J| + l
4b
no) =/-(»-) =no*)= -f-
\-k
12c + d{ +1

16. (C) niIT ,. f(2 + h) t. b(4 + h)


RHL = lim------------- = lim-----------------------
/(x)=xl/3(x-2)2/3 />->0 |g(2 + A)| +1 />-»01(a(2 + h) + 3 - c)| +1
7 I 4b
.-. f(x) = xIZ3 • J (X - 2)-,/3 + (x - 2)2/3 • y r-2/3
|2a + 3-c| +1
1
= — ■2/3 (x-2)"1/3 (3X-2)
v f is differentiable at x = 1 i.e., a = b = 0
3 So, LHL = RHL = 0 v (it is given that limit exists)
.’. f is not defined at x = 0 and at x = 2
Solutions S.27

CHAPTER 4
6. (a) Let x = cos 0,0 = cos-Ix
DPP 4.1
y = cos 2 tan - 2 cos

Single Correct Answer Type 0 . 0


= cos 2 tan tan — - 2 cos sin—
2. . 2.

A-+0 h . 0
= cos0-2| - -sin sin—
.. [7 + A] tan n{l + h) -[7] tan In U 2
= lim----------------------------------------
A-+0 h
= cos 0 —rc + 0
= iim 7 tan + y = x-rr+cos-1x
h-td h
tan nh ^ = 1- 1__
= In lim--------
/>-»o nh dx T17
= 7n

2. (b) From the given functional relations, we can consider 7. (b) y= ?- + Ix7x.22 + 1 +1 log(x + yj(x2 + \))
fix) = sin x and g(x) = cos x
g'(x) = -sinx
g'(0) = 0 => / 1 rr-; 1
y =x+ — Jx +1 +— x
X

3. (b) We have xeV- y = sin2x 2V 2


When x = 0, y = 0
Differentiating w.r.t. x, we get X
Ixe'J' + xxe*>'(y + xy) -y = 2 sin x cos x X 1+
7x2 + 1
dy f x • ye^ + e'i'0'- 2 sin x cos x
~dx~ x2V>-l , (x2 + l + x2) t 1___
2ylx2 + l 2/x2 + 1
=1
dx(0.0)
2x2 + 1 1
4. (c) y=fgh 2y[x2 + l 2y[x2 + l

=fgh +fg'h +fgh'


dx 2x2 + 1 + 1
= x + —,
= L(2fgh + 2fg'h + 2fgh') 2^x2 +1
2
= 1 (A(fg + g'J) + g(fih +jh') +fig'h + gh')) y'=x+ ^x: + l
2
= | [h^gY + g.W'+fW]

(fgh)' (0) = 1 [/i(0) (fg)'(O) + g(0) (/h)'(0) = x2 + Xyjx2 + 1 + 10g(.X + jx2 + 1)


2 +fi0) W(0)]
= 2y
= 1(3x6 + 2x54-1x4) 8. (c) We have g(x) =fix) sin x (1)
2 On differentiating equation (1) w.r.t. x, we get
1 3? g'(x) —fix) cos x +f(x) sin x (2)
= 1(18+10 + 4)= — = 16
2 2 Again differentiating equation (2) w.r.t. x, we get
5. (b) y(x)=/(er)^ g"(r) =/(x)(-sin x) +/(x) cos x +/*(x) cos x
y'(x) =fier) • e^x\f(x) + e^f (e*). t?
+f(x) sin x (3)
= 0 + e°. 2 = 2
S.28 Calculus

g"(~n) = 2f(-n) cos (-n) = 2 x 1 x -1 = -2 1


Hence g'X-n) = -2 — tan x, x<0
2
logf(logt.x) 1 _
9.(d) f(x) —tan x, x>Q
logrx 2
1 x 1 x logf(x) - logf (log/x)) X1 1
x<0
/'(x) = -^ X X
dy 2(1 + x2)’
[logX.r)]2 dx 1__
x>0
--0 2(1+x2)’
f'(e) = ^- = Me 13. (a, d)
h\x) =fix)
10. (c) flx+l) = x+/(x) h"(x)=f'(x)
=> gftx + i) _ ex +flx) = e' x /l(l) = O,/(l)=/'(l) = /l'(l) = /»"(l)=l=g(l)
g(x+ l) = etxg(x) ‘g’ is inverse of
=> ln(g(x+ l))=x + ln(g(x)) fig(x)) = x
g*(x + l) g'( v) = j => f(g(x)). g'(x)=l
g(x + l) g(x) => mm. gz(D=i
/'(i)./(D=i
41 g'd)=i
g '[- + 1
12 , 8 i , G(x) = x2g(x)-x/i(g(x))
G'(x) = 2x g(x) + x2g'(x) - /i(g(x)) - x/i'(g(x)). g'(x)
U ) = 2xg(x) + x2g'(x) -A(g(x)) - x^g'(x)
(v h'(x) =/(x) .-. h'(g(x)) = flg(x)) =x)
g 'f- + 2
12__ ;
g(- + 2
g'H) =1
= 2xg(x) - A(g(x))
G"(x) = 2g(x) + 2xg\x) - h'(g(x)) . g'(x)
= 2g(x) + 2xg'(x) -/(g(x)). g'(x)
= 2g(x) + xg'(x)
G'(l) = 2g(l)-/i(g(l)) = 2g(l)-/i(l) = 2-0 = 2
4 1 1 G"(l) = 2g(l) + g'(l) = 3
g k +2 g' n —
:___ 2 = 1
r r
DPP 4.2
gl”+2 g[n'~2.
A 1
g + Single Correct Answer Type
Adding =n

4" 4) 4) dy
1. (c) ^ = 2x + 2/ —
dx dx
~ dt

11. (b) y = cos-,[x7x - Jl-(Ji)2Jl-x2)


x
Also, t =
= cos-1 X + cos x 1 + x2
dy^ 1 dt 1 - x2
dx .2 2Jx-x2 dx (1 + x2)2
Now putting x = 2, we get
Multiple Correct Answers Type dt -3
dr “25
12. (b, d)
Put x = tan 6 dy = 2(2) + J
df
-22
2 A -3 A 12
2 - — = 4------
488
dx \5) 25 125 125
k n
tan" ’x - 6 e (3 + 2/)
I’I
2. (a) = = Z3
=/
1 + |sec 0| dx dx/dt (3 + 2/)
y = cos-1
2|sec0|

= cos
cos 6 +1
2
Since J(x)
(iJ-*
fix) r3 = 1 +1
dy
dr
= cos-1 (cos 6/2)

J-0/2, -n/2<6<0
=> Xx)=l±£=I
r
[0/2, 0<6<n/2
Solutions S.29

3. (c) y = x3 - 8x + 7 and x =fij) dx -1


Also, (4m)
— = 2 and x = 3 at r = 0 du 71-(2m2-I)2
dt
4m
dy dy/dt 71-4m4-1 + 4m2
dx dx/dt
4m 2
dx dyldt ------- - - —S5 — -
=> 74m2(1-m2)
dt dy/dx
dx = 2 dy ~ — —1
=> —
dt~ 3x2-S dx 2
at t = 0, x = 3

— (atr = O)= —
dt 19
■(a—
(4 3 I i
7. (d) Leiu = sin_1 I—sin 2x +-cos 2x1 = sin sin(a + 2x),

dy Sd9) 8 sec8 0 tan 9 + 8 cos7 9 sin 9 . 3 4


4. (c) where sin a = -, cos a = —
sec 9 tan 9 + sin 9 5 5
i.e., u = a+ 2x.
8 tan 0(sec8 9 + cos8 0)
5 cos x - 4 sin x
tan 0(sec0 + cos 0) Similarly, y = cos
V41
f dy V 64(sec8 9 + cos8 0)2 = COS"‘ COS(J3 + x),
I dr J (sec 9 + cos 0)2
5 4
Here, cos B = —■=, sin /3 = -j=
64[(sec8 9 - cos8 0)2 + 4] V41 V41
[(sec 9 - cos 9)2 + 4] i.e., v = P + x
_64(/ + 4) du
’ (?+4) ^. = ^. = 1 = 2

5. (b) We have,
x2 + 4
</ + 4, ter- dvdv\
dx
8. (b) Taking log on both sides we get
n log (x + y) = log x + log y
x = 2r-lrl,y = /3 + /2lrl Differentiating w.r.t. x, we get
=> x = 3r, y = 0 when t < 0 n , dy 1 1 dv 1 1^:
x = t, y = 2P when t > 0 | + — =- + —-
x + y . dr J x y dx
Eliminating the parameter /, we get
dy _ y x + y - nx
0, x<0 =>
dx x ny-x-y
y= 2x3, x>0

Differentiating w.r.t. x, we get n = 2for — = —


dx x
0, x<0 9. (b) y = 3e2- x
dx 6x2, x>0 •x
x> = -v
• Hence, the function is differentiable everywhere and its fW = *3e2~X
derivative at x = 0 (t = 0) is 0. log(/(x)) = (3e2-x)logx
6. (c) y = tan I “ 1 m x 3e2-x
777 -77-/(-v) =-------- - log x
/(x) x
y = sin" *m Now r(x) = 0
1 3e2 - x = x log x
Also, x = sec => •>
2m2-1 x-e~
100
x = cos
i - *(2m2 - 1)
10. (a) >(x)= n^-O'/(lOl-i)
y' = sin-1 u i=i

dy 1 Taking log both sides, we get


du ‘717? 100
log(/(x)) = 22'*(1O1 -01og(x-j)
= cos-1 (2«2- 1) i=i
S.30 Calculus
Differentiating w.r.t. x, we get d2y d ( dy/dt
dx2 dx I dx/dt J

/(■v) /=1 (a-/)


U/JtZrU/J \dt)dx\dt)
/'(IOD^j.i 101-i
Z(ioi) £ 101 -i
= 5050 O’
f cfrA d f dy\dt f c/y'j d f dx\dt
_\dt Jdt\dt) dx \ dt J dt \ dl J dx
Multiple Correct Answers Type 2

11. (a, d) (S)


2/ d2x
dx 14- 3r2 2 = \dt J dt2 {dt)
3r-4/ + l = 0

12. (b, d)
3. (a) sin x + sin y = 1
.(logx)1^”) cos x + cos y y' = 0
y=x
-sin x + cos y y" + (y') (-sin y) = 0
log y = (log x) (log x)los(io&») (1)
Taking log on both sides, we get „ y' sin y + sin x
=>
log (log y) = log (log x) + log (log x) log (log x) cosy
/ ____ \2
Differentiating w.r.t. x, we get I cosx
sin y 4- sin x
1 J. 1 + 2 log(log x) 1 cos y
log .y y dx xlogx logx x cos y
2 log(log x) +1 sin x cos2 y 4- cos2 x sin y
xlogx cos3y
»
Putting sin x = t, sin y = 1 -1, we get
^=>:.^(2iog(iogx)+i) /-3/2(l-/4-/2)
dx x log x r= (2-/j3/2
Substituting the value of log y from (1), we get 3
a=-
V = (log x),t>e(,og x)(2 log(log x) +1) 2
dx x => 2a= 3
_ f ax 4-
4.(c) y
[ yx 4- 8 J
DPP 4.3 _ a3-py
(.yx+8)2
-2
Single Correct Answer Type y2 = (a8-py) j(r)
_(/x + <5);
1.(0 Wehavej(x2) nx)=r(x) /'(x2) (i) 6
Differentiating w.r.t. x, and y3 = (a8-(fy)
(yx + 5)'
7(r2)
2xT(x2)<(x) ++ 2xf'tf)f(x)
2 2
Putting x= 1, we get >,2 = JW3
2/'(i)r(i)+Ai)m)=ni)-r(i)+2ni)f(i) -d2y
2f(i)r(D+ix8-3r(i)r(i) d 2x
5. (b) We know that ~
-.2
=
r(i)Ho=8 (ii) dy2 8 2
Putting x = 1 in (i)
/d)r(D=r(i)r(i)
rd)=(f(i))2 x4 16 1
6. (c) = x.2 + 3x + 7 +
So from (ii)/(l). (T(l))2 = 8 (x-l)(x-2) x-2 x —1
=> /'(l) = 2and/"(l) = 4 . A A . A 16(—3!) 3!
Third order derivative = 0 + —-—7 +
=> r(i)+rd)=6 (x-2)4 (*-l)4
dy dy/dt -12k = -96
2. (a)
dx dx/dt k=8
Solutions S.31

7. (b) = log x - log (a + bx) And = 2 cos t - 2 cos 2t


x dt
= 2(cos t - cos 2t)
dy
x--y 1 b a dy cos 2t - cos t
dx
x2 x b + ax x(a + bx) dx sin t - sin 2t

ax (sin t - sin 2t)(-2 sin 2t + sin t)


dy
(1)
dx—y- a + bx
x~ d2y - (cos 2t - cos /)(cos / - 2 cos 2l) dt
x—
dx2 (sin t - sin 2t)2 dx
Differentiating again, we get
-2
a' d2y\ = l(l)-(-l)(+2) 1
x------
dx1 (a + bx)2 dx2 Lf (I)2 -2(1)

.3 d2y a2x2 xdy Y 3


X' 2
dx2 (a + bx)2 . dx J
10. (c) We have,
dx_ 1 y>-y = 2x
8. (a) Now —
*■(£) Differentiating both sides w.r.t. x, we get

(1)
<Z2x= 1___1 dx dx 3y~ -1
dy2 dy Again, differentiating both sides w.r.t. x, we get

d2y -2-e3

Ab
dx _____ dx
x—
dy dx2 (3/-I)2
Using (1), we get
d 2y dx d2y -24y
X--- vx (2)
dx2 dy (3/-I)3
Now,
d2x d2y x2 1 V2^ , *dy
dy2 dx~ 27)dx.2? dx
x2 1Y -24yJ + 2x
( V/3 27 J (3y--l)3J (3/-D
dy
dx2 r(r-D; 1 -24y V >(/-!)
fd2jc} 4 27 (3/ -I)3 J 3/-1
(-.'y2-y = 2x)
( \213 {21y2(y2 -1)2 - 4} (-24y) , y(y2 -1)
d y
2
dx2 108 (3y2-!)3 3y2-l
=1+
Sx
y_ -54y2(y2-l)2 + 8 9(y2-l)
< dy1 >
9 (3y2-!)3 3y2-l
2/3

So, R213 A(fy


2/3
Y2yY3
1___
+ Ur2 >
d2yx2'3/r ^x 2/3
_y f-2(l + aXa-2)2 + 8'l
■'
9
a3 >+
3(a-2)
a
where a = 3y2 - 1
\a!r2 .dx2) dx2
=y
1 1 9
,2/3 +
<Z2xf
U2 dy\ Multiple Correct Answers Type
9. (c) We have x = 2 cos t - cos 2t
11. (a, b)
and y = 2 sin t - sin 2t y = e~xcos x
— =-2sin/ + 2sin2r y | = -e_Jcos x - e~x sin x
dt n
cos x —
= -2(sin t - sin 2t) 4
S.32 Calculus
Similarly, we get
y2= (-V2)2 c COS
K 1
X-----
2J
DPP 4.4
y3 = (—*/2)3 e"xcos x-----
3k A
4 )
>4 = (~>/2 )4 f-'cos (x - k) = -4 ercos x
Single Correct Answer Type

=> y4 + 4y = 0 l.(d) /'(0)=lim /(x)-/(0)
x-»0 x-0
A-4 = 4
Differentiating it again four times, we get = lim^
x-»0 X
y8 + 4y4 = 0
y8-16y = 0=>l:8 = -16 = lim^
x—>0 x
>’12 + 4>8 = 0 => J12 + = 0 =>£,, = 64
= lim^
Similarly A16 = -256 x-»o 2x
12. (a, b)
e9 + y cos x = 2 (1)
„g"(0) = 17
Putting x = 0, we get 2 2
e° + y cos x = 2 2. (b) flx + y) =/(x) +/(y) +_flx)/(y), Vx, y 6 R
y=l l+/(x + y) = (l+/(x))(I+Ay))
Differentiating (1) w.r.t. x, we get Put g(x) = 1 +/(x), we get
dy g(x + y) = g(x) g(y)
dy = 0n
x-j- + y - y sin x + cos x ~
dx g(x) = e*
/(x) = eta-l
(xe'y + cosx) — = y(sin x - e9') (2) /'(0) = 1 gives k = 1
dx /(x) = er-l
(A2)] = [e2-1] = 6
dy _ y sin x - ye4’’ 3. (b) Putx=x,y = h
dx xe™ + cos x flx + h) -flx) = Axh + 3X2/!2
dy 1 sin 0- le° |jmZ(x + <|)-/W = Ax
dx (0.1) *-»o h
0+ cos 0
= -l
Putting x = 3, y = 2
Differentiating (2), w.r.t. x, we get /(5)=/(3) + A(6) + 3(3)2(2)2
dy } . dy
dy 48 = 6A+ 108
(xe9’ + cos + e*> + xexy x— + y -sinx —
dx JI dx
Ja!r A = -10
/(x) = -10x
= —(sin x-e9) +y cosx-e,xy
dr
Putting x = 0 and y = 1
4
y+x*J 4. (d) fxy) = fix) fly) -flx)-fly) + 2 (D
/'(X) = lim /0f + />)-/(x)
a-»o h
(O + l)^ + (eoX-l)
dx~ fix 1 + - > -fix)
= (-l)(0-l) +1(1 -1(1)) = lim
I k X/J
-------- (x # 0 given)
^=2 /i-*0 h
dx2 J, h\ _
f 1 + - -2
13.(a, d) = lim \ xj /(*)-!
g(x) = e"+/(x) /i->0 h x
gz(x) = ae"+r(x) x
.-. ’ g'(0) = a+/*(0) = a - 5 Putting x= l,y = 2 in (1), we get/(l)=2
Now, g "(x) = a2^ + fix)
/«= /'(I)-
=> g"(0) = o2+/'(0) = 3 + a2 X

We have g'(0) + g"(0) = 0


5. (a) We have = f^~a .+ xy
a-5+o2 + 3 = 0 2 2
a2 + a - 2 = 0
fx + y) =flx) +fly) - a + 2xy
=> (a + 2)(a-l) = 0 Put x = y = 0
=> a=l,-2 => flQ) = a
Solutions S.33

x /(x + A)-/(x) (x-a)4 (x-a)3 1


f (x) = hm —-------—
*-»o h
8. (c) fx) = (x-6)4 (x-6)3 1
, jm /(x) + f(h) -a + 2xh- /(x)
A->o h (x-c)4 (x-c)3 1
f(h) - a
= lim2^--2---- + 2x 4(x - a)2 (x-a)3 1 (x - a)4 3(x-a)2 1
h-*0 h
/'(x) = 4(x-d)3 (x-a)3 1 + (x-6)4 3(x-6)2 1
+ 2x
A-»0 h 4(x-c)3 (x-a)3 1 (x-c)4 3(x-c)2 1
.3
=/(0) + 2x (x-a)4 (x-a)‘ 0
+ (x-b)A (x-b)2 0
A(x) = 2x+ ^Sa-l-a2
(x-c)4 (x-c)3 0
(x-a)4 (x-a)2 1
=>
= 3(x-Z>)4 (x-h)2 1
Putting x = 0, we get c = a (x-c)4 (x-c)2 I

/(x) = x2 + (y]5a-\-a2 )x + a A=3


/ /'(x) /(x)
75a-l-a2 5a -1 - a2 9. (b) =0
= x+ + a--------------- /"(X) /'(X)
2 4
2 /'(x)/'(x)-JAx)-r(x) = 0
75a-l-a2 * 4a - 5a +1 + a2
x+
2 1 4
+------------------ => (/z(x))2-/(x)r(x)_ ft
(/'(X))2

75a-l-a2>2 a2-a + l d r/(x)' = 0


x+
2
+----------- >0
4 ^[/'(x)]
(as a2 - a + 1 > 0) .=> 2W=C. 0)
/'(X)

fx) > 0 V x 6 R Putx = 0, -^- = C=>C = -


/'(0) 2
/(3 + ^)~/(3)
mx = rhm ------------ /(3-/Q-/(3)
, mi
6. (d) f(3) = hm .. =5
/i—»0 h h-*0 -h u /(-—r)- = -1
Hence —
/'(x) 2
f3 + h)-f3) = 5h +h =
From (i), 2fx) =f(x)
where Ej —> 0 when h -> 0
and f3-h)-f3) = -5h + h = £2 f'M=2
/(X)
where 0 when h -> 0
f3 + h) =f3) + 5h + h = £( = 4 + 5h + hEi
[A3 + A)] = [4 + 5/i + /i£|] = 4 when h > 0 and h is
4o°g/(x))=
ax
2
very small.
log(Ax)) = 2x + k
lim[/(3+ /:)] = 4 Putting x = 0, we get k = 0
a—*o
=> Ax) = ^
Similarly, [A3 - /»)] = [4 - 5h + he,] = [4 - A(5 -£,)] = 3 r e2i-1
When h > 0 and h is very small implying £, is very small. hm------- = 2
Jt-+O x
lim [A3 -/«)] = 3 * 4 = lim [A3 + /»)] 10. (d) Let degree of_Ax) is n;
A->0 A->0
Degree off(x) is n - 1
lim [Ax)] does not exist. Degree of/"(x) is (n - 2)
Hence n = (n-l) + (n-2) = 2/i-3
7. (c) Replace x by -x n=3
f2x1-l) = -2.x?f-x) Ax) = ax2 + bx2 + ex + d, (a*0)
2x3fx) = -2x2f-x) f (x) = 3ar + 2bx + c
f-x) = -fx) f\x) = (mx + 2b
fx) or odd function ax? + bxr + ex + d= (3ar + 2bx + c)(6ax + 2b)
/n) (0) = 0, when n is even 18a2 = a
a =1/18
S.34 Calculus
11. (a) We have/'(x) > 0 and f\x) > 0 1
Letg(x) =r'(x) g'(x) = (1)
/'(g(x))
/(5(x))=x
/(g(x)).g'(x)=l
1 _3g2(x)g'(x)
g'(x) = g"(x)
/'(g(x)) 271 + g3W
g'(x)>0 _3g2(x)
i (using (1))
g"(x) = ~ f"(g(x))g'(x) 2
(Z'(gW))2
15. (c)
<0 C.T(x)<0)
3/(x + y)=/(x)Ay) (i)
d\r\^ = g"(x)<0 r// \ r
f(x)= lim—-------—
/(X + /?)- /(X)
dx2 A->0 h
12. (b) X] = 27? cos A, x2 = 27? cos B, x3 = 27? cos C f&fW -/(x)
^- = -27? sin A i-
= hm----- 3- (using/rule)
dA /i-»0 h
. da .. /(x)-/(A)-3/(x)
Also a = 27? sin A — =27? cos A = lim--------------------------
dA />^o 3h

So, dxi . dx-, _ dx-> „ r(x),Uro/W.[/W-3]


=> f'(x) - lim
da db de /.-»o 3 h
Now tanJ + tanB + tanC> 3>/3 Put x = 0, y ~ 1
3XD=WD
dx} dx-> dr-, X0) = 3
So, —- + —*- + — <-3>/3
. da db de.
/W-/(Q)
13. (a) (fg)'=fg' 3 A —»o h
fg' +fg =f'g' => /'(x)=/W/'(0);
xg' + 3g = 3g' 3
(x-3)g' + 3g = O
(x - 3)3g' + 3(x - 3)2g = 0 => 3• =/(0) = k (say)
/(x)
(g(x-3)3)' = 0
=> Integrating, we get 3 log (/(x)) = kx + C;
(x - 3)3g(x) = c
Using/(0) = 3 => 3 log 3 = C
1 3 log (/(x)) = fct + 3 log 3
g(x) = (5(4) =1)
(x-3)3
31og /(x) = fcc;
14.(c) g is inverse of/ 3
/og(x)=x Using/(1) = 6 => 3 log 2 = it
=> Ax) = 3-2"
/Wg'^i
=> y<2)=12
Solutions S.35

CHAPTER 5

DPP 5.1 dy
at the point A, y = 0 and — = 0 for some t = z.
dx
ae01' = 1 (i) ;
Single Correct Answer Type also 0 = -z, + e"1; e01' = Zj (ii) ,
putting this value in (i)
1. (c) t2 + 3/ - 8 = 2 => t = 2, - 5
we get, aZj = I =>
2z2 — 2z — 5 = — 1 => / = 2, — 1 a
1
now from (i) ae = 1 => a = -
t^2 dy = 4t-2 (dy\ =6 e
’ dx 2z + 3 7=2 7 hence xA = tl+eat' = e + e = 2e
-7 => Point A is (2e, 0)
Equation of normal y + 1 = —(x - 2)
6
6. (a) x = 2Z3, y = 3Z2
2. (d) /(x) = x3 + ax and g (x) = bx2 + c pass through the point (-1,0) tangent at t is x - yt = -z3
=> /(-l) = 0;g(-l) = 0; and normal at z1 is, xZ, + y = 2zj* + 3z2
=> -1 - a = 0 and b + c = 0 (•) Since both equations represent same straight line,
Also curves have common tangent at this point 1^ -t3
=> A-D = g'(-l) Zj 2zi4 + 3z12
=> a + 3 = -2d (ii)
z6-3z2-2 = 0=>z2 = 2=>z = ±V2
From (i) and (ii)
lines are x = ±-fl (y - 2)
a = -l; b = -l; c = 1;
hence a + b + c = -1 7. (c) 2y^- = (2 - x)2 - 2x(2 - x)
3. (b) According to the question we must have dx
f{x) = 0 andf"(x) = 0 for the same x = x0 =
So
now /(x) = 4x3 + 2bx + 8 Mu, 2

f(xQ) = 2[2x03 + bxQ + 4] = 0 (i) Therefore, the equation of tangent at (1,1) is


and r(Xo) = 2[6xo2 + h]=O (ii)
'.2
From (ii) b = -6x,
0
-x + 3
Substituting this value of b in (i)
y=~T~
2x03 + (-6x02) + 4 = 0 => 4x03 = 4
The intersection of the tangent and the curve is given by
=> x0=l.
(l/4)(-x + 3)2 = x(4+x2-4x)
=> b = -6 4x3-17x2 + 22x-9 = 0
4. (a) Let (3, a) be the point on y = g(x) => (a, 3) lies on y =/(x) => (x- l)(4.r2- 13x + 9) = 0
=> a= 1
=> (x-l)2(4x-9) = 0
Also x =^g(x))
Since x = 1 is already the point of tangency, x = 9/4 and
1 1
g'(3) = , 9< 9?
/'(g(3)) /'(I) 4 9 9
r = - 2--
44l 4J 64
Tangent is y - 1 = — (x - 3) or x - 4y + 1=0. Thus, the required point is (9/4, 3/8).
4
5.(b) x = z + efl';y = -z + e"r 8. (c) y = ar4 + bx3 + ex + d

dx , dy . dy -l + ae01 => dy/dx = 4ar3 + 3b.r2 + c


— == 11 ++ ae
ae00''1
,1
. — = -1 + ae"', — =--------—
dt dt dx 1 + ae Point (0, 1) satisfies the curve => d = 1
S.36 Calculus

Also
V d* 7(0,1)
= 0 => c = 0 (1) ez Multiple Correct Answers Type

12. (b, c)
(-1.0) satisfies the curve =>a-b-c+l=0 (2) Let the tangent line be y = ax + b
= 0 => c = 0 The equation for its intersection with the upper parabola is
A!so
(-1.0) £+\=ax + b => x2 - ax + (1 - b) = 0
=> -4a + 3b = 0 and a-b = -\ This has equal roots when a2 - 4( 1 - b) = 0 or a2 + 4b = 4 (1)
For the lower parabola
a = 3,b = 4
ax + b = -x2 => x2 + ax + b = 0
The curve is y = 3a-4 + 4a3 + 1
This has equal roots when a2 - 4b = 0 (2)
dy/dx = 12a3 + 12.x2 From (1) and (2) Sb = 4 or b = 1/4
For dy/dx < 0 we have x3 + x2 < 0
Add 2a2 = 4 or a = ± V2
9.(d) x = 2/-/2 (i) The tangent lines are y = V2x + and y = —J2x + ± .
and y = t +12 (ii)

dy _ dy/dt 1 + 2/ 13. (a, b)


dx dx/dt 2-2/ It can be easily checked that function is continuous and
differentiable at x = 0.
Slope of tangent using the points (1, 1) and (2/ - t2, / + z2)
dy _ / + /2-l Now ±=£Z (i)
(iii) dx /'(/)
dx~ 2t-r-\
/ + /2 — 1 1 + 2/ -sin t2 - 0
Equating (ii) and (iii), g'(0)= lim
g(0-g(0) = lim t
2/-r-l 2-2/ =1
>o /-0 z->Ok. /
3/2-4/+ 1 =0 2 ■ 1
.. //(/)-/(0) 2/ + /,2 sin -
=> /=!.’ f\0) = hm = lim------------ - = 2 + 0 = 2
3 /-O /->0 t
1 <5 4^
For / = 1, point is P (1, 2) and for / = point is I.
3’ Z=1
2yfS3 dx 2
PQ =
9 So the equation of tangent y - 0 = (x - 0)
/-4
10. (b) Slope of line (4 - /)x + ty + (a3 - 1) = 0 is----- .
/ and equation of normal y - 0 = - 2(x - 0)
-1
For xy = I, — = — 14. (a, c)
dx x 7
t—4 dy _dy/dt ________ 3a cos2 / sin /________
Slope of normal = x2 =-----
dx dx/dt 2a cost + a cos3 / - 2a sin2 / cos /
• , t-4 n _____ 3a cos2/sin/_____
Asx2>0, — >0
2acos/(l - sin2 /) + acos3 /
/e(- >, 0) u (4, oo)
3a cos2/sin/
11. (b) ^ = ^ 2a cos3 / + acos31
dx 3 3acos2/sin/
Tangent is y - a/4 = -y(x-a/3) 3a cos3/

a/3 dy
x-intercept= = tan /, equation of normal is
dx
at4 y + a cos3 / = (x - 2a sin / - a sin / cos2 /)
y-intercept = —— sin /

the point of intersection of tangent with the axes are y sin / + a cos3 / sin / = -x cos / + 2a sin / cos / +
a sin / cos3 /
at4
[Xolandlo,- — x cos t + y sin t = 2a sin t cos t
I 4 J k 3 . It meets axis at A (2a sin t, 0) and B(0, 2a cos /)
nt 3 Then AB = 2a.
n 4
Solutions S.37

15. (a, b, d) Angle between curves is given by

— = 3ax2 + 2bx + c = tan 45° = 1 at x = 0 x x


dx -+-
0 = tan y y
c= 1 x2
i _
dy y2
— at (1, 0) = 3a + 2b + c = 0 as x-axis is tangent.
dx 2xy
tan
3a + 2b + 1 = 0 v c = 1
(1,0) lies on curve, Squaring and subtracting the equations of given curves,
a+b+ 1 =0 4x2y2 = a4
Solving we get a = 1, b = -2
2xy = ±a2
fix) = xi-2x2+x
and fix) = 3X2 - 4x + 1

DPP 5.2
0 = tan

= tan(l)
-a
n
4
Single Correct Answer Type
4. (b) Since y = e' and y = log? x are inverse to each other their graphs
1. (c) x2-4y2=4 (i) are symmetrical about y = x.
2x-8yy' = 0 Minimum distance between the curves is the distance between
Slope of normal is 1. the points on the curves where tangent is parallel to y = x or
Slope of tangent is -1. slope of tangent is 1.
x + 4y = 0 (ii) Now for y = e1, y' = e1, for e* = 1, x = 0, so the point on the
Also solving (i) and (ii), 1 by2 - 4y2 = 4 curve y = eK is (0,1) and symmetric point on the curve y = logr
Js(l,0).
y2 = 1/3
( 4 1 Distance between these points is 72 .
Point on the curve is +-?=, ± -7=
V V3 V3,
_ . r . dx
Length of normal = y—
dy
1 1
Given —7= = —7=
4k 43
k=3
2. (b) Point of intersection of curves is (1, 0).
y = 31"1 Inx
3X-1
y' = — + 3X~ ’.log^.log, x
dv y,
5. (d) y( = 72 si — = — where (x., v.) is point on the
m, = 1 4) dx X]
y = xr- I curve
x f 2 \
~ = xt(l+logr) 4 = 2 cos2 x, + —
7t 1 = 2 —+1
dx
m2 = 1
.‘ 4 I2 J
. .2 1 ,
Angle between curves is given by Locus of (Xp y|) is — —- = 1

1-1
tan 9 = = 0 => 0=0 => cos 0° = 1 -3
1+1 6. (a) Slope of 3x + 2y = 1 is —
2
-> = a242 and x2 - y2 = a2 -3
3.(0
2
dy _ -x rfy _ x
dx y ’ dx y
S.38 Calculus

lim
■2x
Multiple Correct Answers Type

2 J 9. (a, b)
2x-l
= lim ^st ■SN = W
m
e2xf
yy/\ + m,2
JI Lt~
m
,£n = |j71 + "»2|
3
where m = — at point P = (x, y) on the curve y =f(x)
dx
7. (a)
( , \2
y Now isL = -L andLsrLsN = y2
^SN m
tr„x13/2) ■y = XyJX

10. (a, b, c, d)
dy 16x
(a) 16x2+y2 = c=> — =m.= -
dx y
0(0,0) 4(4,0)
dy k
yl6 = kx — = m,=
dx 2 16y15
AP will be least if AP is normal at point P. 16x k
Slope of AP x Slope of tangent at point P = -1 /Ml?W2 = ~
T 16j',15
Slope of tangent at point P
\dx)p 2 'x\ = -—k
y“
3 r- X|3/2-0
2^ = -l
X)-4 \16 c '

3x12=8.-2r1 => Curves are orthogonal.


=> 3x12 + 2x1-8 = 0 (b) y = x + ce"x
dy
=* (x1+2)(3x1-4) = 0 = 1-ee’x= l-(y-x) = -<y-x-1)
4
=> x. = - x + 2 = y + ke~y
’ 3

Poi„t pg, gf ] =>


dx dx

[1-^ = 1
dx
OP-^ [l-(x + 2-y)] ^=1
or
V 27 dx
dy 1
8. (d) y = 3x + 3 is normal to the curve at x = 0 — = nij =---------------
dx 2 y-x-\
Slope of normal is 3.
"’l m2 =
Curves are orthogonal.
Slope of tangent at x = 0 is /'(0) = ~
(c) y-cx^
x2 dy _ y 2y
Now limu------ ~it— i - form =2cx= 2x-^- = — = m,
x->(°{/(x-)-5/(4x2) + 4/(7x2)} l0 dx x x

.. 2x and x2 + 2y2 = k
= lim--------- ------------------------------------ —
2x/'(x2) - 40x/'(4x2) + 56x/'(7x2)
2x + 4y — =0
= lim------------ J------------- dx
x-*o/'(0)-20//(0) + 28/,(0)
x
--------- = mj
1 dx iy 2
= 9/'(0)
Mj/Mj = -1
Solutions S.39

=> Curves are orthogonal. 4


4. (b) V = -Jtr3
3
(d) x2-y2 = c=> — = — = m,
dx y dV 4 , 2 dr
dt 3 dt
xy = k=> — = - — = m2
dx x 2 dV dr
Given — = —
dt dt
mlm2 =
2 1 1
Curves are orthogonal. r,2 = — => r = —
4x 2yjtc

DPP 5.3 k=2

dV
5. (d) Given that — = 2
dt
Single Correct Answer Type
d(l
=> — —nr'.3 = 2
dt\3
l.(c) u = >/jc2 + 16 2 dr <■»
=> Ttr2 • — = 2
du 2x x dt
~dx 2y]x2 + \6 7*2 + 16 dr 2
~dt
x
v =------
x-1 d 4
=> (i)
dv _ -1 7,^-7
,2
dx (x -1)' When h = 2 m
du du/dx -12 r = 2m
~dv dvldx 5
Hence,
dt 4
2. (c) Eccentricity of ellipse = —

=-0.1 (given) 6. (c)


dt
Eccentricity of auxiliary circle = 0
o T O
=> J <te = -0.1p X1O-A)' r /
1/2 0
x |/icm
T is time at which it will touch the auxiliary circle.
dh
1 = -0.1 (7-0) T= 5 seconds. — = - 2, r = 10 cm
2 dt
dx
3. (a) y = x2 We have to find — when h = 4, where x is the radius of the
top surface.

7>(x,x2)
From the figure i2 = x2 + (10 - h)-
dx _ _ dh
0 2x— = 2(10-A) —
M dt dt

dx (10 -h)
. x2 => (-2)
dt ~ x
tan 9 = — = x
x
dx -2(10-4) 12
•> nd9 dx (i)
=> sec 9 — = — dt x x
dt dt
When h = 4, then x2 = IO2 - 62 = 64 or x = 8.
=> — = 10 x cos2 9 = 10 x — = 1 (atx = 3 m}
dt 10 dx -12 3
~dt 8 2
S.40 Calculus

7. (b) 1 -2x dx 22.5 d£


V 2 7169-x2 dt £2 dt

e ,2
-12 r -22.5 d£ d£ _\2C
to ---- x5 =
o 5 £2 dt dt ~ 22.5
Q
3
By the given condition, we have
A r r
B
O 15 , 15 tft
—+ 1 = — =10
£ 1.5
Let 0be the semi-vertical angle and r be the radius of the cone
at time t. d£ = 12 (15? 120 _ 40
__ . X
Then, r = 20 tan 6 I 9 J ~ 81 ”27
dt 22.5
dr -> n de
=> — = 20 sec‘0—

=>
dt dt

de = 2
— = 20 sec2 30° x 2 v0 = 3O°and^
DPP 5.4
dt L dt
dt
Subjective Type
dr 4 „ , 160
— = 20 x — x 2 cm/s = — cm/s
dt 3 3
1. Consider)(x) = (x - A) log x for 1 < x < A.
dy The function/is continuous on [1, A]
e
8.(d) y = -^ = a sin 0 = tan — x-A j
dx dx a(l + cos 0) 2 and /(x) = log x + for all xe (1, A)
x
de
Also,/( 1) = 0 and/(A) = 0.
The rate of change of the slope, i.e.,
So, by Rolle’s theorem, there is c e (1, A) such that/Xc) = 0.

*=l±L>sec
dt dt 2
2e dd k
= —sec
2 dt 2 2
=> logc +
c- A
c
=0

c log c + (c - A) = 0
n . . k
Required rate = — • sec
■>n £ 2
c log c + c = A
6 2 Hence, log x + x = A has at least one solution in [1, A].
f(c) t g'(c) _ t
Comprehension Type 2.
.. - — — j /(a)-f(c) g(b)-g(c)

9. (a) Let \VB = x at time t. Then Put c = x


x/13 = cos 6 f(x) + g'(-v) = _j
/W-/(a) g(x)~g(b)
dx Ilf • n.de
— = 13(-sin0)—
dt dt => -j- (logt.(/(x) -fia)) + ~ (log (g(x) - g(b)) = -1
ar dx
When \VB = 12, then
5 dd d[(log,(/(x) -/<?)) (g(x) - g(b))] = -dx
5 = -13----------
13 dt (logf(flx) -/a)) (g(x) - $(/>))] = -x + k
dB
— = -l rad/s
. (fix) -fia)) (g(x) - g(b)) = e~x + *
dt
(fix) -fia)) (g(x) - g(b)) ex - e* = 0
10. (b) WM = 15 m
Now let h(x) = (fix) -fia))(g(x) - gtb))^ -
Height of the lamp post at time t = ^169 - x.2 + 10 m
h(a) = h(b) = -ek
Let / be the length of shadow at time /. Then
Then by Rolle’s Theorem, there exists atleast one
c e (a, b) such that
7l69-x■2 +10 15 + / 15 ,
----------------- =------- = — +1 h'(c) = 0
1.5 £ £
V169-x2+10 = ^ + 1.5 f'(c) | g'(c)
=>
/(a)-/(c) g(b)-g(c)
Solutions S.41

Clearly fiO) =/(!) = 0


Single Correct Answer Type
Hence, fix) = 0 atleast two real roots.
Also for atleast one x e (0, 1 )./'(x) = 0 (Rolle’s theorem)
3. (c) fix) = x- 1, 1 <x<2
Obviously function is many-one.
g(x) = x - 1 + b sin — x, 1 < x < 2 9. (a, b)
2 All four functions tan"’x, sin"’x, cos-Ix, cof’x are continuous
fi 1) = 0:7(2) = 1 => Rolle’s theorem is not applicable to */ but and differentiable in their domain.
LMVT is applicable to/ So by L. M. V. T.
(•.• x - 1 is continuous and differentiable in [1, 2] and (1,2) tan x- tan y
respectively) (a)
x-y 1 + c2
Now g( 1) = b; g(2) = 1 and
Function x - 1, sin — are both continuous in [ 1,2] and (1,2) sin" x - sin" y 1
(b) = >1
2
For Rolle’s theorem to be applicable to g, we must have b = 1
x-y 7T?.2
f(b)-f(a) COS X - cos y
4. (c) (C) = >1
y-x ,2
b-a
=> a+b= 1
cot" x-cot y
5. (b) Let m e (a, c), v e (c, b) (d)
y-x l + c*
Then by LMVT on (a, c), (c, b), we have
= /(c)-/(«); //(v) = f(b)-f(c)
/'(«)
c-a ’ b-c Comprehension Type
But« < v and/"(x) < 0, V x e (a, b)
10.(c)
i.e. /'(u)>/'(v)
(fix) =fib) -fix) -{b- x)f(x) -{b- x)2 A
/(c)-/(Q) , c-a <fib) = Q
=>
J\b)-f(c) b-c tfia) =fib) -fia) -{b- a)f'(a) -(b- a)2 A
6. (a) Let/(x)=x1/3 Since Rolle’s theorem is applicable
=> 4 f(b)~ f(a)-(b-a)f'(a)
= 3x2/3 A =--------------------- ;------------ (>)
(b-a)‘
Applying LMVT in [a, a + 1], we get one c e (a, a + 1)
<p'(x) = -f'(x) +f(x) -(b- x)f"(x) + 2(b - x) A
/(a + l)-/(a)
/'(c) There exists some number c e (a, b) such that
a + l-a
0 = 0'(c) = -(b - c) f'(c) + 2(b -c)A
=> lla + l -l/a= < —Ur
V 3c2/3 3aJi 3n2 A = ^f"(c)
i.e. (ii)
lla + \ -\Ja < —Va > n3
V 3n2 From (i) and (ii).fib) -fia) -(b- a)f(a) = | (b - a)2f\c)
7. (d) Given /'(D=i; .
And f(2x)=f(x)
i.e. fib) =fia) + (b- a)f(a) + | (b - a)2f'(c)
2f(2x)=f(x)
. ..... ZZ(D = 1
Putting x= 1, /'(2) =------ A= '
' ■ ' 2 2
2
Putting x = 2, /'(4) = ^ = 1
2 4
11. (c) fil+h)=fil) + hf(l) + |/rr(c)
Now applying LMVT for y =/'(x) in [2,4]
fi\ + h) = 1 + 3/r c

We get/"(c) =
r(4)-r(2) 4 2= 1 /(1 + ft)-/(I)
= 3c
2 2 8 h2
A=3
I
Multiple Correct Answers Type 12. (b) sin (a + h) = sin a + h cos a+ — lr(- sin (a + th))
2 ’
8. (a, b, c) sin (g + A) - sin a - h cos a 1 . ,
= -—sin (a + th)
1 1 1 h2
fix)= 3-x 5-3x2 3x3-l 1
a=—
2
2x2-! 3x5-1 7x8-1
S.42 Calculus

CHAPTER 6

=> xc-1/2
DPP 6.1 So, no solution for this case.
5. (b) fx) = sin 2x - 8(a + 1) sin x + (4a2 + 8a - 14)x

Single Correct Answer Type


1. (b) fx) = x sin x + cos x + cos2 x
□ f(x) = 2 cos 2x - 8(a + 1) cos x + 4a2+ 8a - 14
= 4 cos2 x - 2 - 8a cos x - 8 cos x + 4a2 + 8a - 14
Now/x) increases for all x e R
f(x)>Q
f(x) = sin x + x cos x - sin x - 2 cos x sin x cos2x- 2(a + 1) cos x + (a + I)2 - 5 > 0
= cos x(x - 2 sin x) (cosx-(a+ I))2 >5
For x e (0. id2), x < 2 sin x. => cosx-(a+ 1)> V5 orcosx-(a+ 1)< -^5
f(x)<Q
i.e.,fx) is decreasing function. => a < - >/5 - 1 + cos x a> V5 - 1 + cosx
2. (d) fx) = 2X3 - 3x2 + 6 a<-V5 -2 ora> >j5
f(x) = fix2 - 6x = bfr2 - x) = 0
=> x = 0 or x = 1 6. (c) /(x) = 2e' + ke~x + (2k + 1)
It
(0,6b\ = 2ex+ — + (2k+ \)
ex
/5-"-
2(ex)2+ k + (2k + \)ex
ex
Now/x) is monotonically increasing.
+ ->x => /'(x)>0i.e.,2y2 + (2fc + l)y + k>0
O 1 where y = ex
For inverse to exist function must be one one onto .•. 2J2 + (2k + l)y + k > 0 for all positive value of y.
Hence domain is [ 1, «>) For this k > 0.
Hence a > 1 7. (a) fx) = 2 cot x + (2a + 1) logjcosec x| + (2 - a)x
3. (c) fx) — 1 -x-x3 =♦ /*(x) ~ ~ 2 cosec2x - (2a + l)cotx + (2 - a)
=> /*(x) = -1 - 3X2 < 0 for all x = - 2 cot2x - (2a + 1 )cot x - a
fx) is decreasing for all x
Now 1 - fx) -f(x) >f 1 - 5x) = (cot x + a)(- 2 cot x - 1) < 0 in ^0, ~
ffx))>f\-5x)
fx) < 1 - 5x (as fix') is decreasing) cot x + a > 0 in I 0, —
1 -x-x3 < 1 - 5x I 2.
=> x3 - 4x > 0 Hence, a > 0
=> x(x - 2)(x + 2) > 0
xe(-2,0)u(2.~)
4. (b) Since/"(x) > 0 Multiple Correct Answers Type
=> f(x) is always increasing.
8. (a, d)
g '(x) = 2/'(2x3 - 3X2) x (6-r2 - 6x) +
/'(6x2-4x3-3)(12x- 12X2) (a) True
= 12(x2 -x). (TC2X3 - 3X2) -/(6X2 - 4X3 - 3)) If fx) is not constant on [-1,1]
= 12x(x - 1) [f(2x3 - 3X2) -/(6X2 - 4X3 - 3)] =* f(x) > 0 at some point in -1 < x < 1
For increasing g'(x) > 0 => /is increasing at that point and we can’t have
Case I X-1)=/(1)
x < 0 or x > 1 =>/'(2x3 - 3X2) >/z(6x2 - 4X3 - 3) (b) Not true
lx3 - 3x1> bx2 - 4X3 - 3 {•//'(x) is increasing} Iff is bounded curve changes its concavity
_1_ If/is unbounded then curve crosses x-axis
=> (x-1)2
2
>o (c) Not true
e.g./x)=x4
=> x> -- xe ——, 0 | U(l,<») (d) /*(x) = 2X2 + 2x-12 = 2(x+3)(x-2)<0forxe (-3,2)
2 2 )
wherefx) decreases.
Case II
0<x< 1 Also f"(x) = 4x + 2 > 0 for x >
=> (x- I)2 (x + 1/2) <0 2
Solutions S.43

9. (a, b, c, d) y = -e~x + k is increasing for x < 0


fix) = X + logc( 1 +X2) y = r1 + 1 is increasing for 0 < x < 1
2x y = er2 + A is increasing for x > 1
==■ /W“1 + T^7 fix) is increasing for V x e R
(X + I)2 => 7(0-) <710*)
> 0, V x e R -1 +k< 1 + 1
1 + x2 =>
So,7(x) is injective and at x = -1, tangent is not parallel to any *£3
of the chords. and7(l-)<7(l*)
Also lim f(x) => e+ 1 <e + A
=> A> I
= lim logf(ex(l + x2))

= logJ lim
1 + x2
e ' J
DPP 6.2
( 2x
= log J hm —— Single Correct Answer Type
—g x )

. ( 2 n I, we know —*■ is decreasing function


= logc hm —— 1. (d) ForxpX2G 0, —
e~x J 2J tan x
= —oo
x, x2
lim /(x) = °° => —!—>——
tan X] tan x2
Thus7(x) has range R, so fix) is surjective.
tan x2 > x2
10. (b, c, d)
tan xt xt
/(X) = x-- 2. (b) Given/'(x)-3/(x)>3
X

/'(x) = l+-4>0 4(e-3j7r«)>3e-3x


X dx
/is increasing
7(i)=7(-i) = o — ie~3xfix) + e~3x) > 0
dx
7(x) is many-one
When x —>0+; => e_Jx (/*W + 1) is increasing function
Whenx —> °°» J —00 Also e~3x (f(x) + 1) >/'(0) + 1 V x > 0
When x -> 0-; /-> +°° => fix) + 1 > 0
And when x -> -<«, => fix) + x is an increasing function.
So the graph of the function is as shown in the figure. 3. (b) We have (x-41)49 + (x-49)41 + (x-2OO9)2009 = 0
Let/x) = (x - 41 )49 + (x - 49)41 + (x - 20O9)2009
fix) = 49(x - 41 )48 + 41 (x - 49140 + 2009 (x - 2009)48 > 0
Hence fix) will cut x-axis only once => 1 real root
4. (d) Given g(x) = log^hfx))
Wt)
gV(x)x = -2-2
hix)

<0 (given)
h2(x)
From the graph any line y = k meets the curve at two distinct g"(x) <0 => g(x) is concave down
points. 5. (b) fix) = sin x + x,
Also/"(x) = -2/x3 => 7*(x) = cos x + 1 > 0
/"(x) > 0 for x < 0 => 7(x) is increasing function
and/'V) < 0 for x > 0 Also fix) = -sin x < 0
Hence x = 0 point of inflection.
11. (a, d) /is concave downward for x e 0, y
As/(x) is even,7(-2) =7(2)
As J(x) is increasing in [2, 6],7(3) <7(5) =7(-5) Now g(x) = I k I x2 is concave upward and increasing
Also7(3)<7(5)=>7(-3)<7(5)
then/x) > gix)
12. (b,d)
-e + k, x<0
=> 1 + - >1*1 —
ex +1, 0<x<l 2 4
ex2 + A,
S.44 Calculus

'-(2^ + 4) (2/r + 4)~ 9. (a) As y+ p - a > y


=> ke it1 ’ 2r2 .
f(Y + P-a)^ fjy)
6. (b) Let/x) = 4x + 8 cos x + tan x - 2 sec x Y+fl-a y
-41og{cosx(l +sinx)} -6
Now/0) =0+8+0-2-4 log 1-6 = 0 and 10. (c) f"(x) > 0 =*>/'(x) is an increasing function
h'(x) = sin 2x (/'(si^x) -/(cos2x))
/(x) = 4 - 8 sin x + sec2 x - 2 sec x tan x + 4 tan x - 4cosx /(x) = 0 => sin2 x = 0 => x = 0
1 + sin x
or/(sin2 x) =/(cos2x) => sin2x = cos2x=> tan2x= 1 =>x=
=4( 1 - 2 sin x)+sec2 x (1 - 2 sin x) + - ±c?sx^ Slnx] ± —it
cos x cos“ x ”4

4 11. (a) /'(sin2 x) < /'(cos2 x) => sin2 x < cos2 x => tan2 x < 1
= (4 + sec2 x)( 1 - 2 sin x) + (sinx- 1 + sinx) f 1t 7T A
cosx
= ( 1 - 2 sin x)(4 + sec2 x - 4 sec x) 4’4J
= (1 - 2 sin x)(sec x - 2)2 12. (b) /i(x) is increasing => /i'(x) > 0
Thus/(x) > 0 if 1 - 2 sin x > 0 i.e. x e (0, te/6).
Case I
Therefore /increases on [0, tz/6).
But/0) = 0, so/x) >/0) for all x e [0, tz/6) (i) sin2x>0=>xe o,— ]
Thus the largest such value of A is tz/6. 2) it it
=>xe
(ii) /(sin2 x) >/' (cos2 x) => tan2x > 1 7’?

L Comprehension Type J Case II

7. (b) Consider g(x) = Till


(i) sin2x<0=>xe Ao
2 . -it n
=>xe
x (ii) f(sin2 x) >/ (cos2 x) => tan2x > 1 T’ 7
O7^_ x/'(x)- /(x) _/'(x) r /(x)
g (x) =-------- 5------- = —r-
X* X /'(x)

as r(x) >0
X2
DPP 6.3
/(x)
Let h(x) = x - Single Correct Answer Type
/'(x)
1. (d) fix) = tan x + cot x + cos x
/'(x)2-/(x)/"(x)
A'(x) = l- 2
/'(X)2 =--------- + COS X
sin 2x
/(x)/"(x)
>0
/'(X)2 Both —-— and cos x decreasing on [rt/6, tz/4] and thus the
sin 2x
r /(x) greatest value occurs at x = rt/6
is increasing function.
/'(x)
2 11 1173 ’
i.e.----------- + COS It/., 4 V3
6 = -7= +----
sin x/3 73 2 = 273 = 6
(x + 1)3 -2<x<-l
,2/3 _j
2.(b) f(x) = < x -l<x< 1
-(x-1)2 l<x<2
a p Y
Clearly/x) is continuous.
r /(X)
>o--^)=o 3(x + l)2 -2<x<-l
/'(X) /z(0)
2 .
g'(x)>0. /'(*) = -xx.-1/3 -1 < x< 1-{0}
3
/(y^/q^/W -2(x-l) l<x<2
Y P a
Sign scheme of/(x)
B is false
8. (b) As the function is concave upward + +
H- ------b + ------ b -H
/(«) + /(£) a+P -2 -I 0 1 2
2 2 Critical points-1,0, 1
Solutions S.45

x + Vx ifx>0 6. (c) Local maximum at x = 2


3. (d) fx) =
x ^x ifx<0 => lim/(2 + A)</(2)
A—>0

The graph of fx) is shown with /(x) = 0 as x = -1/4. Also lim(Va + 14 -12 + h - 48|) < 3 - 22
derivative fails at x = 0. Hence there are two critical points.
=> 7a + 14 < 45 => a < 2011
7. (d) f(x) = Sx4 - 20X3 + 15X2
—I—
/"(x) = 20x3-60x2 + 30x
-1/ -1/4 0 x /(x) = 0=>x2(x2-4x + 3) = 0
=> x2(x- l)(x-3) = 0=>x = 0, 1,3
/"(I) < 0 and/"(3) = 540 - 540 + 90 > 0
and/"(0) = 0 and/"(0) * 0
tan I x + n— .*. Maximum atx= 1 and minimum atx = 3
4. (d) /(x) =
I 6 = cot x tan I x + —
n 8. (c) fix) = lx - II + lx + 41 + lx - 91 + ... + lx - 25001 is non-
tan x k 6. differentiable at x = I2, x = 22,.... x = 502.
n fix) is minimum at the middle term of the above series which
f'(x) = cot x sec2 I x + - cosec2 x tan x + — are x = 252 and x = 262.
6.
9. (b) The slope of the tangent to the curve
n n _ x . fx^l (k x>
f'(x) = 2 cot x sec2 tan x + —
* 7. 6. y-2e sin------ cos-------- = ex cos x
U 2) U 2)
- cosec2 x sec- x + -r „ dy x. . .
k 67 S= — -e (-smx+cosx)
dx
[ n
- cosec2 x sec2 x+"7 dS ,
\ o. Now, — = r (-sin x + cos x - cos x - sm x)
dx
„ , \ n
+ 2 cosec2 x cot x tan x + — = -2e* sin x
\ o.
Now rw=o — = 0 => -2e* sin x = 0
dx
— sin 2x = — sin | 2x + — => x = 0, n, 2/r(v0<x<2^)
2 2 3 Value of S at x = 0 is 1; value of S at x = x is -eK
Value of 5 at x = 2n is e2*.
=> 2x= n-2x- —
3 S is minimum at x = n.
K 10. (b) /(x) = x3 - 3ar + 3(a2 - 1 ).r + 1
=> X= —
6 f(x) = 3-r2 - 6ar + 3(a2 - I)
Then f"(n/6) > 0 = 3(x-(a+ l))(x-(a- 1))
so/'(x) = 0 => x = a + 1 or a - 1
At x = — ,/(x) is minimum and there is no other minimum
a+ 1 6 (-2.4) if a 6 (-3,3)
and a - 1 e (-2,4) if a e (-1.5)
• 0,— .
in
I 27 / x a e (-1,3)
( 7t K
tan —+ —
The minimum value of/(x)= f — I = 16 6 Multiple Correct Answers Type
k6/ n
tan -
6
T=
v/3
3
11. (a, d)
x2 + 2, 1<x<2 /(x) = ar e~bx has a local maximum at the point (2, 10)
x2 +2 _fl2) = 10.2ae-2Z’= 10
5. (b) /(x) = , 2<x<3 => ae-2i = 5
2 ’ (i)
x2 + 2 f(x) = - bx
x=3 Z(2) = 0
. 3
a(e-2A - 2be-2h) = 0
Least value of_flx) in [ 1, 2] is 3
Least value of/(x) in [2, 3] is 3 ae~2b(\-2b) = 0
b = 1/2
713)=^ From (i) if b = 1/2. then
.•. Least value of/(x) is 3 a = 5e or a = 0 (not possible)
.*. a = 5e and b = 1/2
S.46 Calculus
12. (b,d)
.-. /has local max. at x = -
eT /2
/(*) =
1 + x2
Also lim logc(71 - x2 - x) = -oo
g(x) = T(x)=£-XLv O2 x->4=
(1 + x2)2 72
g/(j)= (X -1) (X3 - 3x2 + 5x + l)eJ /has no minima.
(x2 +1)3
Now, h(x) = x3 - 3.x2 + 5x + 1
=> h '(x) = 3.x2 - fix + 5 which is always positive.
DPP 6.4
Hence x3 - 3X2 + 5x + 1 = 0 has only one real root which is not
x= 1.
Also /i(0) = 1, /i(-l) = -8. So h(x) = 0 has one negative root
x=ae (-1.0).
Single Correct Answer Type 1
Now from the sign scheme of g'(x), x3(l - x), x<0
x = a is the point of maxima and x = 1 is the point of minima. l.(d) A*) =
xlnx + 3x, x>0
13. (b, d)
Given/(x) = (x - a)2010 (x - b)2009 and a > b AO) = 0 (using x3(l —x))
Sign scheme of/(x)is A0’) = 0
- + + /(0+)= lim(xlnx + 3x)
------- 1---------- 1-------- x—»0
b a
In x .
From sign scheme, x = b is point of minima and x = a is neither = lim ——+ 0
maxima nor minima. x->0+ 1
X
14. (a, d)
We have lim fix)= lim [/(x)]. 2
x-ta x-»a
= lim -V
This can occur only when lim fix) is an integer, *->o+ __L
x—
x2
Afl+) >fia) andAa") >fia)
=0
x = a must be point of local minima.
15. (a, b, c) Thus, function is continuous at x = 0.
/(x) = loge(^/l-x2 - x) A0)>A0-),A0) >A0+)
x = 0 is point of maxima.
y]\-x2 - x is defined for x e [-1.1]
Also (x In x + 3x)' = 4 + In x
Ifxe [-1,0], 7b^?-x>0 4 + lnx = 0=>x = e^
(x In x + 3x)" = 1/x > 0 for x > 0
Ifx>0, then Ji-x2 >x=> 1 -x2>x2 Thus, x = e’4 is point of minima
lim x3(l—x) = —°°
i.e. x g o'V2,
and lim (x In x + 3x) = oo
Domain of/is -1, -L x—>°°

Range offix) is R.
x3
-^=-1 2. (a) The ordinate of any point on the curve is given by y =
A(x) =
J^x2 (x-a)2

Jl - x2 -x dy_ 3x2 2x3 _x2(x-3a)


I3’ (x-o)3 ’
—x
dx (x-a)2 (x-4
fix) > 0 if -l>0 dy
— = 0=>x = 0orx = 3a
dx

or —x - ^1 — x2 > 0 “ “ +
------- 1---------- 1--------
0 3a
or xe -1.^1 From sign scheme of x = 3a is point of minima and value of
V2
y at x = 3a is
1 1 (3°)’ 27a3 27
.'. fix) increases in -1, decreases in y= = —a
"V2’V2. (2o)2 4a2 4
Solutions S.47

3. (a) Let y = x - xP, where x is the fraction la - 11 +lb - II = lai + Ibl = la + ll + lb + II


^ = 1 - pxp-' => /(0) =/(!) =/(-!)
=> {-1,0. 1} e {a,b}
dx
la-Wmin = 2-
For maximum or minimum, — = 0 7. (c) Letyfx) = 2x3-6x + 2sina + 3
dx => /'(x) = 6x2-6 = 0=>x = ±1
Since equation fix) = 0 has only one real root
=> 1 -pxP~x = 0=>x = /(-l)./(l)>0
p)
=> (2 sin a + 7) (2 sin a - 1) > 0
NOW, —y =-p(p-l)j^~2 1
dx => sin a> — =>ae 6’ 6 J
2
f . \(P-2)/(p-1)
= -p - <0 8. (c)
2/WAx) -2x>0
dx2 \PJ

i/(p -1) => — (sin-1 (/(x))2 - x2) > 0


.. y is maximum at x = I —1 dx
\P. Then g(x) = sin-1 ((/(x))2) -x2 is a non-decreasing function.
2-ex'1, l<x<2 => lim g(x)< lim g(x)
4.(c) f(x) = g'(x) =
x-e, 2<x<3
=> -~x
2 2<--x
x2
g'(x) = 0 => ex~1 = 2 => x = 1 + ln2
and x - e = 0 =$■ x = e
Graph of/(x) is as shown in the following figure. => 2^ K
x,2 ~x2 >-

2a
1-- 9. (b) ab = 2a + 3b => b =-----
a-3

+ 4- A4- XT L 2fl2

1 .2 e3 Now z = ab =-----
a-3
1 + In 2 dz 2[(a - 3)2a - a2 ] 2[a2 - 6a]
=>
di~ (a-3)2 ” (a-3)2
From the figure, derivative changes sign from *+’ to at
x = 1 + In 2 and from to *+’ at x = e Put — = 0 , .’. a~ - 6a = 0, a - 0, 6
da
Hence x = 1 + In 2 is point of maxima and x = e is point of
Clearly a = 6 is point of minima
minima
/ \2 2 r
When a = 6, b = 4 => (ab)-
' 'min.
= 6 x 4 = 24.
,22 I x| x 2 x 10.(b) a + b + c + d + e+f+g + h = 8
5. (b) y -xy = y- — - — => y -xy is decreasing for y-~
k Lj 4 Leta + b + c + d = x
e+f+g + h=8-x
and increasing fory > — Lety = x2 + (8-x)2
2
.*. y = 2.x2 - 16 x + 64
Thus the largest value of y2 - xy must be at y = 0, — or 1. = 2[x2-8x + 32]
=2[(x-4)2 + 16]
x* •■•>’min = 32whenX = 4
The values are 0, —, 1 - x for x g (0,1)
11. (d)
X2 i
And for xe (0, 1) g(x) = max —»l~x
.4
Also x2 + 4x-4 = 0=>x = -2±V8
2
=> g(x) = 1 - x for x < V8 - 2 and — for x > V8 - 2
4
=> g(x) is minimum at x = V8 - 2 and minimum value
is 3-78 Perimeter of a sector = p. Let AOB be the sector with radius r.
6. (d) Let a < b and/(x) = lx - al + lx - bl Vx e R If angle of the sector be 0 radians, then area of sector
So,/(x) decreasing in (-°°, a] constant in [a, b] and increasing (4) = |r2© (i)
in [b, oo).
S.48 Calculus
Further fx) > 0, V x e R.
Length of arc(s) = r6 or 0 = -. From this information, graph of/(x) is as shown below.
r
Therefore perimeter of the sector
p = r + s + r = 2r + s

Substituting 6- — in (i), 4 = -r'

o4

p = 2r+(^)
r 2
1 ■,

KHra
=> s = —. Now substituting the value of s in (ii), we get

or 2A = pr - 2r.
(ii)

0
V +2
Differentiating with respect to r, we get
e2
9^ A Thus, three real distinct solutions k > — ,ke I.
^T = P~4r 4
dr
For maximum value of area So,’ kmin = 2.
13. (d) /(x) = 3X2 - 3 = 3 (x2 - 1)
— = 0 orp-4r = 0orr = — /(-l) = 3J(l) = -l,X3)=19>0
dr 4 Graph of/(x) is as shown in the following figure.
12. (b) Given equation is e' = kx2

=>
x2 3

Now let_/(x) = ~,
x2
1
Z(x)= (X 2)t?* —<-------- h + 4-
x -3 - 3
fix) = 0 => x = 2, which is point of minima.
e2
Also/(2) = 1- -2-

lim — = °° (using L’Hopital Rule twice) fx) = 0 has root xp x2, x3 such that

=> fix) - xt has only one real root since, xf< -1


lim 4 = 0 fix) = x2 has 3 real roots since -1 < x2< 1
x
fix) = x3 has 3 real roots since 1 < x3 < 3
ex
lim—7 = 00 The roots offlfx)) = 0 only if fix) = xrfx) = x2,fx) = x3
*-»ox2 Number real roots offfix)) = 0 is 7
Solutions S.49

CHAPTER 7

DPP 7.1 x26(x-l)17


k
(45x-27)

= 9^26(x-l)17 (5x - 3)
Single Correct Answer Type k
=> k=9
cos 5x + cos 4x ,
l.(d) J ------------------ dx
6. (d) J
x + (cos-13x)2 x (cos" 3x)'tdx
1 - 2 cos 3x
9r x
Jl - 9x2 7>-9x2 71-9x2
2 cos—cos- dx
2 2 = __L271-9x2-i(cos",3x)3+C
1 — 2| 2 cos'
/. A, = —1 ,Bn = —1
3x 9 9
2 cos-| 4 cos.33 —
3x - 3 cos —
=J—1 ■2___ 2■ dx 7.(0 Let/9 = flan9xdx = Jtan7x(sec2x- \)dx
- . 2 3x
3-4 cos — tan8x
2 i.e., i9 = -11
x
8
= - j2 COS-y-COSytfr = - jCOS 2xdx - |COS xdx
2 tan6x
Similarly, /7 = -/5
( sin 2x 6
+ sin x + C
2

2. (a) sec x cosec x


/ = | ------------- dx
2 cot x - sec x cosec x tan2x tan2x
l3 = ~li = - f tan x dx
-J 2 cosdx2 x -1 2
tan2x
2

+ logjcosx|
= fsec2xctr 2
= ^ln |sec 2x + tan 2x| + C /9 = (a polynomial of degree 8 in tan x) + log Icos .rl + C
(ex + cos x +1) - (ex + sin x + x) dx,
8. (b) /= J ------------------------------------
eT + sinx + x
3.(0
= log^ (ex + sin x + x) - x + C
fix) = e' + sin x + x and g(x) = -x
= f—(In x - In ex)dx
Jx fix) + g(x) = e* + sin x
r In x - x , 9. (b) Put x = u6
= I--------- dx
x + m3 +1
= f—Inxdr - f-xdv
-. Z = 6j— du
1 + M2
/x J X
du
= —(In x)2-x + C
= 6 Jw3 du + 6 j
1 + u2
3m4 z i z.
cos” X , f „_i -> , = — + 6 tan-1w + C
4.(0 j ------ dx = J cot x cosec’x dx 2
sin x
2
-cot"x 3v’
+C = — + 6 tan x~b +C
n 2
< >
5. (c) Differentiating both sides gives
3x2 + ex + xex - 3x2 - 3ex
x26 (x-l)l7(5x-3)= -[x27- 18(x- l)17 + (x-l)18 27.r26]
10. (a) J- e*(x-2) dx =| ----------------------------- dx
x(x2 + ex) x(x‘ + ex)
k
x26(x-l)17
k
[18x + 27(x-l)J =J d(xx33++xe
xex
x) ^rcZr
J x
S.50 Calculus

' = In (x3 + xe1) - 31n x + c sin(4x- 2x)cfc


4.(b) J
x2 + ex sin(2x)cos(2x)cos(4x)
= ln + c = .I, ^1 +c
lnl + -y
V X J r sin(4x)t£r - J sec 2x dx
J sin(2x)cos(4x)

DPP 7.2 2 f cos 2x dx


■* cos 4x
- y(log | sec 2x - tan 2x|)

2 r cos 2x dx
J1-2 sin2 2x - y(log |sec 2x - tan 2x|)
Single Correct Answer Type
dt

l.(b) /=|px, 2 sin(x2 +1) - sin 2(x2 +1)


= J 1-2/2
- ■~(log |sec 2x - tan 2x|)
dx
2 sin(x2 +1) + sin 2(x2 +1)
5. (c) Z = Jx2ln(x2+1)tic
x2 + 1 = t=>2xdx = dt
Let x2+l=/;=>xdx = —
, 1 r 2 sin t - sm 2/ , 2
Z = - ---------------- dt
2J y2sin/ + sin2/
Hence/= -f2ln'rf/
2
4/ If
12-2cos/ .
------------ dt
2 + 2 cos t
f J
=±h
2
tal<*
=— tan—dt
2J 2 1 .In 24*1
=-•------ +c
In sec- 2 ln2 + l
_1_ ____ 2_.
+c 1 (x2 + l)''—C
2 + 2’ ln2 + l
2
x2jM
6.(d)
= In sec +c
2
J sin• (sinx
x—
n
dx

cosec x I 4.
2.(c) / = [ ■>(, , x
dx
cos* 11 + log tan- n
sin u + —
k___ 4 du K

, 1 X
=J sin u
put u = x~—
Let l + logtan- = /
n . n
sin i/cos — + cos u sin —
—!—dx = dt = f--------*------- -±du
sin u
sin x
1 1 , , .
dt = -y^w + -^log|sin u| + c
/2
2
COS t
1
= x + -j= log(|sin x - cos x |) + c
= tan t + c 2
I, , x
= tan 11 + log tan—l + c A = -^,f(x) = x

dx x2dx
3. (b) j x-^/x6-16 cosx-xsinx ,
7.(0 / = /■—
X■3-Jx6-16 , dx
y/x cos X
■ dt
(putting x3 = 0
tyjr-16 Put x cos x = t2
=> (cos x - x sin x)dx = 2t dt
=±j-
48J t
dll
=> / = |^ = 2/ + C = 27xcosx + C

4V16
2x+l 2x + l
1 t
= —sec- + c 8. (b) J- (3/2 f dx
(x2 + 4x + 1)
12 4 \ 4 1V/2
1 x3
x3 l + - + -y
X X J
= —sec---- i-c
12 4
Solutions S.51

2xz + x
-J 1 dx So z=j7x+7*^ + 2 dx
if'2
1 + — + —=
x x ) =i2 Jf?'2 1 + 4 P
t J
XT 14,2 dl=x-tm 2
Now put -y + — + 1 = r
X2 X 3
_______ 2x + 3_______
9.(b) 7 = J- 2. (a) 7 = J x2 + l
dx
(x2 + 3x)(x2 + 3x + 2) + 1
x2-x
Put x2 + 3x = t => (2x + 3)<£r = dt

— -r- * ------------- dX
1
7 = J----- ----;f—^y = C-- — = C- 1I /I
x + 2— J—x-1
.
Jr(/ + 2) + l J(r + 1)2 /+1 x.22 + 3x + l x
=> /(1) = 5
'x + 2--
Lett =
e fsinx x
10. (b) e>^=r=»
’ ccsxdx = dt
2^/sin x
1 1
dt = —I 1 + — |ctr
=> e'^ y/cotxyjcosxdx = 2dt x2
2.IX + 2--
V x
Hence J2dt = 2t + C= 2e^ + C

11. (b) 7 = J dx
. Put X6 + 1 = V2
/-J ™
^6
xdxb +1
= 2s in-1 t + c
=> 6X5 dx = 2v dv
iJ-L^-dv 'x-- + 2+c
= 2 sin
3J(v2-l)v X

1,
1 r dv 1___
+C 3.(0 /=f-.2 ==-<&
'PTTi o log
= 7
X 716-x; .2

dx Putx = -
12. (a) J. t
(1 + ^)2010
Vx 1 .
--- ^dt
-J 77(i+ ^)2010 dx dx = -\dt:.I=[
J -1 X
r _f dt
f yl\6t2~l
X t

1 1 Let 16/2 - 1 = u2; 32/ dt = 2u du;


=2 +c
_2OO9r2009 2oo8r008 j u j. t udu
I fudu 716-x'.2
tdt=—du:.I=---- ------- = -- + C -+ C
=>> a = 2009,/? = 2008 16 16 J u 16
■ t/(sinx + cosx) 1
13. (a) -t= tan ’(sin x + cosx) + B
V2
^2 J•* (sinx + cosx)2 + 1 72 3(tanx- l)sec2x
4.(c) 7= J ___________ — dx
„ , ,^„22x +tanx
(tanx + 1) -Jtan33 x+tan

DPP 7.3 Put tanx = t


1-
1

M ----------- Z.- - n dt

Single Correct Answer Type


r2+;jr/+i
1. (b) Put yjx2 + 2=t
Put t + - + 1 =z^
t
x2 + 2 = z2 + x2-2tx
I = 3.2 tan-1 (^cotx + tanx +1) + C
1 2
=> x = —| t-- => K=6
2 t.
S.52 Calculus

5. (a) Let x = sin &, dx = cos Odd 2


. j r (cos 0-sin 0) cos OdO 9. (c) x4yla22 + x.2‘ = x5 I +1
J cos 0(1 + sin 0 cos 0)
_^r (cos0-sin 0)r/0
J1 + (cos 0 + sin 0)2
'=J- X2
= 2 tan-1 (cos 0 + sin 0) + c
= 2 tan*1 (x + -x2) + c
T a2 -2a
Let —7 = /, so —dx = dt
3x2 + 2x x x3
6.(b) /= J (x3 + x2 +1)2 + 4 dx
z+1-1 ,
7 = —[-4= dt = ~ f ■ -dt
Put x3 + x2 + 1 = t 2a4 JT+T 2a4 J Jt + i

'=J =T [777T _1(/ +1)3,2 +c

1
=—tan
2
1
=—tan
( x3 4- x2 + 1
a- 1
a4
/ 2
a +x
X
2
1 / 2 2U/2
“T-y(a +x )
3xJ
+c

2 I 2 10. (b) 1 = Jx24(l + x + x2)6 {x3(4 + 5x + 6x2)}<&


1
7. (c) Let j ■ dx = J (x4 + X5 + X6)6 .(6x5 + 5x4 + 4x3)dx
'x CT?
= x28(l + x + x2)7
Putting x = sin21 and dx = 2 sin t cos t dt, we get +C
7
_r 2 sin t cos tdt
(1 + sin z) ^/sin2 f — sin4 / 11.(b) dx
J(logx)4-x4
=> /=2J 1 + sin1 t dt = 2[-----
1 - sin t ,
—dt
cos*/
1 - log X
4
dx
(logx
= 2j(sec21 - sec t tan f)dt
x2
I X
I "I
= 2(tan t - sec t) + c
1 - log X
1 Put lpg-r_. = dt
7=2 +c X x2
dt
5x8 + 7x6 J(?-l) J (r2 + l)(/2-l)
8.(b) /(x) = J-
_11 f(/2 + l)-(/2-l)
x1 '<244
\ X X "2J (/2 + l)(/2-l)
5 7
= _ f dt 1(1, /“I -1
x6 x8
-J 2 + l + l —dx ”2|Jz2-l \2 + l ----- In-------- tan t
2^2 7+1

n 1
X

1
X .
= —In
4
In x-x
Inx + x
1
— tan
2 \ X J
->M+C
Put,
2+7+7=' 12. (b) We have J dx
=> (x2 + l)(x + Ityx3 + X.22 + X
x(x2-l)

/
-J J .2

/
x7
2x7 + x2 +1 ?
+C X3
1-4] .^dx
A0) = 0
1
x+- X
1 V lx + -iT
+1
c=0 XA vx X
/
x7
/(*) =
k2x7 + x2 + l
=J x + -1 X+X + 2WX + X+1
dx
=> /0) = 7 X
4
Solutions S.53

2'
--------------------j=dt Single Correct Answer Type
(t2-\\t2 + \^t2
dx3
u 1 . 2
where x + - + l = f ?(x" + l)
x x2dx
f 2
= —5------- 5----- dt
=4?(x"+l)
J(f2-lX'2 + l)
=4-^-Jx(x" + 1)
= f-f— dt- f“T— d‘ f x"-‘ ,
Jt2-1 Jt2 + 1 = ------------ dx
Jxn(xn + 1)
/-I _ 3 r dt
4'°g f+1
-tan" t + c n' f(/ + l)

1,
L+-+1-1
X 1 ,
41 /(/ + !)
- tan 'x + — + 1 +c
' =2 °8 Ix + —i-+ 71 +1, X
3 ri 1
---------- L
1 \dt
n t f+1J
X
= - (In t - ln(r + l)) + c
n
3, ( t
=—In ----- + C
n f+1
DPP 7.4 4. (b) J (x + 1)2
x2 + 1 + 2x
dx
x(x2 +1) x(x2 +1)

Subjective Type x2+l


-I x(x2 + l)
dt + 2j ---- 5----- dx
x(?+I)
edx . r dx
x4 + X* + 1 = -+2 —— = loge x + 2 tan 1 x + c
J x J x2 +1
x2 + l
-J x4 + x2 + l
dx - [ , 2x
",—dx
Jx4 + x.22 + l
5. (a) /=|j
2Jl + t3
dt (where f = x2)

-2 1 (2z-1)
1 + “7
2x 3(l + f)+3z2-/ + l \dt (After partial fractions)
2
=J (x-‘f+3
_______ X
----- dx
if3 x4 - x2 +1
x +- + - = — In +c
\ xj 2J 4 (x2+l)2
6
1 _|X—l/x 2 2x2 + 1
= -j=tani —f=— —r^tan +c x3-l
V3 y/3 V3 73 6.(c) J-(x4 + lXx + l) 1 (x4+lXx + l)
dx

x2-4
2. •4
~ Gv.21 ■ dx
j.
jx4 + 9x2 + 16
= Jf_fL f—— dx
x4 + 1 dx- J x+1
Divide both numerator and denominator by x2
= -ln(x4 + l)-ln(x + l) + c
4

/=H - „ 16
x2 + 9 + -j
H)
4Y i
dx
cos3 X
7.(b) 7 = J-—- ------------dx
sin x + sinx
x + - +1
x.2 xj r cos x • (1 - sin2 x) ,
= —7------------------ dx
J sin x(l + sin x)
4 t 4 )
Put u = x +—,du = \ 1—- cfr Put sin x = t, then cos x dx = dt
X \ X J

du => f f(l-/Xi + O<a


= tan \u) + R J '(1 + 0
u2 + l
=/ (y-w
x + -| + C t
= tan
xj

U J
S.54 Calculus
= log,Id -t + C
- log, Isin xl - sin x + C = -2j (! + ')’
1 1 x 1 -2
8.(b) dr =jl 1- — + -5— dx = -2je' + (1 + /)3 dt
J x +x 1+x2 X X2 + l (1+/)2

= ~^T + C
= x - tan_,x - log,lxl + log, (x2 + 1) + C
(1 + /)2
= -2euu,2x cos4 x + C
Comprehension Type
r log,(l + sin2x)dr
9. (d) f(ftx)-2g(x))dx
3.
J cos2 x

ex(e2x-2) sin2x + l-l


= log, (1 + sin2x) ■ tan x- 2 j dx
dx put ex = Z l + sin2x
e4x + 8c2x + 4
( 2^| sec2 xdx
z+- = log, (1 + sin2x) • tan x - 2x + 2 j
. I l + 2tan2x
=[ 2
2 —dt = -tan
2 \ 2 )
—L +C
= log, (1 + sin2x) • tan x - 2x + VI tan' (VI tanx) + C
Z+- +4
t
xlog,x
4. dx
(x2-l)3'2
10. (b) jtx)+2g(x) = J dx pulex = t
J (Tx+8rx+4 x x
= log,xf dr
( 2>
(AF (Z^F2 dx
t-~
, i
dt =—7= tan __t_ + C =^fd+f ■.1 -dx

2? +12 2^3 Ux/3 J xJx2-\


tj
= ^d + sec-x + C
Jx^\

DPP 7.5 Single Correct Answer Type

5. (a)
J (2 + cos xf
Subjective Type
(2 +cos x)cosx + sin2
1. /cos 2r log,( 1 + tan x) dx -J (2 + cos x)2
—dx

2
= - sin 2x log,(l + tan x) - | j■ sec2 x -sin2x
sin 2xdx = [-C0SX <fr_f ------------ 7 dx
2J (I + tan x) J 2 + cos x J (2 + cos x)
sin x
.1 sin 2xlog^(l+tan x) - =---------- + C
2 + cos x
6. (c) /= J/-,(x)dr = /-‘(x) x-Jx ((/-,Xx))'dr
1 f2cosx + sinx-sinx
= sin2xlog,(l +tanx)-x+ dx Put/"1 (x) = t
sin x + cos x
I=r'(x)-x-SM-dt
x 1 rcosx-sinx ,
= y sin2xlog,( 1 + tanx)-x + - + - ---------------dx
I=rlM-x-g(t) + C
2 2J sin x + cos x = x-r,(x)-g(f1(x)) + C
7. (a) t = cot x => dt = - cosec2x dx
= y [sin 2x log,( 1 + tanx)-x + log Isinx + cosxl] + C 1 f eC°,Jt fi 2 2 tan x
I = —5— In cosec x +------- 5— dx
Jsin x^ 1 + tan x
2. Jsin4x-e'lan* x dx = 4f sin x cos x cos 2x etfln2 Xdx
/ 2 >
= 4j tan x • sec2 x • cos4 x ■ cos 2x e' t
= -Je'- ln(l + z2) + dt
= 2l—e1 dt
J(l + Z)2 1 + Z 1+?>
(Putting, tan2x = t => 2 lan x • sec2 xdx = dt) -2eco,x In | cosec x| + c
= 2ecolx In | sin x | + c
Solutions S.55

l-7cos2x , | sec2 x ' 7 11. (a) / = /(sin(100x + x) ■ (sinx)"dx


8.(0 J — ------ — = Iksin
• 7 ~ dx
sin x cos x x sin x = J[(sin(IOOx)cosx + cos lOOx sin xXsin x)"]</x
2
sec x , r 7 , , , = jsin(100x)cos x • (sin x)99 dx + Jcos(100x) • (sin x)1
—— dx ~ —7 ax = /. + /,
sin7x J sin7 1 2
i n
sec2x , sin(100xXsin x)100
A=J sin x " 100
-
100
f cos(l OOxXsin x)100 dx

tan x f tan x cos x



sm x
+7JJ sin8x dx + fcos(l OOxXsin x)100 dx

tan x sin(l OOxXsin x)


-'2
sin7x 100

Zl+/2 = ^-+c
sin7x Multiple Correct Answers Type
fix) = tan x
2x 12. (b,c)
9. (a) If tan x +------ z-y <dx I = J{(^ - e-2r)ln(et + - (e2' - e'2x)ln(et - r*) }dx
(1 + x2)2 J
1 2x = Jtlntdt- Ju In u du (where 1 = ^ + e~x and u = ex- e-r)
1
+-------2
1 + x2 1 + x2 + (l + x2)2 r2 t2 u 2 u2
=—inr-----------lnu + — + C
1 2 4 2 4
= eJ tan X----------- :
.2
+c
1 + X' 13. (b, d)
/(0) = 0=>c= 1 xex
/w - v I 7Z 1 I ~ J dx
^1)=e 7‘7 +1
\4 27

10. (d) J. ex(x-lXx-lnx)^


(Putting t = >][ + ex)

'-‘Lr
—I.In---- +C
=fiZ
J X 4- kx.
2 t+l

= 2xJl+ex -4(71 + ex)~21n yLL’-Uc


ex ex ex
= ln — |---------+ c Jl + r +1
kx x x

= —(x-lnx-l) + c
x
S.56 Calculus

CHAPTER 8

DPP 8.1 =>

Single Correct Answer Type


=>
3ar3 2bx2
----- + + cx =0
n2 ,2 . 3------- 2
1 n' 1 0
1. (a) lim —+ + (^2)3 + ... + —
n (« + l)3 8n => a+h+c=O
,2 ,2 n2
n2 n' n
= lim + (" + D3 + (^+2)3 + ...+ x -x/2 X
n—♦«> _(n + 0)3 (n + n)3 5. (a) j|cosx|dr= j |cosx|dr+ | |cosx|dr
n2 -2 -2 -x/2
= lim £ (n + r)3
n~***r-0 -x/2 x
n 1
= j -cosxdr + J cos x dr
1
= lim X- x3 -2 -x/2
"->“r=0«

I nJ = 1 - sin 2 + sin x + 1
1 i x
dx =_____ 1
______
-J (l + x3)~[
0
2(l + x)2
o
j (cosx|dr = 0

if 1
1-1 u8
214
1 - sin 2 + sin x + 1 = 0
sinx = sin2-2<-l
1 1 K\
2. (c) 5 = lim y,—■ No solution in 0, — .
«-»-r=)n .in nr I 2J
sin < — + —
[4 4n
6. (b)
1
dx
= /rj
{ . (n n
= 41oge(V2-l)
/ = je2,cedx
tX
Osin —x + —
14 4 0

Let =1
n ^l/a{iIn a +k }
1 o-A a
e
lim y
3. (a) n-»°o no+\ => 1 = jte' dt = (re' - e')* - ee(e -1)
*=1

sr-en
0
" 1
= limX1- tan 1 00
7. (b) 7 = J /(x)dr+ J /(x)dr
1 0 tan I
= J(x,/a + x°)dr lanl
0 = J cosxdr + 0
1 0
x(i/«)+i xa+1 = sin (tan 1)
+----- •
a+1
8. (a)
a o

=-2-+J-=l /= J0f— , A —
(x + l)V(x + l)(3(x + l)-2)
a+1 a+1

3 3
4. (c) j(3ax2 + 2hr + c)dr = J (Sox2 + 2bx + c) dr Putx+ 1 = -
t
0 1
1 r 1
dt
=> jOax2 + 2bx + c}dx+ J(Sax2 + 2bx + c)dx
0 1
'=J 1/2
= V2-1
Solutions S.57

3 • a
9. (a) Let/= Jx /(x2)dr 3. (b) 1 = J In (cot a + tan x)dx
0 0
Putx2 = / a
cos(a - x)
=> 2x • dx = dt = Jin dx (1)
1 9 1 0
sin a cos x
=> /=7J/«)<*=74=2
10 2 a
cos x
2x
1 = Jin c£c (2)
sin (x + /)-sin x sin a cos (a - x)
10. (b) lim f dx 0
/-♦o J
0
t
a
= f Inf—5— )dr
/
( ■ 1
Adding (1) and (2) we get 21
2k 2 cos x + - sm- i I sin2 a J
0
2 2
lim dt a
0
^/-40 / = - 2J Ln (sin a)dx
2k 0
= J |cosx|<fr = 4 = -2aln (sin a)
0
4. (d) Puty = 0=>/(x+l)=^x)
DPP 8.2 Thus/(x) is periodic with period ‘1’
10
J/(10-x)dx
0
Single Correct Answer Type J 10

I
= J/(10-(10-x))A
1. (b) 1= J(l + x),/2(l-x)?; ,2dx o
10
1 = J f{x)dx
=> 1= J(l-x)l/2(l + x)3/2<& 0
I
1 = 10j/(x)A
2/ = J (1 + x),/2(l - x)l/2[(l - x) + (1 + x)]<fc 0

X/2 z
1 2k . 2 | j
5. (a) m = J cos —sm x dx
2Z = 2 f x2 dx
3 J
0 '
x/2 /

/ = 2j^ x2 dx u = JC cos[ ZTT



f
- cos2 x | ar
0
I0 I 3 J
k
=—
=>
_2u= *?rcos (2— . 2x
k sin ( 2/r .2j
2 0L <3 > + cos t ------
3
COS X dx

2.(b) I = J x (sin2 (sin x) + cos2 (cos x))dx (1) k/2 f \


K
0 2u = 2cos—
J 3 -cos —- coszx ar 3
1 = J(/r - x) (sin2 (sinx) + cos2(cosx))n!x (2)
0 If K
I It 1 .
Adding (1) and (2) => u=—
2J0 cos —cos/ \dt [Put 2x = /]
3
X
2 x/2
X/2 / \
=> 21 = 2k J (sin2(sin x) + cos2 (cos x))dx = COS —COS/ dt

0
1u
0 J
lOOx
2 6. (a) /= J (tanx + tan 2x + tan 3x + ... + tan 10x)dx
I = k J (sin2 (sin x) + cos2(cos x))dx (3) 0
0
X
Period affix) = tanx + tan 2x + ... + tan IQxis K
2
=> I = K J (sin2(cos x) + cos2 (sinx))/& (4) / = 100j(tanx + tan2r + tan3x + ... + tan 10lr)<£r
0
0
Adding (3) and (4)
x Now.^x) = -_/[7r-x)
2 2 /=O
=> 2Z = 7r 2<fr=>/ = —
0 2
S.58 Calculus

7. (c) 11.(b)
x/2 x/2 a
7= j sinxsin2xsin3xsin4xdv= j cosxsin2xcos3xsin4xdr (i)
0 0
x/2
=> 21 = j cos 2x sin 2x sin 4xdx f-^-dr
= J^+l
o
. x/2
= —2Jof sin24x<fr
J 6-«w+i dx
1 1 - cos 8x ,
= — ---------- dx
2 Jo 2 J 1+MW dx
-n12
sin 8x
x--------- /(x)7>g(x)
~4 8 Jo 8 dx (ii)
l + 5s(x)
2013
8.(d) 1 = j (?••-l)(x-2)(x-3)...(x-2013)dr a
i Adding (i) and (ii), 2/ = j /(x) dx+ j f(x)-bs(x) dx
b b l + 6s(x) l + b8(x}
Using j/(x)dv = j/(a + b - x)dx a
a a = J fWdx
2013
j (2013-x)(2012-x)...(l-x) = -/ a
i .-. 7=j/(x)A,
=» 21 = 0=>l = 0 o
4 4 which is independent of g(x).
9. (c) f f\x)dx = - Jx/"(x) 12. (b)
1 1 Alt
4 4 j ln|13sin x + 3^3cosx|dx
Now 7 = Jx/"(x)dr = j(5-x)/''(5-x)dr o
/
1 i
4
n
= 4 jin 14 sin x+ tan 3vn dx
= 5j/"(x)<*-/ o k
1 »/2
= 8 j (In 14 + log|sin x|)dr
o
4 , = 4n (In 14 - In 2) = 4n In 7.
J/'(x)^ = -[/'(4) + //(l)]
13. (b) Given fix) +/(-x) = 2
Let/(x) = t
Now. <(x)=r(5-x)
=> fi-x) = 2-t
/'(x) = -/'(5-x) + c
=> /*(0)+r(5) = c=>c = 8
ri(/)+ri(2-r)=o
so /(x) +jf(5 -x) = 8 =>/(4) +/*(!) = 8 1+x J+x
7 = j f~i(t)dt = j f~'(2-f)dt
4
10. (a) Let 7= j ex sec2 xdx
K e2x-\
4
ex sec2 x
If/W = 7=0
e2x-l
sec2x
/(-x) = - Multiple Correct Answers Type
e 2x-l
_ ex sec2 x 14. (b, d)
' l-e2x A(x + y) = A(x)A(y)
exsec2 x A(0 + 0) = A(0)A(0)
A(0)=l
e2x-l Put y = -x, we get
= -/(x) A(0) = A(x)A(-x) (i)
fix) is an odd function.
7=0
Solutions S.59

A(-x)
5(-x) = 2(tan ’t)]” =-
l+(4-x))2 0

1 tan”1 Jex - 1 - —=
A(x) v 4
1 +— yje1-) = tan—=>
(A(x))2
A(x) 2. (b) Let log, x = t
\ + (A(x))2
/ = 108J [logxe']<*
= B(x)
1
Thus, B(x) is even.
log,X2 logfx 2iog,x
2011 2010 2011
7= J [rlogxe]i/r = j 0 dt + J Idt = log, n
J B(x)dx= J B(x)dx+ j B(x)dx
i i logex
-2010 -2010 2010
2010 2011 3. (d) Put i2 = sin 9 in numerator (Nr.)
= 2 J B(x)dx+ J B(x)dx X

0
0
2010
2010
2011
= J B(x)dx+ J B(x)dx
0
• '•4 r==d9
‘sin 9
Put i2 = tan a in denominator
X
15. (b,c,d) t 4
x
f(x) = |cos t cos(x - t)dt (i)
r--
^ = -12 J ^sin
'2 J
2a
I ,
da=-r=-l,
V2 1
0
X
4. (a)
= J-cos t cos (x - K + f)dt * x/a
-e,b!x
0
;—dx
a
x
K
/(x) = jcos t ■ cos(x + t)dt (ii)
X b_
Put - =
o a y

Adding (i) and (ii), we get y


a
ey - ea
n
2/(x) = J cos z(2 cos x ■ cos C)dt
=>
'-Ib
ab
o y
-e,x/a
n x/2
f (x) = cos x j cos2t dt = 2 cos x | cos.221 dt
=J-
a
x
—dx = -I

o o 27 = 0
7=0
zrcosx which is continuous and differentiable,
logs
2 5.(b) 7 = 7J 22
having maximum value nil and minimum value -id2. o a +x
Also/(x) satisfies all the conditions of Rolle’s theorem in [0,
aj- , a ,
2zr]. Put x = —,dx = —,dy
y y‘

DPP 8.3 I p2'


o log — (
VyJ -A
dy
, a4
- ^2
- a‘ + —
Single Correct Answer Type y‘

X r (log a2 - log y)
du n = I----- $——(~ay)
l.(c) f J a' + y
log 2
(T^F
Put e“ - 1 = z2 = log (a2)J-A-
la~+y H7 logy 0
, 21
i+/2 = log (a2)—I tan-1 -7
al o
S.60 Calculus

2logo n 1
21 = Let x = -
a 2 t
=>
j log a
1
2a o
r\Jt 'dl
3
x + x2
X /‘=J n6
6. (c) 1 = J-log, dx ” I 1 + -t j
1/3* x - x2 + 1 y
1 1 j 1J

A = 7J (l t4i
Letx = - =>dr = —^dt
t t2 dt = I2
+ r)6
/ X 0
1/3
1 ,
-1 + -=--1
7 = - Jr log,
3
r
1
t2
--T + ,l
t t2
J r Multiple Correct Answers Type
10. (a, b,c)
3
3fl fr-r2 + l
dt We have \e~xidx = —^~
Jo 2
31 z X + x2 - P
= -J-log, dx
1/3X ^-X2 + l, For Je"2? dx, put VIx = t.
0
=> 7 = -/=>Z=0
7.(c) Letx = r2 1 7 _,2 , 4n
41 P l~242'
4=j2z(l + r2 + r4 + ... + t2n -2) (1 + 3r2 + 5r4 + ...
0 "x2<ic,putx2 = r
For
+ (2n -3) t2"-4 + (2n - l)/2"-2)^ 0
1
= J2(r + r3 + r5 + ... + r2" - ’) (1 + 3Z2 + 5? + ...
o 1 X^
0
, *7 -O 1
dx = —
2J0
e dt = —
2

+ (2n - 3) i2" - 4 + (2n - 1 )& ~ 2)dt OO OO

2e~x2 dx = jx(xe~x )dx, integrating by parts,


For J x1-x
Lety = r + r3 + r5 + ... + r2n-1 o o
n

In = 2 J r dt = n‘
,2
*
1 -?
0 2 Jo 2i 4
11. (b,c)
e^[/(x + fl)-/(l+a)]
1+a t
x?e'dt
8.(b; =e
1 x { x i; x .1 1
x+a
c e'dt + Jo —t dt
Put x = - in second integral -e~a
0
t da1
'x+a
dx = —<4dt
r
=e J e‘ t dt
i+a
c -a t ey+a
7_f/WA+j U
t
(-dt)
I------
■dy (Put, t = y + a, dt = dy)
i x i
=
fW
‘ = J^4&+ J^4& = 6 1 y+a
1 X 1 X fe'dl
] t +a
.2
x vx ,
9.(d) Z, = J -------
o (l+x)‘
Solutions S.61

DPP 8.4 5. (c)

j /(x) s*n 1 dt = constant


o
Single Correct Answer Type Differentiate both side w.r.t. x
f(x) (1 - cos x) +fix) sin x = 0
1. (d) Differentiating both sides w.r.t. x, then
! sin x ,
---------- dx
^x2(l+x))x(2x + 3x2)=l /(x) J cos x -1

Atx=l=>/(2) = | In l/(x) I = -2 In sin — + In c


2
2x-8 c
2.(b) /'(x)>0=> log05 >0 => J{x) = 7T
x-2 x)

sin -
2x-8 2J
=> J[k) = 2 => c = 2
x-2 <

^-1<0 4
=>
x-2
di
=> X6 (2,6) 6. (c) Since, / is independent of x, so — =0
dx
Also ^>0 x < 0 or x > 4 /(xy) • y-/(x) = 0
x-2
7(x • y) • y =/(x)
xe (4,6)
2 2
(2\
Put, y = -;/(2) - =/(x)
3. (a) Z = limloge fl'r
- j(l + osin bx)f'
1 xdx 0r
— form
x x
0
\ o / /(X) = -
X
t \
j(l + a sin bx)'\dxdx (Using L’Hopital r4
=> |— <ir = 4 In e = 4
= logc lim---------------- Rule)
k'-»° t 7
a+1
f(1 + a sin bt)c"
= loge hm-i---------------- 7. (d) f{a)= / —l—dx
1 1 + x4

f _i_ i 1
| t = 0 at a = 0
= log. Iim(l + asin6t)osin4' l + (a + l)4 l + (a-I)4
>0
/"(0) < 0 at a = 0
= log,e"k /(a) has a local maximum at a = 0.
= abc
]^dt
4. (a) /(x)=j dt 8. (d) lim ---------
2
1 + /4’ jr-»o x-sinx
1 fx •>> A
/'(x) = d_
1 + x4
In [2, 3], apply mean value theorem to/(x)
dx
= lim — ,fl + ' J
jr-»O 1 - COS X
/(3)~/(2) =f(cy where c 6 (2, 3) x2
3-2
.. y/a + x
1 = lim—------- (by L’ Hopital’s Rule)
J(3)-0 = "°2sm2^
1 + c4 2
f \2
Now 2 < c < 3
17 < 1 + c4 < 82 = rlim—-r 1 - _ 2 •4
*-»o 2y]a + x . x
1 1 1 sin-
— >--------- T>-----
17 1 + c4 82
l 2)
= —t= • 1 • 4 = 1 => a = 4.
2>/a
— < /(3)< —
82 J ’ 17
S.62 Calculus

x i (x \3 1
From (i), we get a =-------
’•(«) = — J/(D<* 3-2e
• o x lo 7
13. (a, b, c)
f(/(r))’A=-L(g(x))3 X
0 x fix) = sin x + J/*(/) (2 sin t - sin2/) dt
Differentiating w.r.t. x, o
(/to)3 = -y 3(g(x))2g'(x) - -y(g(x))3 => fix) = cos x +/(x) (2 sin x - sin2 x)
x x (1 - 2 sin x + sin2x)/*(x) = cos x
xg'jx) x-g'to' + 2 = 0 cosx
=>
g(x) gto > (sinx-1)2
^ = lor-2 1__ +
/(x) =
g(x) 1 - sin x

xg'(x)=1 since/(0) = 0 we have c = -1


if
g(x) sin x
/« =
=> xf(x) = \fit)dt 1 - sin x
o
=> jf(x) +fx) =fix) Comprehension Type
=> fx)=l
or )f(x)+J(x) = -2fx) 14. (b), 15. (c)
f\x)_-3 X
f(x) = J (5 In (1+r2) -1 Or tan-1 / +16sin/)J/.
fix) x
o
log/(x) = -3 log X + log c
fix) = c/x3 /*(x) = 5 In(1 + x2) — 10xtan_|x+ 16sinx
fl) = 1 =*/(x) = 1/x3 (decreasing function) f\x) = 2(8 cos x - 5 tan-1 x)
5
10. (c) g(x) = j C-jfiu)du dt f"ix)--2 8sinx + <0Vxe (0,1)
1 + x2
o I J
So,/"(x) is decreasing V x e (0,1)
X

=> g'ix) = x2jfiu)du hence g'(l) = 0 f 5/r


i
f\x)>f\\) = 2 8cosl- —
k 4
<x \
g"(x) = x2fix) + j fiu)du ■ 2x n 5n
>28 cos---------
U > I 3 4
=> g"(l) = /(l) + 0 = 3
g'(l) + g"(l)-3 j . 5n} n
= 2 4------>0
I 4 J
Multiple Correct Answers Type So,/*(x) is increasing, for x > 0,fix) >/*(0) = 0
So,/(x) is increasing, for x > 0,fix) >/(0) = 0.
11. (a, c) X
7(2) = 0 So, f fit) is positive and increasing.
Also,/*(x) =fx1 - 3x + 4) o
/*(2) =/(2) = 0.
12. (a, b)
i
Put x = 0 => e° - a J f(t)e~'dt = 0
DPP 8.5
o
i . Subjective Type
(i)
o a 1. x^-'-x^+x2'’-1
= x"-,(l-x) + x2fl-|>0forx€ (0, 1)
=> J/(/)<ft = ex-Ge,2x 1 _ x
’’ — = e — e
2x
a 1fx"-l-x"
- x" ++x X'2B-1 J A
o J-------:----- z-------dx>Q
Differentiating we get o 1 + x"
=> fx) = ex-2eljc
Solutions S.63

i ( 1
1
=>
J
0 I 1 + x"
-a-x"’1) t&>o 5. (a) Let l(a) = J|x"-a" |A
0
i
r dx /. /(a)=f(a’-x")dr+f(x"-a")A=—a"+,-a"+—
=>
Joi77 >j(l-x"
o
-l)(Zr
J0 J a
n +1 n +1
d
=>
r dx =» —(/(fl)) = n(2a-l)a" => only critical point of
[7^7 >1- —n ;WneN da
/'(a)<0for aefo,—
in (0,1) is a = 1
2
Single Correct Answer Type
and I'(a) > 0 for ae , 1(a) is minimum for a = —
ff/2
2
2. (c) 4 = J (sin x + cos x)""1 (sin x + cos x) dx 1 for0<x<l
0
6. (d) /(lnx) =
x for x > 1
= [(sin x + cos x)"_| (sin x - cos x)Jq/2
k!2
Put log x = t
+ j (sinx + cosx)"“2(cosx-sinx)2dr => x = e‘
0 Forx>l;/'(/) = e/,/>0
n/2 integrating^/) = d + C;
= 2+ j (sin x + cos x)"-2 (2 - (sinx + cosx)2) dx 7(0) = e° + C
0 => C = -l (given 7(0) = 0)
n./n^2 + 2(n-l)I;,_
> 2^,t./„-0 2(n-l)/n_2 = 2 7(r) = d - 1 for t > 0 (corresponding to x > 1)
hence fix) = e* - 1 for x > 0 (I)
1 _______ I
again for 0 < x < 1
3. (b) 4 = Jx"71- x.22 dx = jx"“’ xjl- x2 dx /(log x) = 1 (x = et)
0 o
f(t) = 1 for / < 0
I
(l-?)3'2' .2 x3/2
fit) = t + C
= -xn + J(n-l)x"*2 (1-X—
) dx
fi0) = 0 + C
3 3
o o
=> C=0
= 0 +—jx"’2(l - x2)Vl -x2dx => fit) = t for t S 0
0
=> fix) = x forx< 0
r n~ 1 f 7. (c) Let
n-2 3 'n
Z25 = p sin25x/Zr
=> -2
0

=> 4 = ”-i sin24xcosx<&


l„-2 n + 2

=> lim-^- = l.
= 0 + 25p sin24 x • cos xdx
o
4. (c)
= (-e sin24 xcosx)|~+25 j e-x(24sin23 xcos2 x-sin25 x)dx
‘“dx o
o
= 0 + 25 J e-x(24 sin23 x - 25 sin25 x)dx
= xn — -jnx'.11-1 ---- dx o
-a
~a Jo o = 600/33-625/25
1 r x" n t
=“,im-^+-4-i
a 626—= 600
/23

r n i xn 8. (d) y=fix)
4=-4-1 v lim — = 0
a x->^eax => X =/‘,Cv) = g(y)
_n n- 1 dy=f(x)dx
— Ai-2 where y is 2 then x = 1 and y is 10 then x = 5
a a
_ n(n-l)(n-2) 5 io
4-3 //(*)<*■+ jg(y)dy
1 2
n!7 n\ n! 5 5
=— e dx=—IQ = —-r
o nl an an+i = ]7(x)<fc + Jx/'(x)dr
i i
S.64 Calculus
5
Z= J(/(x)+xf(x))dr =>
^^-^D^-XO-M))
i
=> 16-(g(-l))2>-6
= x/(x)|’=5/(5)-/(l) = 5-10-2 = 48
=> (g(-l))2<22
9. (b) Putting x =ft) =f dx =f(f) dt A(5) > /«(0)
2x g2(0)
1= Jr-f\t)dt (v/(7r) = 7rand/(2^) = 2^) /»(5)> + 3(0)-5 = 3
2
X
2x

J' •/(')<*
JC
Comprehension Type

I
2x
t2 13. (a) g(m,O= Jx^'O-x)1 dt
= 2nf{2n}- -cost
o
X

1 3jt2 Put x = —
= 4tr2 - n2 - -(4rr2 - n2) + (cos 2n - cosrr) = +2 1+J
0 n—I
1__
10.(0 Let/"’(x) = u g(m,») = J: (i+y)2
\dy
/(*) ia+jy
J 2x(b-f~'(x))dx =j y" dy
/(«) {(\ + yT+n
b
= j2/(w)(*-w)/'(w)rfw
a
b b
=J (l + x)m+n dx
0
-1(* - u)f2 (w)|fl + J f2(»)du 14. (c) Putting log,- =t
a
x
b
x = e~*
= J(/2U)-/2(o))A
n
a
dx
11. (a) Let/(x) = 2x3-9x2+ 12x + 4, 0 k
/'(x) = 6x2-18x+ 12 = 6(x-l)(x-2)£0forxe [1, 0
2]. = je-m,fn(-e-,)dt
fix) is decreasing in [1,2]
f2)<fx)<fV)
8<y(x)<9
1 1 1 0
-<■ ■
<-7“
^8 1
3 V2*3-9*2-*-12**4 J t"e~ydy (putting (m + l)r = y)
(m + l)n+l o
1 f , r dx
- 1 dx < I , ---- _ /(* + D
3i1 172?-9x2 + 12x + 4
3 (m + l)n+l
I 1 I
xw-| + x"-'
3* /8 15. (a) Z = J dx
o (l + x)n+"
12.(0 i i
2 x"'1
A(x)=
2
+3x3-5 = f— dr + J
J0(l+x)',m+n 1
o (l + x)ro + fl
dx

h'(x) > 0 = Z,+Z2


=> g(x) g(x) >-9jt 1
j i
i i In Z2, put x = -, then Z2 = J r"'1__
. yn+n
\g(x)g'(x)dx>-\9x2dx
"I 1 + -
o o k tj
(g(i))2-(g(Q)):
2
->-3(1-0)
-JJ(l + x)w+n dx
(g(l))2-16>-6 i
X
(g(i))2> io dx + J dx
o o 0
(\ + x)m+n J (i7x)m+"
Jg(x)g'(x)<Zr>- j9x2dx
X
dx = g(m, n)
0
(l + x)m+"
Solutions S.65

CHAPTER 9

3. (d) The graph of the functions is as shown in the following figure:


DPP 9.1 4--
= e'x =
3-
Single Correct Answer Type
2 •
1. (c) y = sin2xandy = cos2x
Solving sin2 x = cos2 x
x = k/4,3k/4
=F=----- b +■

Graph of functions is as shown in the following figure. -2 1 2 3

= cos2 X y = sin2 x
1 1
A= dx
o
■+-
n/4 n/2 3n/4 K = (eI + e"z){)
= (e + e’l)-(l + l)

From the figure, the required area =e+--2


e
f3n74 2 *7
= J /4 (sin x-cos" x) dr 4. (b) We have y = I cos-1 (sin x) I -1 sin-1 (cos x)
f3x/4 = y - sin"’(sm x) - y - cos"‘(cos x)
=- cos 2x dx = 1
J x/4

2. (b) y = x2 is upward parabola ..Ik,.


= |-(x-2/r)|-|*-(2;r-x)
'I 12
4x, x>0 5k
J = VIXI = 4^x, x<0
=—
2
The graph of the functions is as shown in the following figure:
-(t-H-t)
2- = 4^-2x
The graph of the function is as shown in the following figure.
y=(-*)1'2 1- y=xi/2
4k \
+ ------ 1— -f-
1 2 3z--

2^t
.2
^ = 2ff>/x-—2 V K--

2 K
=— X
3 3k12 2k

Kfl
S.66 Calculus
From the figure. Required area From the figure, bounded area
= Area of shaded triangle in figure l/rz \l/3 x/2 kI4
1 (n k2 Jy - J cos ydy + | COS ydy + J sin ydy
2 l2J 4 kh k!4 o
5. (a) («. 0) lies on the given curve K
-3y4'3
0 = sin 2d - VJ sin <1 - Isin y\"n_ + |sin y+ |-cos _y|£/4
4/rl/3 0
sin a = 0 or cos a = J3 12

u= — (as u>() and the first point of intersection with --n -I-II+1- '1 ♦ ’ +1
positive X-axis) 4 V22 V2
K'f>
3 , . 1 . 1
Required area A = J (sin 2x - 75 sin x)dt = — K - I + I---- 7= + 1----
4 V2 d2
0
x/f>
( cos 2x
=---------- +
\ 2 0

8. (b) y = sin2 x - 2 sin x = sin x (sin x - 2)


k 4 2M 2 )
y = 0 /. sin x = 0 x = 0, K, 2k
7 „
= — 2 cos a Also sinx-2<0
4
y > 0 if sinx<0
=► 4.4 + 8 cos a = 7
6. (a) Graph of the functions is as shown in the following figure: and y < 0 if sin x > 0
Graph of the function is shown in the figure:
3n72f

3-

2--
Ml I-
—i—
-3^2 -/z72 kJ2
3^2 2K
'K 3k/2 2k
-k!2
-1

The required area = The shaded area


K 2jc
From the figure hounded area A
= j(sin2x - 2 sin x)dx + | (sin2 x - 2 sin x)dx
= (Area of quarter of circle) - (Area bounded by y = sin x
o K
and x-axis between x = 0 and x = n/2)
_ ff(Jr)2 j K K 2ks
2K
-2 = — J(l-cos2x)dr - 2jsinxdr + J I l-cos2x -2sinx dx
4 2~
“0 0 K '
_(^t3-8)
2*
4 _l_ sin 2x . "Fir
1
x - —— + 2 cos x + -| x -
sin 2x
+ 2 cos x
7. (d) Graph of the functions is as shown in the following figure: 2 2 o 2 2

K- 1 2k -k
-k - 4 + +4
2 2

1 . /r 1 . K # _
= 14 - — \ + 4 + — = 8 sq. units
2
y = sin"1 x
9. (a) y = 2x - x2 is downward parabola intersecting x-axis at x = 0
y = cos x and x = 2
I—---- 1---- Solving/(x) = 2x -x2 and g(x) = x"
2
we have 2x - x2 = xn
x = 0 and x = I
1
A = j(2x-x2-x")dr = x2 xn+1 T
0
3 n+1 o
Solutions S.67

2
A = 2j I- r2-b Mdx
x~ +1
1

J„.r + I
= 4 • tan-1 C" = 4 ■ — = 2k
2
13. (a) y = xe'
y' = e~x- x e~ x = (1 - x)e x
= 1-1 y increases for x < 1 and decreases for x > 0
x = 1 is point maxima.
2
hence. Also lim xe'x = 0
3
=> n=5 y" = -e-x- [e~x - xe"r]

w, -2<x<-l = e-x[-l - 1 + x]

10. (b) fix) = ■ |sgn x|, -1<x<1 = (x - 2)e-x


l<x£2 For point of inflection y" = 0 => x = 2
The graph of/(x) is I
y
i 1 3 4
-1 2

O 1 2
2 2
Required area is A = J .re dx = -xe
2+4 o u
Area = x 1 = 3 sq. units
= (-2e"2)- (e-t)5
11. (b) Solving given curves
= 1 - 3e"2
x2 + 2 = 2 Ixl-cos ?rx
14. (c) 0 < sin2x < I => [-sin2xj = 0 or -1
=> x2 - 21 x I + 2 = -cos itx
=► (lxI - 1)2 + 1 = —cos nx But sec-1 (0) is not defined.
=> x = ±l (0.4)
i y=n
Required Area = J (x2 + 2 - 2 |x| + cos nx)dx = 8/3

x2-l =1-^-
12. (b) y = flx) =
x* + l x‘ + l => y = sec-1 (-sin2x] = sec"1 (-1) = n
x2-l 16-x2
y = 0, .*. =0 or x = ±l Solving with y = . 16-.r = 4^
x2 + l 4
Also function is even. x= ±274-^
y(0) = -1 is the minimum value of function. 2J4-K .
. . f 16- x 2
!t>- 1 8
2 Required area = I------ --it kZv = -(4-ff),3 -
lim 11 - =1 -2^ 4 3
X—>±ool x2 + l

The graph of the function is as shown in the following figure:

+
EZZ15. (b.d)
Multiple Correct Answers Type

—4- -+- z +- 4- +- / = J(6.v - 3x2kZv = 1^- “ = 3x2 ~ v' = ^(3 -x)


-3 -2 7i 2 3 4
S.68 Calculus

y
DPP 9.2
4 4
1. (c) Given J/(x)dr-jg(x)dr = 10
o o
A, = /(2) —/(I) = 4 — 2 = 2 units
A2 =/(2) -/(3) = 4 - 0 = 4 units 7'

A3 = /(3)-/(4) = 0- (-16) = 16 units (0,4) ’


■y=fa)
=> one value of a will lie in (3,4). i
Using symmetry, other will lie in (-2, -1) 2 1(2^2)
16. (a, b)
^3
y
I 1
O 2
^4
(4,0)X

EO
it_
(A( + A3 + A4) - (A2 + Aj + A4) = 10
6 A,-A2=10 (i)
4 4

x/2 Also jg(x)c&- J/(x)a!r = 5


Area OABC = jsin2xdr = l 2 2
o (A2 + A4)-A4 = 5
X/6 . A2 = 5 (ii)
Area OAD = [ sin 2xdx = -
o 4 /. Adding (i) and (ii), A, = 15
2. (a) C,: 4x2 +^y = 4^
v sin 2x is symmetric about origin, therefore,
x2 y2
it
(•.• area OAD = Area OEF) C.:^7 + ^-=l
C~~~6 1 n2 4

f . , , 1
n
Now, sin 2x dx = —
Curve C2:.y = - sgn I x — cosx
; 2
2
6 n
„ 1 -cosx, x>y
cos 2c = --
2
n
c- —
it
= 1°, x=—
2
3
n
c =—,—
it it cosx, x< —
6 3
2
17. (a, b) y
i i
■2-
Required area = j tan x dx = J tan(l - x)dx
o o
*-x
3- -3rf ^it!2 A2
2-

r- Area A ( and A2 are equal.


—b—
-id2 7 (i.
A 3)rJ2 => Required ratio =
it2-2
it2 + 2
3. (c) Rotating the system through an angle 60° in clockwise direction,
-2- the equations in the question become
y = logex,y = 0,x = 2
uni
Also required area = tan 1 - J tan 1 y dy
o
I
Solutions S.69

2- Thus, graph of the functions is as shown in the follow it I tgure.


31
2-

1 •

-4- + +■ +
-3 -2 1 ' I
3

Since after rotation the area remains unchanged


2 ■2-
Required area = j (loge x)dx
i -3f
= [■* loge *“*]? = I21Qge 2 - 2] +1 As shown in the figure area = 4 x 1 x 1 = 4
= 2 logr 2 - 1 sq. units 6. (d)
4. (a) Graph of the functions is shown in the following figure
cosx 05x<-
4-x2 2
y= Given fix) = \2 and/is pe> with
4 + x2 —----- I
----- X n/2<x<n
25y2 = 9x 2 )
period n
1 2
Graph of the function is as shown in the following iftiie

y=
3|x|-6 si1'
5 7T/4 ? ? ? ?
2,
I .2
Required area = Z^-Jxdx + 2 4-x' 3x-6 7—¥—I—Y—I—Y—I—K—I- »
dx
1 14 + x2 5 X' -n -id! 0^/2 2T 3tz/2 27r57z/2 3?r 7;t/2 4/r \

4 ■2
=-+
8 3x-l
dx
r
5 1 |4 + x.2: 5 n2
From the graph, the range of the function is 0. —
2 . 4
-]2
4 + 4. tan X 1 |3x2 It is discontinuous at x = nn, n e /.
=—
5 L 2j, 5l 2 j)
Area bounded by y = f(x) and the X-axis from 0 to nn for
4 + zr-4tan 7 ne N
=— X
5 2 10
= njf(x)dx
• -J- + —1 1Isq.umts o
zr-4tan
'x/2 x / v2
2 10J
=n
j.
f cos x dx +fl*-- x II dx = n 1 + —
f, n3
J
0
1 \2 J I 24.
x/2v '
5. (a)
Min. {1x1, lyl) = 1
Ixl = 1 if Ixl < lyl, graph of which is two parallel lines x = ±1 7.(c)
excluding for -1 < y < 1 /(x) = [x] + 7x-[x]
and lyl = 1 if Ixl > lyl, graph of which is two parallel lines
y = ±1 excluding for -1 < x < 1 0 + /x, O^X<1
Max.{Ixl, lyl) =2 'x —I, l<x<2
Ixl = 2 if Ixl > lyl, graph of which is two parallel lines x = ±2
'x— 2<x<3
for -2 < y < 2
and lyl = 2 if lyl > Ixl, graph of which is two parallel lines
y = ±2 for -2 < x < 2
S.70 Calculus
The graph of the function is as shown in the following figure: 10. (b) J(x) and g(x) are periodic with period 1.
6 I
.-. Required area is J[/(•*) - g(x)]cfr = 6j(Vx - x:‘2)dx = 2
oo

11.(b)
xy(x2-3x + 2)
x2-7x + 12
+-x
0 12 3 4 n-1 n xy(x-l)(x-2)
(x-3)(x-4)
Area for 0 < x < 1. f* dxdx
r
Jo
fi /— x(x-l)(x-2) >Q
Area for 1 < x < 2, Vx dx + 1 x 1 y>0, if
Jo (x-3)(x-4)
x(x-l)(x-2)
Area for 2 < x < 3, J Vx dx + 2 x 1 y<0, if
(x-3)(x-4)
Area for n - 1 <x < n, Jx dx + (n - 1) x 1
y>0, if xe(0,l)u(2,3)u(4.°o)
So total area = n D] y<0, if x e (-0) u (1,2) u (3,4)

2n n(/z-l) y2 < 16x is interior of the parabola y2 - 16x


"T+ 2
Region is as shown in the following figure:
8.(d) x2 + y2<100
=> Points lies inside circle or on the circle. 8 •
sin(x + y) > 0 7-
x + y e (0, n) u (2jt, 3ff)... 6-
x + y = c is equation of a line 5-
Required area is shown in the following figure: 4 ‘ y

4-
-1 5
-2\

-4-

-6-

-8-

From the figure, required area


Required area = Shaded region
- Half of the area of the circle = Half of the area of the region bounded by
= |>r(10)2 = 50>r y2=16xandx = 4

9. (b) The region is enclosed between two parallelograms of areas 4 64


x 2 and 2 x 1 2 3
The area = 8-2 = 6.
Solutions S.71

12. (b) Graph of the inequalities is as shown in the following figure: 2-1
Equation of Ly. y - I = -—- (x - 1)
3
or 2y-2 = x-l
2y - x = 1
1
=> Slope of L,= 1/2
—fr-
-3 Fi 3 Equation of L,= y - 3 = — (x - 2)

or 2y - x = 4

• -3

Solving y = x - 1 with the circle x2 + y2 = 5 we get Area under/(x) = 4


x2 + (x- l)2 = 5orx = 2 .•. Shaded area = Area of trapezium DEFG - Area under/(x)
Required area
1(5 7
- - + — x2-4
fl r_ 2k2 2.
=8^-.i.i+JV53 x2dx
=6-4=2

5 . .
= 4 + 8 -J5-x2 + — sin
2 (tf Comprehension Type

= 4 + 8|0 + — -1- —sin"


I
= 10/r — 4 — 20 sin
4 2 a 14. (b),15.(c)
a
A! = j(sinx-/(x))dr = l + (a-l)cosa-sino
o
Differentiating w.r.t. 'a' both sides.
2 sin a-/(a) = cosa-(a-l)sina-cosa
= 20 --sin -4
12 J5 .*. jfa) = a sin a
= 20 cos -4 fix) = x sin x
Solving y = sin x with y = x sin x, we have x sin x = sin x
= 20 tan -4 .*. x = 1 or x = m 2n
X

_1_ (")) A2 = J (x sin x - sin x)dx


= 5 K- tan = 7t- tan
2 i

13. (b) = (- x cos x)jT - j - cos xdx + (cos x)*


I
x= 1 (2, 3) E,
rO,7/2) = (x + cos 1) + (0 - sin 1) + (-1 - cos 1)
(1,5/2)
= (n - sin 1 -1) sq. units
A (3, 2)
I
(l.D x=3
n_
(l,0)G F(3,0)
2
S.72 Calculus

CHAPTER 10

7. (d) e~x (y + 1 )dy + (cos2 x - sin 2x) y(dx) = 0


DPP 10.1 (1 + y-1 )dy + p'(cos2 x - sin 2x)dx = 0
Integrating we get,
y + log^y + cos2x = c
Single Correct Answer Type y = 1 when x = 0
1 +0+ 1 =c
1. (a) y = C] cos 2x + c2 cos2 x + c3 sin2 x + c4 /. c=2
Curve is y + log y + e* cos2x = 2
= Cj cos 2x + ^-(1 + cos 2x) + y (1 - cos 2x) + c4 8. (a) The given equation

)+(q+£2._£1
2' (x2 - yr2)— + y2 + xy2 = 0
= I—C2
+— C}
+ c4 cos 2x dx
k2 2 2 2. i-y. i+x . .
= A + B cos 2x => —T~dy + —5-ctc = 0
y x
That means there are two independent parameter.
Thus, the order of differential equation will be 2. 1 1V fl1 . 11V A
=> ----rfy + -r + — dr = O
ar y2 y) x‘ xj
X)
2.(b) y =
bx + c On integrating, we get the required solution
x . f xA 1 1
y^-b—c log — = — + — + c
-x + - x y
a a
x 9. (d) ^ = 1
y =-------
Ax + p
Order is 2 ( dy
p where p = —
2
dx \ dx
d 2y In p = x + c
3. (d) y2
dx2
+ x2y2 - sin x = - 3x|
(4)’ p = ex+c
( ^ = kex
i2 Y
y..2 d-y + x2y2-sinx = -27x dx
<dx2 > y = kex +1
Here order = 2 = p Satisfying (0,0), So 1 = -k
Degree = 6 = 9 y = k^-l)
P < <7 m . dx x x*2 x , „
4. (d) Multiply by y'" 10.(a) —+ ,2 — + 1 = 0
Order = 3, degree = 2 dy y1 y
5. (d) (px +g)dx + (3y+f)dy = Q Putx = yy
1 . .■ px2 3y2 , . dx dv
Integrating £— + gx + -^— + Jy + c = Q —=v+y—
dy dy
=> p = 3 (for a circle)
v + y — + v2 — v + 1 =0
6. (d) ± = i_x2_/+^y dy

-y =(1 +V2)
=(!-?)(!-/) dy
dv _ dy
dy = (l-x2)r£r =>
v2 + l y
l-y2
Integrating
f dy tan' v + C = -In y
Jj-y2" J(1 — x2)tZx
x i .
tan'
11. (1+/|
1 +y' X3 y)
-log ----- =x~ — + C
2 611-yJ
i-yj 3 where C is arbitrary constant
Solutions S.73

11. (c) (1 -xy + xtyydx = x2dy


(1 - (xy) + (xy)2)dx = x2dy v = x2 + cory = — Inx + c, x>0.
Let xy = v. 2
15. (a, b)
v
y= -
X
\dx J
+2>'cotx7--/
dx
=0
dv
X--------- V
dy________
dx dy _ -2y cot x ± \j4y2 cot2 x + 4y2
dx x2
X dx 2
x2dy = xdv -v dx
So (1 - v + v2)dx = xdv - vdx dy
— = -y(cot x ± cosec x)
=> (1 + v2)dx = xdv dx
dy
=> J 1 +dvv2 _ Jtdxx => — = (- cot x ± cosec x) dx
y
tan-1v = log Ixl + log c => log y = -[log sin x ± log (cosec x + cot x)] + log c
tan-1 v = log Icxl sin x
v = tan log Icxl =—log
c(cosec x ± cot x)
xy = tan log Icxl
c(l ± cos x)
—---------------------------- --------- -------------------- ------------------------ ■ = log
sin2x
Multiple Correct Answers Type
f c A
= log
12. (a, b, d) I 1 ± COS X J
y = c sin x (i)
c
y=
•• — - c cos X (ii) I ± COS X
dx
From (ii)

(SJ = c2cos2x (iii)


DPP 10.2
y 2 Single Correct Answer Type
Putting c = —— from (i),
sin x
■ 1. (d) y'(t) + 2y(t) = 2e2/ is linear differential equation
Eliminating c from (i) and (ii), = y cot x I.E = e2‘
dx solution is y e21 = 2t + C
Given y(0) = 2
Squaring and adding (i) and (ii), y2 +
SP- 2 2
=>
2=C
>■^ = 20+ 1)
Putting the value of *c’ from (iii), y2 + secx Whenr= l,y(I) = ■—
e
13. (a, c) 2. (a) ^-ylog,2 = 2lin'
(cosx-l)log,2
x'=Ae~v + Be~’ dx
This is linear differential equation
— =-lAe"21 - Be~*
dt I.F.=e-‘°s'!^ = e-l«'2=2-'
When t = 0, x = 0
A+B=0 Solution is
(>)
2sinx
y 2~x = J 2"* 2SU1J (cos x - 1) log,2 dx
At 1 = 0, — =12
dt put sinx-x = r=> (cosx- l)dx = dt
12 = -3A-fi (ii) y2~x = log, 2 J 2' dt
Solving, we get A = -6, B = 6.
14. (a,b) y2~x = 2' + c
1 Ady y = 2x*, + c2x
v = 2smx + c2x
4x J dx
3. (b) yev dx = (y3 + 2xey)dy
=>
(J- dy 1
~dx~2^c
=0 dx y2 2x
— = —+—
dy e2 y
dy = 2x dx or dy= — dx
2x
S.74 Calculus

dx 2 y2
-------- x = —
dy y e>' y4 dx 3dx
-1 dv
— ------ F
cosx-p
I.F. = e-2lnv = -4 3 dx x — —
y
dv 3
_ ... 1 r v2 1 . — + — v = 3cosx
General solution x —- =----- ^dy + c dx x
y" i ey y‘
if-dr
I.F. = e Jjt = x3
Ar = e"y + c = -e-v + c
y solution v. x3 = 3jcos xdx + c
x = 0, y = 1 => c = e"1
=> x = -y2(e->-e-1) —r-x3 = 3 sinx + c
If y = 0, then x = 0 y
x3y“3 = 3 sin x + c
4. (a) ^-/(x).y = 0
dx 8.(d) — = xy3 + x2y1
dy
dy
— = fto<ir
y =>
x2 dy X
In y = J7(x) dx
e\fWdx d(-Ux) --(/)=/
}’i to = =>
dy X
Then for given equation, I.F = l.E=^*'4
Hence, solution is y.yt(x) = Jr(x) ■ yt(x)dx
,y*'* 4
-e
y=y^jjr(x)yl(x)<Zr + C Solution is ---- = yV/4 -4e>'4/4 -e1'4
x
7^) u , 1
*+^=x-|logx

wheny = -l,x= —
5. (a)
dx x dy 16
(log*)2 log* 1,
~d^~~~5
-logx
I.F = eJ * =e 2 = (e'Jog*) 2 = X2
9. (c) We have — = y tan x - y2 sec x
^logx dx
Solution is x2 ■y 1 dy I
=> — tan x = -sec x
i, y
-lOgX
or yr•22 =x+c rv • 1 ~1 dy dv
Putting — = v => —= —, we get
6. (b) We have, (1 + tan y)(dx -dy) + 2x dy = 0 y y dx dx
=> (1 + tan y)dx = (1 + tan y - 2x)dy
dv
dx 2
— 4--------------x = 1, which is linear differential
— + tan x. v = sec x which is linear diff. equation
=>
dy 1 + tany equation.
_ Jog secx
2r_^_
I.F. = =e 6 =secx
j p _ e ’ l+lan^
l+tan^ _ siny+cos^ The solution is
(fl i cos>-sin>’ vsecx = /sec2 xdx + c
dy
= e sinj'+cosy ' _ gX+logtcosy+siny) — sec x = tan x + c
y
= (cos y + sin y)ey
sec x = y(c + tan x)
So, the solution is
xe-v(sin y + cos y) = Je3, (sin y + cos y)dy + c
i.e. xe>(sin y + cosy) = ev sin y + c.
i.e. x(sin y + cos y) = sin y + ce~y. 10. (a, b, d)
dy
7.(c) x2y-x3—=y4 cosx — + y = f(x) is linear differential equation.
dx
I.E = e
1 dy 1 cosx
y4 dx + y3 V Solution is ye1 = je* f(x)dx + C

1 now if 0 < x £ 2 then ye* = j dx + C


put — = v
y
Solutions S.75
=> ye* = x + C 12. (a, b, c, d)
y(0)= 1,=>C = 1 y\2xy + e*)dx - e*dy = 0
ye* = x + 1 (i)
x+1 y(2xy + e*)=exx^-dy

dy 2xy2
-r-y=—exr
XD=-e dx
I dy —I —---
2x
ex-(x + l)ex y2 dx y ex
Alsoy' = y' -
e2x
TX 1 1 dy dv
Put — = v=>— —
e-2e -e y y dx ~dx
/(!) =
7 e
dv 2x
Ifx>2 .-. ------- v = —
ye* = jex~2dx dx ex
dv _ 2x
yex = ex~2 + C dx ex
y = e~2 + Ce~x
As y is continuous I.F. = = e*

lim = lim (e~2 + Ce~x) Solution v. e* = j ——exdx + c


1 ex
3e~2 — e~2 + Ce~2 => C = 2 or — =-x2 + c
for x > 2 y
y = e~2 + 2e’x Curve passes through the point (0, 1)
y(3) = 2e-3 + e~2 = e-r2(2e-1 + 1) => c= 1
=> y'=-2e‘x Curve is y-1 e* = 1 - x2
=> y'(3) = -2e-3
11. (a, b, c)
X
or y=
rv
f(x) = j {f(j) cos t - cos(r - x)} dx Clearly lim /(x) = -» and lim /(x) = 0
0
X X
eJ(l-x2 + 2x)
= j f(I) cost dt - Jcos(-r) dt Alsoy' =
(1-x2)2
0 0
a a y' = 0=>x = 1 ±72
using Jf(x)dx = j/(a - x)dx
From sign scheme, we can check that x = 1 + 72 is the point
o 0
of local maxima and x = I - 72 is point of local minima.
X

/(x) = j/(r) cost e/r-sin x


o
Differentiate both sides w.r.t. x,
DPP 10.3
f(x) =/(x) cos x - cos x

or dy -ycosx = -cosx

dx
Single Correct Answer Type 1
1. (c) Given differential equation can be written as
_ g-sin x
xdy + ydx _ dx d(xy) _ dx
Hence, solution is y e~'-■sin x=.
-je"Iinx cos xcZr (jy-1)2 x2 (xy-1)
(xy-1)2 x2
or y g-sin x = £ .f. g-sin x '
y=CgSinx+ i
Integrating both sides
or
11 iII
At x = 0; y = 0 => C = -1 ---------- = — + c =?------- = — + c
^x)=l-es,nx (xy-1) x xy-1 x
xy-l
I
/min=1“e (whenx = zt/2) 2. (d) xdy + 2y3dy = ydx
and 4ax = 1 “ e~l (when * = _ ydx-xdy
f(x) = —€slnxC0SX => 2y dy 2ydy = dl —
y2
Z(O)=-1
Also f\x) = -[cos2x e',sinx — esinx • sin x] => x = y(c + y2).
xdx + ydy xdy - v dv
3.(c) ^2 +~~^
r =e .2
=0

1 d(x2 + y2)
0
~ y/x2 + y2
S.76 Calculus
y
=> Jx2 + y2 + — = c => xy = csin—
x x
4. (a) (3y.x~)d!.r - x3^)^ 2y4dy 10. (c) - x2y2 dx-ydx + xdy
yd(x^) - x3dy= 2y4dy
d{xy)
=> dx =
/-(xy)2
y
f 3A
= sin_,(xy)
d — = 2y2dy xy = sin (x + c)
11. (b) y dx + x dy + xy2dx - x2y dy = 0
2
3 ydx + xdy dx dy
y 5—5 h — V.
xy x y
x dy - y dx _ On integrating, we get
5. (c) dy
y2
1 . , . x 1 ,
---- + logx - logy = k log— = — + k
xy y xy
y1
12.(a)
x dx e2x + y2
—=y+c
y Dividing by e21
Atx= l,y= 1 c = -2 dy= y3e~2x
dx 1 + y2e~2x
Atx = -3, — = y-2
y dy + y2e~2xdy = y•3e<*2x‘ dx
y2 - 2y - 3 = 0
=> y=3 j2—+12(ye~2xdy - y'e
■2<'2tcZr) = 0
6. (b) The equation can be written as
2 x dy - y dx = x/-(y/x)2dx j2^- + jd(e~2xy2) + c = 0
x'
- y ■

d(y/x) _ dx 21n|y| + e"2ly2 = c


=>
/-(y/x)2 x
sin-1 y/x = log Ixl + c
Since y(l) = 0, c = 0.
DPP 10.4
Hence y = x sin(logld)
7. (c) The given equation can be written as
y( 1 + x"‘)dr + (x + log x)dy + sin y dx + x cos y dy = 0
Single Correct Answer Type
=> J(y(x + log x)) + d(x sin y) = 0 1. (c) According to the question
=> y(x + log x) +x sin y = C
dP
— = 0. IP, where P is population at time t.
8. (b) (x +1)-— +1 = ——•
dx '
dP
dy — = OAdt
e>(x+ 1) — + e> = ex P
dx Integrating we get logrP = 0.1/ + c
Let at t = 0, P = PQ
^^(^(x + 0)<& = J e*dx
log/o = 0 + c
logfP = 0.1/-log/0
e*'(x + 1) = e* + c
log/P/P0) = 0.1r
ydx + xdy _ cot^ xdy-ydx\ For double population we have
9. (c)
xy x ? J log/2P0/P0) = O.U
±rf(xy) = cot^)^] r=101oge2
2. (b)
y
fyI B
log xy = log sin — +logc
\x J 3
J’Uy)
o A x
Solutions S.77

3X y. -i
— + — = 4 is equation of tangent at P(x,y) having slope ——
x y x dy
dy 3y dy . n Putting this value in the given equation, we have
— = —— => x— + 3y = 0
dx x dx nx'1-1 — y = x"
dy
y x dy dx dx
Replacing — by----- , we have ny = -x —
log y = -3 log x + log c dx dy dy
c nydy + xdx = 0
=> y = — which passes through (1,1) ny2 + x2 = const.
x
c= 1 Which is the required family of orthogonal trajectories.

Equation of the curve is y = -y


r Multiple Correct Answers Type
3. (c) Clearly PQ = Length of tangent
6. (a, d)
If (x, y) is any point on the curve, the sub tangent at (x, y)
-
dx
Given PQ = 1 y dy

4*0 •'
2
dx
y— = nx (given)
dy
(dy}\ y2 dy dx
or n— = —
1-y2 y x
Integrating nlog y = log x + log c
(as curve lies in first quadrant)
dx or log y" = log ex
or / = ex (i)
Putting y = sin 6 which is the required equation of the family of curves.
nd9 -sin0 3”
cos0— =------- Putting x = 2, y = 3 in (i), we have 3" = 2c or c = —
dx cos0
Putting this value of c in (i),
cos26 ,
------- dd = dx
sin 0 . 3"
/= y*
sin 0---- -—]d0 = dx or 2/ = 3"x (ii)
sin0)
Which is the particular curve passing through the point (2,3)
Q
Putting n = 1 in (ii), we have 2y = 3x
-cos 0 - log, tan — = x + c
Which is a straight line
y = 1 when x = 0 Putting n. = 2 in (ii), we have 2y2 = 9x.
x = 0, 0 = n!2 c = 0 7. (a, d)
The curve in parametric form is Y C
Curve
x =-cos 0 - log, tan I — y = sinO
2/’
B
4. (a) Equation of tangent at the point
/f(x,/(x)) isr-Ax)=/'(x)(X-x) P(x,y)
Coordinates of point P are (0,/(x) -x/*(x))
The slope of the perpendicular line through ‘P’ is
I /(x)-xf(x)____ 1_ Tangent/1'7 0 (0,0) X
A
-1 /V)
Normal
2
=> = 1 is differential equation.
dx {dx) The equation of normal at P(x, y) is (T-y) = -~{X - x)
5. (b) Differentiating, we have
o'1-1 =nx"-‘ .( dy ,(„ x '
dx A x + y—,0 and 0,y + -r-
' dx' ' dy
dx >
S.78 Calculus

(/y Comprehension Type


1 x + y^ +2(0)
Now dx)
■ dy -
-x^x + y— = 3x
1+2 dx 9. (b) Equation of Tangent at P(xp jj) of y =flx)

dy = 2x (i)
=> y— (1) dx
dx
This tangent cuts the x-axis, where
y2 2
^- = x2 + C x-x__ Zl_
2
Also (0,4) satisfy it, so C = 8.
y2 = 2X2 + 16 (equation of curve)
v X!, x2, x3,... xn are in AP
Which represent a hyperbola
E y.
^2-^ = -^ = 10g2C

~dx
(0,4)
-y = (log2e)^
X' ----------- ar
(0,-4)
•*. — log2e = -dr
y
Integrating both sides
r log y - -x logr2 + log c
y = c 2-Jt
2(4) _ 2 v y =fi,x) passes through (0,2).
Also —
-1(4,475) 4^3 " y/3 2 = ce° =$ c = 2
8. (b,c,d) y = 2,-x
dx ■ y(5) = —
Slope of normal tan 9= - 16
dy
The given equation becomes at a general point (x, y) 10. (b) Again x2 = x( -
as
f
dx + ^
= *(? + !)
dy
^ = 1-
= ^x(y2 + l)
--■•’(a.
,2
xi

-(a I -xCv2+1)^-yr2=°
yy'2 - xy2y' -xy' - yr2 = 0
=> 1-—— = 2
x dy
_ydx=i
x dy
=> yy\y’ -xy)-x(y'-xy) = Q
dx _ dy
dy dy x
-v x y
j~=y~
dx =xy°Tdx
Integrating,
x2 log x = -log y + log c
log y = — + c(or) x2 - y2 = c
xy = c
Curve passing through (1,2) => c = 2
2. 2 2
y = ke2 (or) log y = x - log k Curve is xy = 2.

Also y = ke1 => log ky2 = x2


I
!

You might also like